Re: [QE-users] Request for Assistance: Positioning Metallic Element(Na) at Tetrahedral Interstitial Site (TIS) in Ag2O BCC Structure

2024-03-19 Thread Stefano de Gironcoli
if you look at the wikipedia page of Ag2O 
https://en.wikipedia.org/wiki/Silver_oxide


you see that the structure is actually SC in the sense that Oxygens 
occupy a BCC lattice while the Ag atoms form an FCC lattice.


in unit of alat the 6 atoms in of Ag2O should be in

ATOMIC_POSITIONS [alat]

O 0.25 0.25 0.25

O 0.75 0.75 0.75

Ag 0.00 0.00 0.00

Ag 0.50 0.50 0.00

Ag 0.00 0.50 0.50

Ag 0.50 0.00 0.50

I guess you want one of the 6 Silver terhahedra sites not occupied by 
Oxygens


any site with two 0.25 and one 0.75 or two 0.75 and one 0.25 should 
work. I think they are all equivalent


HTH

stefano

On 19/03/24 18:30, MOSES NTSIFUL wrote:
I want to model Na-Ag2O using Quantum Espresso, therefore I seek 
assistance in determining the atomic position of the metallic element, 
sodium(Na), at the Tetrahedral Interstitial Site (TIS) within the bcc 
structure of Ag2O. Could someone provide guidance on how to accurately 
position the element at the TIS?

Thank you for your help.

Best regards,
Moses Ntsiful

___
The Quantum ESPRESSO community stands by the Ukrainian
people and expresses its concerns about the devastating
effects that the Russian military offensive has on their
country and on the free and peaceful scientific, cultural,
and economic cooperation amongst peoples
___
Quantum ESPRESSO is supported by MaX (www.max-centre.eu)
users mailing listus...@lists.quantum-espresso.org
https://lists.quantum-espresso.org/mailman/listinfo/users___
The Quantum ESPRESSO community stands by the Ukrainian
people and expresses its concerns about the devastating
effects that the Russian military offensive has on their
country and on the free and peaceful scientific, cultural,
and economic cooperation amongst peoples
___
Quantum ESPRESSO is supported by MaX (www.max-centre.eu)
users mailing list users@lists.quantum-espresso.org
https://lists.quantum-espresso.org/mailman/listinfo/users

Re: [QE-users] What to modify in "dynamics_module.f90" for making constant zero pressure during NVT simulation(pw.x)

2024-03-16 Thread Stefano de Gironcoli

Dear Jayraj,

adding to what Lorenzo already replied, P and V as well as E and T are 
conjugated quantities: if you fix one the other fluctuates (of an amount 
\propto 1/sqrt N that vanishes in the thermodynamic limit).


so you can have NVE simulations where the average kinetic energy is 
related to the system temperature but fluctuates in time, or using a 
thermostat you can fix the system temperature (sampling NVT ensemble) 
but you give up energy conservation.


the same for V and P, you can have NVT and NPT, or NVE and NPH (because 
the conserved quantity is in this case the enthalpy). The larger the 
system the smaller the fluctuations but for finite number of particles 
the fluctuations are always present (and the thermostats and barostats 
do have their own degrees of freedom whose contribution become 
negligible in the thermodynamic limit).


stefano

On 16/03/24 13:56, Lorenzo Paulatto wrote:

On 16/03/2024 12:17, Jayraj Anadani wrote:

hello Paulatto,
but in cell_dynamics all options for pressure control are for vc-md 
and vc-relax. and i want to control pressure during 
ion_temperature=reduce-T option.


Dear Jayraj,

it is not very clear what you want to do. You can have constant NVT, 
or NVE or NPT, or NPE. But it looks to me that you want constant 
instantaneous P and V, which is impossible as P is a macroscopic 
quantity, it only makes sense on average.


kind regards




___
The Quantum ESPRESSO community stands by the Ukrainian
people and expresses its concerns about the devastating
effects that the Russian military offensive has on their
country and on the free and peaceful scientific, cultural,
and economic cooperation amongst peoples
___
Quantum ESPRESSO is supported by MaX (www.max-centre.eu)
users mailing listus...@lists.quantum-espresso.org
https://lists.quantum-espresso.org/mailman/listinfo/users

--
Dr. Lorenzo Paulatto
IdR @ IMPMC - CNRS UMR 7590 & Sorbonne Université
phone: +33 (0)1 442 79822 / skype: paulatz
http://www.impmc.upmc.fr/~paulatto/ - https://anharmonic.github.io/
23-24/423 B115, 4 place Jussieu 75252 Paris CX 05

___
The Quantum ESPRESSO community stands by the Ukrainian
people and expresses its concerns about the devastating
effects that the Russian military offensive has on their
country and on the free and peaceful scientific, cultural,
and economic cooperation amongst peoples
___
Quantum ESPRESSO is supported by MaX (www.max-centre.eu)
users mailing listus...@lists.quantum-espresso.org
https://lists.quantum-espresso.org/mailman/listinfo/users___
The Quantum ESPRESSO community stands by the Ukrainian
people and expresses its concerns about the devastating
effects that the Russian military offensive has on their
country and on the free and peaceful scientific, cultural,
and economic cooperation amongst peoples
___
Quantum ESPRESSO is supported by MaX (www.max-centre.eu)
users mailing list users@lists.quantum-espresso.org
https://lists.quantum-espresso.org/mailman/listinfo/users

Re: [QE-users] conflicting values for igcx

2022-11-22 Thread Stefano de Gironcoli

Dear Natalia,

You are using 2 pseudos generated with two different XC functionals.

The code does not know which functional to use if you don't give proper 
instructions.


3 options (the first is the correct one, the others are approximations 
you can use when you can't  generate new pseudos, for hybrids, for instance)


1- chose a functional (PBEsol, PBE or other) and generate both pseudos 
with the same functional. the pseudos of the ps library come with their 
input so it is easy to regenerate them with different functionals with 
the ld1.x code.


2- hack the file of the pseudo with the XC you DON'T WANT and write in 
its DFT field the functional description you WANT... this is an 
approximation because the potential unscreening and core states and 
charge where done with the "wrong" functional and you just hope it does 
not make much of a difference.


3- specify in the system namelist the variable input_dft='XC YOU WANT'. 
this overwrites the values read  from the pseudo files. it is equivalent 
to option 2 but it is less invasive. it is still approximated.


stefano

On 22/11/22 11:04, Levin Rojas, Natalia wrote:


Dear All,

My name is Natalia Levin, I’m a Postdoc at the Max Planck Institute 
for Chemical Energy Conversion in Germany.


I’m using QE V.6.7MaX and trying to optimize a supercell of Rh2O3 and 
I get this error message, even if I change different parameters:


 Error in routine set_dft_from_name (1):

  conflicting values for igcx

How could I solve it?

Thank you!

The input is here below:



    calculation   = "vc-relax" ! cell parameters and atoms relax

forc_conv_thr = 1.0e-03

    max_seconds   = 1.3e+07

nstep = 100

    pseudo_dir    = "/home/levin-rojas/pseudopot"

/



a =  4.7602

c   =  12.9933

degauss   = 0.05

ecutrho   =  361

ecutwfc   =  53

ibrav = 1

    nat   = 30

    nspin = 2

    ntyp  = 2

    occupations   = "smearing"

    smearing  = "gaussian"

    starting_magnetization(1) = 2.0e-01

/



    conv_thr =  1.0e-06

    electron_maxstep = 200

    mixing_beta  =  4.0e-01

    startingpot  = "atomic"

    startingwfc  = "atomic+random"

/



    ion_dynamics = "bfgs"

/



cell_dofree = "ibrav"

/

K_POINTS gamma

ATOMIC_SPECIES

Rh    102.90550 Rh.pbesol-spn-kjpaw_psl.1.0.0.UPF.txt

O  15.99940 O.pbe-n-kjpaw_psl.1.0.0.UPF.txt

ATOMIC_POSITIONS {angstrom}

Rh  0.0 0.0  4.575721000

Rh  2.38010 1.374151000  8.906821000

Rh  0.0 2.748303000  0.244621000

Rh  0.0 0.0  11.072371000

Rh  2.38010 1.374151000  2.410171000

Rh  0.0 2.748303000  6.741271000

Rh  0.0 0.0  1.920929000

Rh  2.38010 1.374151000  6.252029000

Rh  0.0 2.748303000  10.583129000

Rh  0.0 0.0  8.417579000

Rh  2.38010 1.374151000  12.748679000

Rh 0.0  2.748303000  4.086479000

O 1.457764000  0.0  3.248325000

O 3.837864000  1.374151000  7.579425000

O 1.457764000  2.748303000  11.910525000

O 0.728882000  1.26246  9.744975000

O 3.108982000  2.636612000  1.082775000

O 0.728882000  4.010763000  5.413875000

O -0.728882000    1.26246  3.248325000

O 1.651218000  2.636612000  7.579425000

O   -0.728882000 4.010763000  11.910525000

O   3.302436000 0.0  9.744975000

O   0.922336000 1.374151000  1.082775000

O   -1.457764000 2.748303000  5.413875000

O   1.651218000 2.859994000  3.248325000

O   1.651218000 0.111691000  7.579425000

O   -0.728882000 1.485842000  11.910525000

O   -1.651218000 2.859994000  9.744975000

O   3.108982000 0.111691000  1.082775000

O   0.728882000 1.485842000  5.413875000


___
The Quantum ESPRESSO community stands by the Ukrainian
people and expresses its concerns about the devastating
effects that the Russian military offensive has on their
country and on the free and peaceful scientific, cultural,
and economic cooperation amongst peoples
___
Quantum ESPRESSO is supported by MaX (www.max-centre.eu)
users mailing listus...@lists.quantum-espresso.org
https://lists.quantum-espresso.org/mailman/listinfo/users___
The Quantum ESPRESSO community stands by the Ukrainian
people and expresses its concerns about the devastating
effects that the Russian military offensive has on their
country and on the free and peaceful scientific, cultural,
and economic cooperation 

Re: [QE-users] How H2O molecule reflect DFT scf calculation at 0 Kelvin calculation

2022-08-28 Thread Stefano de Gironcoli

CP dynamics requires the existence of a gap in the electronic spectrum.

you can force cp.x to minimize the electronic state by using "cg" as 
electron_dynamics method.  I would use pw.x to perform BOMD but if you 
are more familiar with cp.x you can try that.


stefano

On 27/08/22 15:47, Jayraj Anadani wrote:

Hello sir,
I have one more question regarding this : can I perform the cpmd 
simulation method using cp.x in quantum espresso to simulate the 
metallic glass system ? I did this cpmd but I couldn't control the 
kinetic energy of the electrons because my system is amorphorse 
metallic and according to CP user guide the cpmd is not 
performing well with amorphous metallic systems. or is BOMD using 
pwscf by doing the reduce-T option is good for studying metallic glass ?


kindly suggest a feasible method to do DFT study of metallic glass 
alloys using quantum espresso package.


thank you

regards
jayraj anadani

On Fri, Aug 26, 2022 at 10:04 PM Jayraj Anadani 
 wrote:


I appreciate your help
Thank you very much sir.

On Fri, 26 Aug, 2022, 7:16 pm Stefano de Gironcoli,
 wrote:

I would take out small clusters from your 1 cells
(possibly saturate with H atoms if you cut covalent bonds),
not relax (with the exception of relaxing the extra H atoms
you added to saturate), compute atomic PDOS of the central
atom(s) and verify that for increasing number of atoms in the
clusters these quantities stabilize and converge.

if a significant size dependence results you may want to play
with the environ feature (http://www.quantum-environ.org/) to
minimize boundary effects

stefano

On 26/08/22 15:01, Jayraj Anadani wrote:

I have one amorphorse metallic system of 1 atoms in which
several clusters are in it. I want to calculate atomic
bonding and electronic interaction through finding DOS/PDOS
and Charge analysis of a small cluster part of a large
amorphorse metallic system.
My cluster size is upto 13 to 25 atoms which is part of large
md simulation.

So can i have to do relaxation or i can directly do scf
calculation with restricted parameter ?

Thank you

On Fri, 26 Aug, 2022, 6:04 pm Stefano de Gironcoli,
 wrote:

I dont think that a 10-20 atom cluster configuration
extracted from a 1 condensed phase has anything to do
with the same system at 300 K.

possibly some electronic properties of the central
molecule may be similar to the corresponding properties
in the condensed phase but no guarantee.

if you relax or perform md for the cluster you loose your
(tiny) connection with the original cell.

I fail to understand what is your problem and what you
would like to compute/understand

stefano

On 26/08/22 13:52, Jayraj Anadani wrote:

Thank you sir, for your reply
but I have one particular molecular system of 10-20
atoms which I am getting from a simulation of a large
atom (around 1 atoms) at a 300K temperature from a
molecular dynamic simulation (LAMMPS). and I also did
optimization of that MD structure which is
amorphous metallic glass. Now,
 Can I do dft study of a small part of a single molecule
of a large MD simulation(~1 atoms) ?
till now i did below things but i am bit confused which
way is best way to study DFT . i also dis AIMD/CPMD but
for metallic glass CPMD doesn't work good with PAW PP.
so,
1. Can I do direct SCF calculation(without relax) by
giving a condition of *forc_conv_thr* and
*tot_conv_thr* with creating a vacuum(for a non-periodic
system) of around 15 Angstrom to converge properly. *OR*
2. I should do the RELAX calculation with vacuum and
then go to the *scf* calculation to find the total
energy and then *nscf*.

***questions:
1. what if I relax a system and position is changed a
bit(which is obvious) to minimize the total energy but
that energy is at zero kelvin dft calculation and I want
to study the system at 300K. So how can small parts of a
structure reflect large MD simulation structural properties?
Thank you
regards
jayraj anadani
Ph.D. Research scholar Department of physics
sardar patel university
india

On Fri, Aug 26, 2022 at 5:00 PM Stefano de Gironcoli
 wrote:

Dear JAYRAJ ANADANI,

  calculation="scf"  corresponds to a minimization
   

Re: [QE-users] How H2O molecule reflect DFT scf calculation at 0 Kelvin calculation

2022-08-26 Thread Stefano de Gironcoli
I would take out small clusters from your 1 cells (possibly saturate 
with H atoms if you cut covalent bonds), not relax (with the exception 
of relaxing the extra H atoms you added to saturate), compute atomic 
PDOS of the central atom(s) and verify that for increasing number of 
atoms in the clusters these quantities stabilize and converge.


if a significant size dependence results you may want to play with the 
environ feature (http://www.quantum-environ.org/)  to minimize boundary 
effects


stefano

On 26/08/22 15:01, Jayraj Anadani wrote:
I have one amorphorse metallic system of 1 atoms in which several 
clusters are in it. I want to calculate atomic bonding and electronic 
interaction through finding DOS/PDOS and Charge analysis of a small 
cluster part of a large amorphorse metallic system.
My cluster size is upto 13 to 25 atoms which is part of large md 
simulation.


So can i have to do relaxation or i can directly do scf calculation 
with restricted parameter ?


Thank you

On Fri, 26 Aug, 2022, 6:04 pm Stefano de Gironcoli, 
 wrote:


I dont think that a 10-20 atom cluster configuration extracted
from a 1 condensed phase has anything to do with the same
system at 300 K.

possibly some electronic properties of the central molecule may be
similar to the corresponding properties in the condensed phase but
no guarantee.

if you relax or perform md for the cluster you loose your (tiny)
connection with the original cell.

I fail to understand what is your problem and what you would like
to compute/understand

stefano

On 26/08/22 13:52, Jayraj Anadani wrote:

Thank you sir, for your reply
but I have one particular molecular system of 10-20 atoms which I
am getting from a simulation of a large atom (around 1 atoms)
at a 300K temperature from a molecular dynamic simulation
(LAMMPS). and I also did optimization of that MD structure which
is amorphous metallic glass. Now,
 Can I do dft study of a small part of a single molecule of a
large MD simulation(~1 atoms) ?
till now i did below things but i am bit confused which way is
best way to study DFT . i also dis AIMD/CPMD but for metallic
glass CPMD doesn't work good with PAW PP.
so,
1. Can I do direct SCF calculation(without relax) by giving a
condition of *forc_conv_thr* and *tot_conv_thr* with creating a
vacuum(for a non-periodic system) of around 15 Angstrom to
converge properly. *OR*
2. I should do the RELAX calculation with vacuum and then go to
the *scf* calculation to find the total energy and then *nscf*.

***questions:
1. what if I relax a system and position is changed a bit(which
is obvious) to minimize the total energy but that energy is at
zero kelvin dft calculation and I want to study the system at
300K. So how can small parts of a structure reflect large MD
simulation structural properties?
Thank you
regards
jayraj anadani
Ph.D. Research scholar Department of physics
sardar patel university
india

On Fri, Aug 26, 2022 at 5:00 PM Stefano de Gironcoli
 wrote:

Dear JAYRAJ ANADANI,

  calculation="scf"  corresponds to a minimization of the
total energy
at fixed atomic positions. you chose where to put the ions.
the code
does its best to return the corresponding energy.

  calculaiton="relax" corresponds to a series of scf
calculations,
followed by forces evaluation and geometry update to minimize
the energy
of the system. The code does its best to converge to a
minimum (non
necessarily the global minimum) in the GS potential energy
surface.

  there is no temperature in any of these calculations.

  if you want to simulate the system at finite (classical)
temperature
for the atoms you should perform MD simlations

stefano

On 26/08/22 05:02, Jayraj Anadani wrote:
> Dear QE
>               I put H2O molecular atoms small far apart
from each
> other within large supercell so that the periodic image of
the H2O do
> not interact with each other and run a scf calculation my
convergence
> done successfully with good amount of accuracy upto 10^-9.
But it is
> not reflecting original H2O molecule at 300 Kelvin...but
when i
> relaxing these atoms and then did scf calculation they are
reflecting
> H2O at 300k by observing bond angel and length.. how ? And
my question is
> 1) scf is also doing calculation using iteration method
algorithm by
> using electronic densities at ground state so why they
converge when
> two hydrogen atoms and one oxygen atom are not at appropriate
> dist

Re: [QE-users] How H2O molecule reflect DFT scf calculation at 0 Kelvin calculation

2022-08-26 Thread Stefano de Gironcoli
I dont think that a 10-20 atom cluster configuration extracted from a 
1 condensed phase has anything to do with the same system at 300 K.


possibly some electronic properties of the central molecule may be 
similar to the corresponding properties in the condensed phase but no 
guarantee.


if you relax or perform md for the cluster you loose your (tiny) 
connection with the original cell.


I fail to understand what is your problem and what you would like to 
compute/understand


stefano

On 26/08/22 13:52, Jayraj Anadani wrote:

Thank you sir, for your reply
but I have one particular molecular system of 10-20 atoms which I am 
getting from a simulation of a large atom (around 1 atoms) at a 
300K temperature from a molecular dynamic simulation (LAMMPS). and I 
also did optimization of that MD structure which is amorphous metallic 
glass. Now,
 Can I do dft study of a small part of a single molecule of a large MD 
simulation(~1 atoms) ?
till now i did below things but i am bit confused which way is best 
way to study DFT . i also dis AIMD/CPMD but for metallic glass CPMD 
doesn't work good with PAW PP.

so,
1. Can I do direct SCF calculation(without relax) by giving a 
condition of *forc_conv_thr* and *tot_conv_thr* with creating a 
vacuum(for a non-periodic system) of around 15 Angstrom to converge 
properly. *OR*
2. I should do the RELAX calculation with vacuum and then go to the 
*scf* calculation to find the total energy and then *nscf*.


***questions:
1. what if I relax a system and position is changed a bit(which is 
obvious) to minimize the total energy but that energy is at zero 
kelvin dft calculation and I want to study the system at 300K. So how 
can small parts of a structure reflect large MD simulation structural 
properties?

Thank you
regards
jayraj anadani
Ph.D. Research scholar Department of physics
sardar patel university
india

On Fri, Aug 26, 2022 at 5:00 PM Stefano de Gironcoli 
 wrote:


Dear JAYRAJ ANADANI,

  calculation="scf"  corresponds to a minimization of the total
energy
at fixed atomic positions. you chose where to put the ions. the code
does its best to return the corresponding energy.

  calculaiton="relax" corresponds to a series of scf calculations,
followed by forces evaluation and geometry update to minimize the
energy
of the system. The code does its best to converge to a minimum (non
necessarily the global minimum) in the GS potential energy surface.

  there is no temperature in any of these calculations.

  if you want to simulate the system at finite (classical)
temperature
for the atoms you should perform MD simlations

stefano

On 26/08/22 05:02, Jayraj Anadani wrote:
> Dear QE
>               I put H2O molecular atoms small far apart from each
> other within large supercell so that the periodic image of the
H2O do
> not interact with each other and run a scf calculation my
convergence
> done successfully with good amount of accuracy upto 10^-9. But
it is
> not reflecting original H2O molecule at 300 Kelvin...but when i
> relaxing these atoms and then did scf calculation they are
reflecting
> H2O at 300k by observing bond angel and length.. how ? And my
question is
> 1) scf is also doing calculation using iteration method
algorithm by
> using electronic densities at ground state so why they converge
when
> two hydrogen atoms and one oxygen atom are not at appropriate
> distance.still scf converge how ?
> 2) also by doing relax calculation all of sudden two hydrogen
and one
> oxygen come closer and they formed a bond and bond angel which is
> reflect 300k H2O molecule structure. So how two hydrogen and one
> oxygen atoms knows during relaxation that i have to make
structure at
> a room temperature?
>
>
> Thank you
> Regards
>
> JAYRAJ ANADANI
> Ph.D Research scholar
> Department of physics
> Sardar Patel University
> Anand,Gujarat,
> India
>
> ___
> The Quantum ESPRESSO community stands by the Ukrainian
> people and expresses its concerns about the devastating
> effects that the Russian military offensive has on their
> country and on the free and peaceful scientific, cultural,
> and economic cooperation amongst peoples
> ___
> Quantum ESPRESSO is supported by MaX (www.max-centre.eu
<http://www.max-centre.eu>)
> users mailing list users@lists.quantum-espresso.org
> https://lists.quantum-espresso.org/mailman/listinfo/users
___
The Quantum ESPRESSO community stands by the Ukrainian
people and expresses its 

Re: [QE-users] How H2O molecule reflect DFT scf calculation at 0 Kelvin calculation

2022-08-26 Thread Stefano de Gironcoli

Dear JAYRAJ ANADANI,

 calculation="scf"  corresponds to a minimization of the total energy 
at fixed atomic positions. you chose where to put the ions. the code 
does its best to return the corresponding energy.


 calculaiton="relax" corresponds to a series of scf calculations, 
followed by forces evaluation and geometry update to minimize the energy 
of the system. The code does its best to converge to a minimum (non 
necessarily the global minimum) in the GS potential energy surface.


 there is no temperature in any of these calculations.

 if you want to simulate the system at finite (classical) temperature 
for the atoms you should perform MD simlations


stefano

On 26/08/22 05:02, Jayraj Anadani wrote:

Dear QE
              I put H2O molecular atoms small far apart from each 
other within large supercell so that the periodic image of the H2O do 
not interact with each other and run a scf calculation my convergence 
done successfully with good amount of accuracy upto 10^-9. But it is 
not reflecting original H2O molecule at 300 Kelvin...but when i 
relaxing these atoms and then did scf calculation they are reflecting 
H2O at 300k by observing bond angel and length.. how ? And my question is
1) scf is also doing calculation using iteration method algorithm by 
using electronic densities at ground state so why they converge when 
two hydrogen atoms and one oxygen atom are not at appropriate 
distance.still scf converge how ?
2) also by doing relax calculation all of sudden two hydrogen and one 
oxygen come closer and they formed a bond and bond angel which is 
reflect 300k H2O molecule structure. So how two hydrogen and one 
oxygen atoms knows during relaxation that i have to make structure at 
a room temperature?



Thank you
Regards

JAYRAJ ANADANI
Ph.D Research scholar
Department of physics
Sardar Patel University
Anand,Gujarat,
India

___
The Quantum ESPRESSO community stands by the Ukrainian
people and expresses its concerns about the devastating
effects that the Russian military offensive has on their
country and on the free and peaceful scientific, cultural,
and economic cooperation amongst peoples
___
Quantum ESPRESSO is supported by MaX (www.max-centre.eu)
users mailing list users@lists.quantum-espresso.org
https://lists.quantum-espresso.org/mailman/listinfo/users

___
The Quantum ESPRESSO community stands by the Ukrainian
people and expresses its concerns about the devastating
effects that the Russian military offensive has on their
country and on the free and peaceful scientific, cultural,
and economic cooperation amongst peoples
___
Quantum ESPRESSO is supported by MaX (www.max-centre.eu)
users mailing list users@lists.quantum-espresso.org
https://lists.quantum-espresso.org/mailman/listinfo/users

Re: [QE-users] syntax of using starting_ns_eigenvalue for two different localized orbitals of the same atom

2022-08-11 Thread Stefano de Gironcoli

suppose your Hubbard atom of type 1 is an Fe ( U on 3d orbitals)..., then

starting_ns_eigenvalue(3,1,1)=1.0  overwrites the third eigenvalue of 
the occupations with spin=1 (up) to 1.0


starting_ns_eigenvalue(4,1,1)=0.3 overwrites the forth eigenvalue of the 
occupations with spin=1 (up) to 0.3


starting_ns_eigenvalue(5,2,1)=0.0 overwrites the fifth eigenvalue of the 
occupations with spin=2 (dw) to 0.0


the other elements are left unchanged.

In general at the first iteration the code computes the occupation 
matrix ns, diagonalizes it, overwrites the eigenvalues if required in 
input and then calculates the output ns matrix from the (overwritten) 
eigenvalue decomposition.


if you specify more starting_ns_eigenvalue lines for a given atomic type 
the code will apply all of them independently


stefano

On 11/08/22 21:00, Mahmoud Payami Shabestari via users wrote:


Dear QE-7.1 Developers,
Hi. I would like to use the starting_ns_eigenvalue()=1.0 for two 
different localized orbitals of the same atom. I do not have problem 
in the HUBBARD card, but I do not know how to use 
starting_ns_eigenvalue()=1.0 for different orbital angular momenta of 
the same atom.

Any comments is highly appreciated.
Best regards,
Mahmoud Payami
---
Mahmoud Payami
NSTRI, AEOI, Tehran, Iran
Email: mpay...@aeoi.org.ir
Phone: +98 (0)21 82066504


___
The Quantum ESPRESSO community stands by the Ukrainian
people and expresses its concerns about the devastating
effects that the Russian military offensive has on their
country and on the free and peaceful scientific, cultural,
and economic cooperation amongst peoples
___
Quantum ESPRESSO is supported by MaX (www.max-centre.eu)
users mailing listus...@lists.quantum-espresso.org
https://lists.quantum-espresso.org/mailman/listinfo/users___
The Quantum ESPRESSO community stands by the Ukrainian
people and expresses its concerns about the devastating
effects that the Russian military offensive has on their
country and on the free and peaceful scientific, cultural,
and economic cooperation amongst peoples
___
Quantum ESPRESSO is supported by MaX (www.max-centre.eu)
users mailing list users@lists.quantum-espresso.org
https://lists.quantum-espresso.org/mailman/listinfo/users

Re: [QE-users] Query regarding plot for fcc PDC using matdyn.x

2021-08-25 Thread Stefano de Gironcoli
I'm not sure but I guess the path is something like this (all points in 
cartesian coordinates in unit of 2pi/a)


Gamma (000) -> K (0,0.75,0.75) -> X (0,1,1) [this not the usual X but is 
an equivalent one along the sigma line]


X(100)-> Gamma(000) -> L(0.5,0.5,0.5)        [standard delta e lambda lines]

L(0.5,0.5,0.5) ->X(1,0,0) [this is not a symmetry line, it's just my guess]

X(100) -> W(1,0, 0.5) -> L (0.5,0.5,0.5)  [these are quite short 
lines with some symmetry]


HTH

stefano

On 25/08/21 13:51, Pooja Vyas wrote:

Dear users,
I want to plot PDC for fcc-CaO using matdyn.x, for symmetry points 
mentioned in the attached image. Can I know the order of points to be 
given in the i/p file so as to reproduce the same plot? Because, 
though I mention the same points, my plot shows some extra plane and 
looks quite different from the attached plot.


___
Quantum ESPRESSO is supported by MaX (www.max-centre.eu)
users mailing list users@lists.quantum-espresso.org
https://lists.quantum-espresso.org/mailman/listinfo/users
___
Quantum ESPRESSO is supported by MaX (www.max-centre.eu)
users mailing list users@lists.quantum-espresso.org
https://lists.quantum-espresso.org/mailman/listinfo/users

Re: [QE-users] under pressure PbTaSe2 vc-relax calculation

2021-05-26 Thread Stefano de Gironcoli
more than with a different set of k-points, the last scf  is done 
resetting the spherical cutoff according to the last cell (during the 
vc-relaxation the G vectors in the list are deformed but the miller 
indices included in the list are not updated and the sphere actually 
deforms into an ellipsoid ).


As the pressure converges slowly with cutoff if the pressure changes 
significantly this is an indication that the cutoff might have been too 
low or that  the cell has changed a lot.


stefano

On 26/05/21 09:40, mkondrin wrote:

Dear Rencong,

I believe that the last step in vc-relax calculations is made in 
slightly different Brillouin zone (calculated in relaxed variable cell 
parameters), so the pressure calculated in the last step may be 
different from whose obtained in the previous step (where Brillouin 
zone of the original cell was used convergence criteria with desired 
pressure is checked).


It also seems to me that your pressure is rather small. May be it has 
sense to increase the pressure to at least 10 kbar.


Sincerely yours,
M. V. Kondrin


On 26.05.2021 05:48, 连云龙 wrote:

Dear Rencong,

I had similar experiences when I use QE pw.x to relax structures 
under pressure and I want to follow this conversation.


My experience is that, the behaviour and result of relaxation depend 
largely on the pesudopotential.


It is also helpful to investigate the energy and pressure during the 
relaxation.


In order for other experts to help you, could you please provide:
(1) the pseudopotentials used in the calculation
(2) the entire output file of pw.x, or at least the section contains 
the final coordinates? (search "End of BFGS" in the output file)

(3) your name and affiliation (this is the rule of the mailing list)

Best,
Yunlong



--
Yunlong LIAN ORCID : https://orcid.org/-0002-9474-2181 CSNS, 
Dongguan Neutron Science Center Dongguan, Guangdong, P. R. China 连云龙 
中国散裂中子源 中子科学部 A1-629 (内线 303) 中国广东省东莞市

-Original Messages-
From:Amadeus <760158...@qq.com>
Sent Time:2021-05-25 15:22:02 (Tuesday)
To: users 
Cc:
Subject: [QE-users] under pressure PbTaSe2 vc-relax calculation


Dear QE developer


I did the structural relaxation calculation for Pb at pressure of 
3kbar, this is relax.in




   prefix='PbTaSe2',
   calculation='vc-relax', pseudo_dir='../../', outdir='./tmp', 
verbosity='high',

   tprnfor=.true., tstress=.true., forc_conv_thr=1.0d-4, nstep=100,
/

   ibrav= 0, nat= 4, ntyp= 3,
   occupations = 'smearing', smearing = 'gauss', degauss = 1.0d-2,
   ecutwfc = 50, ecutrho = 500,
/

   conv_thr = 1.0d-8
   mixing_beta = 0.7d0
/

/

   cell_dynamics  = "bfgs"
   press  =  2.7e+00
   press_conv_thr =0.1
/
ATOMIC_SPECIES
   Ta 180.9479 Ta.UPF
   Pb 207.2 Pb.UPF
   Se 78.96 Se.UPF
CELL_PARAMETERS (angstrom)
    3.483470  0.00  0.00
   -1.741735  3.0167735133  0.00
    0.00  0.00  9.475530
ATOMIC_POSITIONS (crystal)
   Pb  0.00  0.00  0.00
   Se  0.00  0.00  0.324671
   Ta  0.33  0.67  0.50
   Se  0.00  0.00  0.675329
K_POINTS {automatic}
   15 15 5 0 0 0


In relax.in, press=3.0kbar. But after pw.x is done, relax.out show 
that total stress  (Ry/bohr**3)  (kbar) P=-0.83, and This is 
inconsistent with press=3. this is part of relax.out



[rencong@mu01 phonon]$ grep 'Computing stress (Cartesian axis) and 
pressure' -A 15 relax.out






  Computing stress (Cartesian axis) and pressure


   total   stress  (Ry/bohr**3) (kbar) P=    2.93
    0.2030  -0.   0.    2.99 -0.00    
0.00
   -0.   0.2030  -0.   -0.00 2.99   
-0.00
   -0.   0.   0.1921   -0.00 0.00    
2.83



  kinetic stress (kbar)  26877.23  0.00  0.00
 0.00  26877.23 -0.00
 0.00  0.00  26796.57


  local   stress (kbar)-103340.53  0.03  0.00
 0.03-103340.50  0.00
 0.00  0.00  89394.48


  nonloc. stress (kbar)   7797.15  0.00  0.00
--
  Computing stress (Cartesian axis) and pressure


   total   stress  (Ry/bohr**3) (kbar) P=    2.96
    0.2009  -0.   0.    2.96 -0.00    
0.00
   -0.   0.2009  -0.   -0.00 2.96   
-0.00
    0.  -0.   0.2028    0.00 -0.00    
2.98



  kinetic stress (kbar)  26879.49  0.00  0.00
 0.00  26879.49 -0.00
 0.00 -0.00  26798.86


  local   stress (kbar)-103299.75  0.03  0.00
 0.03-103299.72  0.00
 0.00  0.00  89348.84


  nonloc. stress (kbar)   7797.79 -0.00   

Re: [QE-users] U correction, transferability

2021-05-10 Thread Stefano de Gironcoli

Dear Sergey,

   if you calculate the projected DOS you can see which atoms are 
involved in the states at the edges of the Valence and Conduction Bands. 
moreover you can see which angular momenta are participating into the 
frontier states...


  this should tell you what's going on. whether the states involved 
are not affected by the U part of the Hamiltonian or some more tricky 
cancellation is happening


 HTH

stefano


On 10/05/21 00:35, Sergey Lisenkov wrote:

Dear users and developers,
I have a question about transferability of U value computed with HP code.
I have two materials, lets call them "A" and "B". "B" contains Mn 
atoms, so I calculated U self-consistently using HP code (U 
calculation - relax with U - U calculation - relax with U until U is 
converged within 0.01 eV). Without "U" band gap of material "B" is 2.6 
eV, with U - 4.7 eV. Material "A" does not have transition metal 
atoms, so I used plain DFT for this. Bandgap for "A" is ~4.6 eV.
Now I study "A"-"B" superlattices - layers of "A" material alternating 
with layers of "B" material. What I found that band gap with and 
without U correction I calculated before is almost the same - ~2.6 eV.
Now I'm curious to know why there is no difference in band gap in 
those two calculations. Is it possible that I have to calculate U 
correction for this new material - "A/B" superlattice?

Thanks,
 Sergey
Physics Department,
USF

___
Quantum ESPRESSO is supported by MaX (www.max-centre.eu)
users mailing list users@lists.quantum-espresso.org
https://lists.quantum-espresso.org/mailman/listinfo/users
___
Quantum ESPRESSO is supported by MaX (www.max-centre.eu)
users mailing list users@lists.quantum-espresso.org
https://lists.quantum-espresso.org/mailman/listinfo/users

Re: [QE-users] ?==?utf-8?q? Same run not accelerated when starting from converged rho and wfc

2020-08-16 Thread Stefano de Gironcoli
6.d-9 is still too large.. it should be something like 1d-13 to aim at a 
smaller scf estimate.


are you really starting from the scf charge and wfcs of the same 
conficuration ?


stefano

On 16/08/20 22:05, Antoine Jay wrote:

Dear Stephano,
adding diago_thr_int=1.0D-8 does not change the first conv_thr (exept 
the average#of iterations)
As you said, the first value 1.0D-2 is detected to  be too large and 
is updated to 6.0D-9 so I don't see why changing manually the first 
value would change something if it is already automatically changed...


Antoine Jay
LAAS-CNRS
Toulouse, France

Le Samedi, Août 15, 2020 17:10 CEST, Stefano de Gironcoli 
 a écrit:


Hi Antoine,

  don't know exactly why you get this result but one thing you can try 
is to set diag_thr_init ~ conv_thr/Nelec/10 so the first 
diagonalization is pushed tighter (if the wfcs are already very good 
it should not take too many iterations) and the computed dr2 estimate 
should be more faithful


  now diag_thr_int is 1.d-2 then updated to 6.e-9 which is consistent 
with conv_thr ~6.d-5...


  idk. you can try

  stefano

On 14/08/20 17:09, Antoine Jay wrote:

Dear all,

I'am doing two consecutive scf calculations with exactly the same 
structure and parameters by calling qe6.5 as a library (attached 
output files).

For the second call, I use the options:
startingwfc='file' and input_rho ='rho.in'
where these inputs are the converged wfc1.dat and charge-density.dat 
of the first step.

Here I face two problems:

-I expected that the initial scf accuracy is 10^-11 as obtained at 
the end of the first step, but it is only 10^-4.
How is it possible to explain such a decrease? I generally loose only 
2 orders of magnitude by doing this.


-Even with less scf iterations, the cpu time is greater.
Is it possible that some extra memory is allocated by qe when input 
rho and wfc are asked, and not desallocated?


Note that until now, I have these troubles only when I use paw 
pseudopotentials on big systems.


Regards,

Antoine Jay
LAAS-CNRS
Toulouse France





___
Quantum ESPRESSO is supported by MaX (www.max-centre.eu/quantum-espresso)
users mailing listus...@lists.quantum-espresso.org
https://lists.quantum-espresso.org/mailman/listinfo/users





___
Quantum ESPRESSO is supported by MaX (www.max-centre.eu/quantum-espresso)
users mailing list users@lists.quantum-espresso.org
https://lists.quantum-espresso.org/mailman/listinfo/users
___
Quantum ESPRESSO is supported by MaX (www.max-centre.eu/quantum-espresso)
users mailing list users@lists.quantum-espresso.org
https://lists.quantum-espresso.org/mailman/listinfo/users

Re: [QE-users] Same run not accelerated when starting from converged rho and wfc

2020-08-15 Thread Stefano de Gironcoli

Hi Antoine,

  don't know exactly why you get this result but one thing you can try 
is to set diag_thr_init ~ conv_thr/Nelec/10 so the first diagonalization 
is pushed tighter (if the wfcs are already very good it should not take 
too many iterations) and the computed dr2 estimate should be more faithful


  now diag_thr_int is 1.d-2 then updated to 6.e-9 which is consistent 
with conv_thr ~6.d-5...


  idk. you can try

  stefano


On 14/08/20 17:09, Antoine Jay wrote:

Dear all,

I'am doing two consecutive scf calculations with exactly the same 
structure and parameters by calling qe6.5 as a library (attached 
output files).

For the second call, I use the options:
startingwfc='file' and input_rho ='rho.in'
where these inputs are the converged wfc1.dat and charge-density.dat 
of the first step.

Here I face two problems:

-I expected that the initial scf accuracy is 10^-11 as obtained at the 
end of the first step, but it is only 10^-4.
How is it possible to explain such a decrease? I generally loose only 
2 orders of magnitude by doing this.


-Even with less scf iterations, the cpu time is greater.
Is it possible that some extra memory is allocated by qe when input 
rho and wfc are asked, and not desallocated?


Note that until now, I have these troubles only when I use paw 
pseudopotentials on big systems.


Regards,

Antoine Jay
LAAS-CNRS
Toulouse France






___
Quantum ESPRESSO is supported by MaX (www.max-centre.eu/quantum-espresso)
users mailing list users@lists.quantum-espresso.org
https://lists.quantum-espresso.org/mailman/listinfo/users
___
Quantum ESPRESSO is supported by MaX (www.max-centre.eu/quantum-espresso)
users mailing list users@lists.quantum-espresso.org
https://lists.quantum-espresso.org/mailman/listinfo/users

Re: [QE-users] negative fermi level

2020-05-26 Thread Stefano de Gironcoli
the zero of eigenvalue energy in a solid is arbitrarily set at the 
electrostatic average potential.


there is nothing special in the zero value for the eigenvalues in a 
solid, in particular it is not the value of the potential at infinite 
distance (which cannot be defined in a solid extending to infinite).


HTH

stefano

On 26/05/20 12:30, Neelam Swarnkar wrote:

Dear Expert , user and all


I am calculating , the scf ,nscf calculation of B-Zn4Sb3 material . by 
PBE-PAW xc . and nscf.out file found the fermi energy = -1.6482 eV
 So, i want to know that , negative fermi energy is possible in the qe 
calculation or not .


thanks in advance


___
Quantum ESPRESSO is supported by MaX (www.max-centre.eu/quantum-espresso)
users mailing list users@lists.quantum-espresso.org
https://lists.quantum-espresso.org/mailman/listinfo/users
___
Quantum ESPRESSO is supported by MaX (www.max-centre.eu/quantum-espresso)
users mailing list users@lists.quantum-espresso.org
https://lists.quantum-espresso.org/mailman/listinfo/users

Re: [QE-users] How to obtain spin-state in QE?

2020-05-23 Thread Stefano de Gironcoli
you can choose tot_magnetization = total majority spin charge - minority 
spin charge


in your input.

this way you can determine the energy of the configuration with maximum 
multiplicity (like the triplet state in a dissociating hydrogen dimer). 
by choosing other values you can access linear combinations of the 
different states (like the Sz=0  state for a dissociating hydrogen simer 
is a 50/50 superposition of the singlet and triplet (Sz=0) states).


at the equilibrium geometry tho the Sz=0 state is non degenerate and it 
is therefore the desired singlet... it really depends on the degeneracy 
of the different (broken symmetry) solutions that you can stabilize.


useful read U. von Barth, Local-density theory of multiplet structure, 
PRA 20 1693 (1979),  DOI:10.1103/PhysRevA.20.1693


HTH

stefano


On 23/05/20 12:38, sahar mohammadi wrote:

Dear QE users

I have performed spin-polarized DFT calculation for Fe2O3 cluster. now 
I want to obtain the spin-state of the relaxed (optimized ground 
state) cluster. for example it is doublet or quartet. How can I know 
what is the spin state?


Kind regards

___
Quantum ESPRESSO is supported by MaX (www.max-centre.eu/quantum-espresso)
users mailing list users@lists.quantum-espresso.org
https://lists.quantum-espresso.org/mailman/listinfo/users
___
Quantum ESPRESSO is supported by MaX (www.max-centre.eu/quantum-espresso)
users mailing list users@lists.quantum-espresso.org
https://lists.quantum-espresso.org/mailman/listinfo/users

Re: [QE-users] nscf calculation of 2x2x2 Si supercell

2020-05-14 Thread Stefano de Gironcoli

do you really need conv_thr=1e-20 ?

stefano

On 14/05/20 09:18, Fariba Islam wrote:

Dear all,
My  conv_thr=1e-20 and ecutwfc=200 ecutrho=800
My nscf calculation is crashing with the error

task #        10
     from c_bands : error #         1
     too many bands are not converged

How can I solve? From suggestions found online, I have reduced the 
threshold and increased ecutwfc and ecutrho.

Regards
Fariba Islam
Bangladesh University of Engineering and Technology

___
Quantum ESPRESSO is supported by MaX (www.max-centre.eu/quantum-espresso)
users mailing list users@lists.quantum-espresso.org
https://lists.quantum-espresso.org/mailman/listinfo/users
___
Quantum ESPRESSO is supported by MaX (www.max-centre.eu/quantum-espresso)
users mailing list users@lists.quantum-espresso.org
https://lists.quantum-espresso.org/mailman/listinfo/users

Re: [QE-users] How ev.x calculate enthalpy?

2020-05-08 Thread Stefano de Gironcoli

dear Inna,


  in the output file energies and enthalpies are written in Ry, volumes 
in A^3 and pressures in GPa


  in order to apply directly the enthalpy formula everything should be 
given with consistent units.


  if you check the ev.f90 code (in PW/tools) there are unit conversion 
factors when using the pressure



line 304:     epv(i) = etot(i) + p(i)*v0(i) / ry_kbar


   and when printing the volume


lines 350-352:

  WRITE(iun,'(f8.2,2x,f12.5, 2x,f12.5, f12.5, 3x, f8.2, 
3x,f12.5)') &

  ( v0(i)*bohr_radius_angs**3, etot(i), efit(i), &
   etot(i)-efit(i), p(i)/gpa_kbar, epv(i), i=1,npt )


   best


stefano


On 08/05/20 09:01, Inna Nangoi wrote:

Hi,
I can't figure out how ev.x calculate enthalpy. I checked the ev.f90 
and found out that

Enth(i)=Efit(i)+Pfit(i)*V0(i) is the enthalpy (Ry)
But when I try to calculate by hand I don't reach the same value as 
reported in ev.x

output file (see below) . For example, for V = 120.51
Enth(i)=Efit(i)+Pfit(i)*V0(i)
Enth(i)= -181.23265 + 2.93*120.51 = 171.86161

What am I doing wrong?

# equation of state: murnaghan.    chisq =   0.4329D-07
# V0 =  996.51 a.u.^3,  k0 =   92 kbar,  dk0 =  4.10  d2k0 = 0.000  
emin = -181.24780

# V0 =  147.67  Ang^3,  k0 =   9.2 GPa

##
# Vol.    E_calc    E_fit   E_diff Pressure  Enthalpy
# Ang^3 Ry   Ry    Ry GPa   Ry
##
  124.13    -181.23698    -181.23704 0.5 2.34 -181.10397
  120.51    -181.23265    -181.23269 0.3 2.93 -181.07085

Best
Inna Nangoi
Stockholm University /Universidade Federal de Juiz de Fora


___
Quantum ESPRESSO is supported by MaX (www.max-centre.eu/quantum-espresso)
users mailing list users@lists.quantum-espresso.org
https://lists.quantum-espresso.org/mailman/listinfo/users
___
Quantum ESPRESSO is supported by MaX (www.max-centre.eu/quantum-espresso)
users mailing list users@lists.quantum-espresso.org
https://lists.quantum-espresso.org/mailman/listinfo/users

Re: [QE-users] POSCAR to QE input

2020-05-06 Thread Stefano de Gironcoli

dear Poonam,

it would be helpful if you describe the meaning of each term in the 
input for those not familiar with POSCAR format..


as for QE this is described, for instance, in 
https://www.quantum-espresso.org/Doc/INPUT_PW.html


best

stefano

On 06/05/20 07:13, Poonam Kaushik wrote:

Dear all,
I want to convert a POSCAR file into the QE input file. I would be 
very grateful if somebody helps or give suggestions.

Fe1 S1
   1.
     5.9487492159093023   -0. -0.
    -2.9743746079546511    5.1517679417117126  0.
    -0.    0. 11.7211034581915818
   Fe   S
    12    12
Direct
  0.3954866316692124  0.0721204949184797  0.1217724210678790
  0.9278795160815250  0.3233661487507409  0.1217724210678790
  0.6766338512492591  0.6045133683307876  0.1217724210678790
  0.3954866316692124  0.0721204949184797  0.3782275859321179
  0.9278795160815250  0.3233661487507409  0.3782275859321179
  0.6766338512492591  0.6045133683307876  0.3782275859321179
  0.0721204949184797  0.3954866316692124  0.8782275859321180
  0.3233661487507409  0.9278795160815250  0.8782275859321180
  0.6045133683307876  0.6766338512492591  0.8782275859321180
  0.0721204949184797  0.3954866316692124  0.6217724140678820
  0.3233661487507409  0.9278795160815250  0.6217724140678820
  0.6045133683307876  0.6766338512492591  0.6217724140678820
 -0. -0.  0.
 -0. -0.  0.5000
  0.33342996  0.66687029  0.0251416953082458
  0.33342996  0.66687029  0.4748582936917498
  0.66687029  0.33342996  0.9748582936917498
  0.66687029  0.33342996  0.5251417063082502
  0.6643064194527374  0.477489745914  0.2500
  0.522750254107  0.6643586704781460  0.2500
  0.3356412995218515  0.3356935805472627  0.2500
  0.477489745914  0.6643064194527374  0.7500
  0.6643586704781460  0.522750254107  0.7500
  0.3356935805472627  0.3356412995218515  0.7500
Thanks,
Poonam S



-
Poonam Sharma
Research Scholar
Department of Physics
Indian Institute of Technology Bombay
Mumbai - 400076
India.


___
Quantum ESPRESSO is supported by MaX (www.max-centre.eu/quantum-espresso)
users mailing list users@lists.quantum-espresso.org
https://lists.quantum-espresso.org/mailman/listinfo/users
___
Quantum ESPRESSO is supported by MaX (www.max-centre.eu/quantum-espresso)
users mailing list users@lists.quantum-espresso.org
https://lists.quantum-espresso.org/mailman/listinfo/users

Re: [QE-users] Dipole correction: electrostatic potential partially corrected

2020-05-01 Thread Stefano de Gironcoli

I think the code does what you ask for

it set the dipole double layer stating al 95% (emaxpos = 0.95) of you 
cell extending for 5% up to 100% (eopreg = 0.05 )


the slope from 95% to 100$% is just the internal potential drop in the 
double layer...


stefano

On 01/05/20 16:57, Pacome NGUIMEYA wrote:

Dear QE users,

I generated a (1x1) surface slab ( with 20-angstroms vacuum) of the 
(001) surface using a 5-layer slab model.
I included the dipole correction to cancel the artificial field as my 
slab is asymmetric.


As you can see in the picture here attached, the correction did not 
work on both sides. In fact, I got a perfectly
flat potential in the vacuum region to the left of the slab but not to 
the right.


I tried different values for emaxpos and eopreg following the example 
on this blog 
https://christoph-wolf.at/2018/05/02/dipole-correction-in-quantum-espresso/, 
but only those in the input file below could give something close to 
the expected result.


PS: My slab is centered at the center of the cell.

Can you please advise how I can get this fixed?

Thanks in advance for your help


                 calculation = 'scf',
                      prefix = 'TaAs',
                restart_mode = 'from_scratch',
                  pseudo_dir = './',
                      outdir = './',
               etot_conv_thr = 1.0e-5,
                  wf_collect = .true.,
                   verbosity = 'high',
                     tefield = .true.,
                    dipfield = .true.
/

                       ibrav = 0,
                         nat = 40,
                        ntyp = 2,
                     ecutwfc = 60,
                     ecutrho = 480,
                 occupations = 'smearing',
                    smearing = 'm-p',
                     degauss = 0.03,
                        eamp = 0,
                        edir = 3,
                     emaxpos = 0.95,
                      eopreg = 0.05
/

             diagonalization = 'david',
                 mixing_mode = 'local-TF',
                 mixing_beta = 0.7,
                    conv_thr = 1.0e-6,
            electron_maxstep = 300
/
ATOMIC_SPECIES
 Ta  180.948000    Ta.pbe-spn-kjpaw_psl.1.0.0.UPF
 As   74.921600    As.pbe-n-kjpaw_psl.1.0.0.UPF
CELL_PARAMETERS {angstrom}
 3.46982824000       0.000 0.000
 0.000       3.46982824000 0.000
 0.000       0.000  76.718787642331264
ATOMIC_POSITIONS {angstrom}
  As  0.  1.734914119991  63.301754635841
  As  0.  1.734914119991  51.564173075888
  As  0.  1.734914119991  39.826591515935
  As  0.  1.734914119991  28.089009956053
  As  0.  1.734914119991  16.351428395887
  As  0.  0.  66.236150025900
  As  0.  0.  54.498568465947
  As  0.  0.  42.760986905994
  As  0.  0.  31.023405345899
  As  0.  0.  19.285823785946
  As  1.734914119991  0.  57.432963855935
  As  1.734914119991  0.  45.695382295911
  As  1.734914119991  0.  33.957800735958
  As  1.734914119991  0.  22.220219175934
  As  1.734914119991  0.  10.482637615981
  As  1.734914119991  1.734914119991  60.367359245924
  As  1.734914119991  1.734914119991  48.629777685971
  As  1.734914119991  1.734914119991  36.892196125875
  As  1.734914119991  1.734914119991  25.154614565922
  As  1.734914119991  1.734914119991  13.417033005898
  Ta  1.734914119991  0.  62.3364794036686973
  Ta  1.734914119991  0.  50.5988978436687091
  Ta  1.734914119991  0.  38.8613162836686996
  Ta  1.734914119991  0.  27.1237347236686972
  Ta  1.734914119991  0.  15.3861531636687161
  Ta  1.734914119991  1.734914119991  65.2708747936686962
  Ta  1.734914119991  1.734914119991  53.5332932336687009
  Ta  1.734914119991  1.734914119991  41.7957116736687126
  Ta  1.734914119991  1.734914119991  30.0581301136687102
  Ta  1.734914119991  1.734914119991  18.3205485536687149
  Ta  0.  1.734914119991  56.4676886236687139
  Ta  0.  1.734914119991  44.7301070636687257
  Ta  0.  1.734914119991  32.9925255036687162
  Ta  0.  1.734914119991  21.2549439436687280
  Ta  0.  1.734914119991 9.5173623836687113
  Ta  0.  0.  59.4020840136687056
  Ta  0.  0.  47.6645024536687103
  Ta  0.  0.  

Re: [QE-users] Convergence of hybrids: set dexx >> conv_thr ?

2020-04-24 Thread Stefano de Gironcoli
in the scf cycle the estimated scf accuracy is an estimate of the 
overestimation of the current energy wrt the scf solution.


in the hybrid case, due to the double loop procedure, it is made of two 
terms:
1) how much the inner scf cycle (when V_x is computed with  frozen HF 
orbitals)  is accurate (this is maesured by dr2 in mix_rho and  targets tr2)
2) how much the scf energy is overestimated because the in/out HF 
orbitals do not agree in the outer loop (this is measured by dexx and 
targets tr2_final)


as you say    tr2_final=tr2

the (ir)rational of this is that what one is asking  is that the overall 
error in energy is  not larger than   tr2 ( actually 2*tr2)
and it would be pointless to ask a tight tr2 in one term when one is 
happy to accept a larger error elsewhere...


but  different ratios of  tr2/tr2_final were not explored so maybe some 
other combinations might be more effective to reach the final result


what one should keep in mind is that the estimated error in the final 
energy (the overall scf accuracy) at the end of the

calculation is     dr2+dexx targeting    tr2+tr2_final

stefano

On 24/04/20 03:18, Christoph Wolf wrote:

Dear all,

I was wondering if there is a particular reason that a hybrid 
calculation converges at the same threshold as the SCF as


tr2_final = tr2

Since my dexx converges painfully slowly after the first few 
iterations I was wondering if it could be "safe" to terminate the 
calculation earlier?


     est. exchange err (dexx)  =       0.08363460 Ry
     est. exchange err (dexx)  =       0.00463121 Ry
     est. exchange err (dexx)  =       0.00159110 Ry
     est. exchange err (dexx)  =       0.00100574 Ry
     est. exchange err (dexx)  =       0.00063909 Ry
     est. exchange err (dexx)  =       0.00037948 Ry
     est. exchange err (dexx)  =       0.00021565 Ry
     est. exchange err (dexx)  =       0.00012503 Ry
     est. exchange err (dexx)  =       0.7862 Ry
     est. exchange err (dexx)  =       0.5529 Ry

...
(30 ACE cycles later)
     est. exchange err (dexx)  =       0.0155 Ry
     est. exchange err (dexx)  =       0.0154 Ry
     est. exchange err (dexx)  =       0.0153 Ry
     est. exchange err (dexx)  =       0.0152 Ry
     est. exchange err (dexx)  =       0.0152 Ry

when looking at eigenvalues at gamma (this is from a molecule) they 
are basically static at this point.


I am mainly wondering because there is no variable that controls 
tr2_final independently from tr2 so I guess there must be a good 
reason for that...


thanks in advance!

Best,
Chris
--
Postdoctoral Researcher
Center for Quantum Nanoscience, Institute for Basic Science
Ewha Womans University, Seoul, South Korea

___
Quantum ESPRESSO is supported by MaX (www.max-centre.eu/quantum-espresso)
users mailing list users@lists.quantum-espresso.org
https://lists.quantum-espresso.org/mailman/listinfo/users
___
Quantum ESPRESSO is supported by MaX (www.max-centre.eu/quantum-espresso)
users mailing list users@lists.quantum-espresso.org
https://lists.quantum-espresso.org/mailman/listinfo/users

Re: [QE-users] Born charges of PbS

2020-04-13 Thread Stefano de Gironcoli
I didnt follow completely your argument about the Bader charge but what 
one should keep in mind is that the effective charge is the change in 
polarization due to displacement not the change in charge...


think of a core electron shell (as the d orbitals of Pb can 
approximately be considered) ... as you move the atom they follow 
rigidly (that would make a contribution of 10 not far from your estimate).


from the PHONON result it looks like Pb gave away 4 all its valence 
electrons to S. or rather they are so weakly bound to Pb that they don't 
follow it, even if they can still belong its Bader volume.


it looks a bit extreme but this appears to be the result. You could 
compute the atomic projected density of states and see if this seems the 
case.


stefano

On 13/04/20 17:11, Vahid Askarpour wrote:

Dear QE Community,

I have calculated the Born charges using the PHONON code for PbS. The only 
non-zero elements are the diagonal ones and are 4.122 and -4.168, respectively.

In the zstar_eu.f90, Born charges consist of two terms as seen below: a part 
due to polarization calculation (dynamic) and the other is zv (static) which is 
the z_valence according to read_upf_v2.f90.

  do ipol = 1, 3
  do na = 1, nat
 zstareu (ipol, ipol, na) = zstareu (ipol, ipol, na) + zv (ityp ( na) )
  enddo
   enddo

The zv values for Pb and S are 14 and 6 given in the PSP. If we subtract zv 
from the Born charges, we get the term due to polarization: -9.878 and -10.168. 
These values seem too large because of the argument below.

To estimate the polarization term, I reduce the alat by 1% and relax the atoms. 
This shifts the atoms from the unstrained position. I calculate the Bader 
charges for the unstrained and the strained cases. The change in the Bader 
charge is related to the atomic displacement. I have also tried keeping alat 
fixed and moving the atoms by 1%.

For unstrained PbS, the Bader charges are 12.998 and 7.001.
For the strained PbS, they are 13.004 and 6.995.

So a ~1% change in atomic positions results in a +/-0.006 change in Bader 
charge. From this calculation, I expect the contribution from polarization to 
be u(dZ/du), where u is interatomic distance, which amount to +0.6 for Pb and 
-0.6 for S.

The contribution from polarization I get (0.6 and -0.6) are quite different 
from the those of the PHONON code (-9.878 and -10.168). I am assuming that the 
code is correct and my logic is flawed. I would appreciate any thoughts you may 
have on this discrepancy.

Thank you,
Vahid


Vahid Askarpour
Department of physics and atmospheric science
Dalhousie University
Halifax, NS
Canada
___
Quantum ESPRESSO is supported by MaX (www.max-centre.eu/quantum-espresso)
users mailing list users@lists.quantum-espresso.org
https://lists.quantum-espresso.org/mailman/listinfo/users

___
Quantum ESPRESSO is supported by MaX (www.max-centre.eu/quantum-espresso)
users mailing list users@lists.quantum-espresso.org
https://lists.quantum-espresso.org/mailman/listinfo/users


Re: [QE-users] Questions related to vibrational analysis (for both molecule and adsorbate)--update

2020-03-17 Thread Stefano de Gironcoli

as an addition to my previous reply..

If you decide to diagonalize the restricted nat_todo matrix.. DO NOT 
impose ASR on it.


It does not apply to the truncated matrix..

stefano

On 17/03/20 16:08, Stefano de Gironcoli wrote:



On 17/03/20 15:46, Paolo Giannozzi wrote:
On Mon, Mar 16, 2020 at 1:53 AM Shen, Ziheng <mailto:zshe...@gatech.edu>> wrote:



1) May I ask is there any literature that can prove it’s feasible
to neglect the small frequencies?


no idea, but you can easily verify whether the neglect of almost-zero 
frequencies has any effect: if you impose the ASR on dynamical 
matrices, only frequencies of translational modes (and of rotational 
ones for a molecule) should change, while all other frequencies 
should stay almost the same.


2) I saw you also replied to others that “nat_todo” does not make
any sense, but there were still people claimed that they got some
useful results. I read from some books (like the one written by
D.Sholl) saying that it’s possible to calculate only frequencies
of adsorbates which could save computational resources. Some
other ab initial calculation software (like VASP) also have
similar functions to calculate adsorbates only. Why is “nat_todo”
not working here?


Presumably it doesn't work because it is not sufficient to just 
ignore all surface atoms and compute the dynamical matrix restricted 
to adsorbate atoms only. I guess there are tricks to approximated 
phonons for an adsorbate on a surface by computing a  "reduced" 
dynamical matrix for the adsorbate only, in which the information on 
the adsorbate-surface interactions is "refolded",  but I don't know 
any. This same question has been asked over and over again in the 
list and nobody (that I remember) ever gave an answer.


Paolo

I think that if you have light atoms adsorbed on some heavier material 
and are looking for the adsorbate related frequencies then you can 
diagonalize the reduced matrix and the frequencies that you obtain 
should be variational upper bounds to the real ones


just pretend you computed the whole matrix but restrict the atomic 
motion to a subset of atoms... so to the extent the modes are 
localized on the adsorbate with little involvement of the substrate 
you should be ok...


but the low lying frequencies (the ones resonant with the substrate 
modes) will be completely wrong.



one could start with nat_todo equal to just the adsorbate and then the 
first shell, then the second one ... and see  how things converge ...


another option (better, but would need some ad hoc coding) would be to 
build the dynmat of the nat_todo atoms with interatomic force 
constants for the rest of the cell borrowed from some model (the bulk, 
or the clean surface) ... and again monitor the convergence as you 
include more and more shells  of atoms in the nat_todo.



stefano




Best regards
Ziheng Shen
PhD student @ Georgia Institute of Technology

On Mar 15, 2020, at 7:00 AM, 
users-requ...@lists.quantum-espresso.org 
<mailto:users-requ...@lists.quantum-espresso.org> wrote:


On Fri, Mar 13, 2020 at 4:22 AM Shen, Ziheng <mailto:zshe...@gatech.edu>> wrote:


1) When doing frequency analysis for molecules, I expected to get 
zero or
extremely small value for the first six frequencies (i.e. 
translational &
rotational modes). According to suggestions from those previously 
posted
problems, I tried to apply more restrict convergence thresholds and 
ASR. It
seems that ASR help a lot to reduce the number. But I still got 
frequencies
at ~50 level. Is it possible to completely remove those small 
values? Or

are those values small enough to be neglected?



They are small enough to be neglected. They can be removed by 
applying the
ASR to the computed dynamical matrix. See the various kinds of ASR 
in codes

"dynmat" and "matdyn", in particular the "zero-dim" one. Note that  more
sophisticated ASR than "simple" can be surprising slow.

2) My ultimate goal is to perform frequency analysis for adsorbate 
so that
I can both determine transition state structures and apply ZPE 
corrections.
I tried to use ?nat_todo? to fix the surface atoms and only did 
calculation
for adsorbate (CH in my case). I got crazy result (~1 cm-1) 
when using
large tr2_ph, and got improved results when I decrease the 
threshold. But I
still got fairly large translational & rotational frequencies like 
below




I don't think you will obtain anything sensible by fixing the 
surface atoms

and making the calculation for the adsorbate atoms  only

Paolo




 freq (    1) = -25.618746 [THz] =    -854.549399 [cm-1]
freq (    2) =  -7.333895 [THz] =    -244.632409 [cm-1]
freq (    3) =  -6.696884 [THz] =    -223.383991 [cm-1]
freq (    4) =  -6.248674 [THz] =    -208.433322 [cm-1]
freq (    5) =  -4.947831 [THz] =    -165.041892 [cm-1]
freq (

Re: [QE-users] Questions related to vibrational analysis (for both molecule and adsorbate)--update

2020-03-17 Thread Stefano de Gironcoli


On 17/03/20 15:46, Paolo Giannozzi wrote:
On Mon, Mar 16, 2020 at 1:53 AM Shen, Ziheng > wrote:



1) May I ask is there any literature that can prove it’s feasible
to neglect the small frequencies?


no idea, but you can easily verify whether the neglect of almost-zero 
frequencies has any effect: if you impose the ASR on dynamical 
matrices, only frequencies of translational modes (and of rotational 
ones for a molecule) should change, while all other frequencies should 
stay almost the same.


2) I saw you also replied to others that “nat_todo” does not make
any sense, but there were still people claimed that they got some
useful results. I read from some books (like the one written by
D.Sholl) saying that it’s possible to calculate only frequencies
of adsorbates which could save computational resources. Some other
ab initial calculation software (like VASP) also have similar
functions to calculate adsorbates only. Why is “nat_todo” not
working here?


Presumably it doesn't work because it is not sufficient to just ignore 
all surface atoms and compute the dynamical matrix restricted to 
adsorbate atoms only. I guess there are tricks to approximated phonons 
for an adsorbate on a surface by computing a  "reduced" dynamical 
matrix for the adsorbate only, in which the information on the 
adsorbate-surface interactions is "refolded",  but I don't know any. 
This same question has been asked over and over again in the list and 
nobody (that I remember) ever gave an answer.


Paolo

I think that if you have light atoms adsorbed on some heavier material 
and are looking for the adsorbate related frequencies then you can 
diagonalize the reduced matrix and the frequencies that you obtain 
should be variational upper bounds to the real ones


just pretend you computed the whole matrix but restrict the atomic 
motion to a subset of atoms... so to the extent the modes are localized 
on the adsorbate with little involvement of the substrate you should be 
ok...


but the low lying frequencies (the ones resonant with the substrate 
modes) will be completely wrong.



one could start with nat_todo equal to just the adsorbate and then the 
first shell, then the second one ... and see  how things converge ...


another option (better, but would need some ad hoc coding) would be to 
build the dynmat of the nat_todo atoms with interatomic force constants 
for the rest of the cell borrowed from some model (the bulk, or the 
clean surface) ... and again monitor the convergence as you include more 
and more shells  of atoms in the nat_todo.



stefano




Best regards
Ziheng Shen
PhD student @ Georgia Institute of Technology

On Mar 15, 2020, at 7:00 AM, users-requ...@lists.quantum-espresso.org 
 wrote:


On Fri, Mar 13, 2020 at 4:22 AM Shen, Ziheng > wrote:


1) When doing frequency analysis for molecules, I expected to get zero or
extremely small value for the first six frequencies (i.e. 
translational &

rotational modes). According to suggestions from those previously posted
problems, I tried to apply more restrict convergence thresholds and 
ASR. It
seems that ASR help a lot to reduce the number. But I still got 
frequencies

at ~50 level. Is it possible to completely remove those small values? Or
are those values small enough to be neglected?



They are small enough to be neglected. They can be removed by 
applying the
ASR to the computed dynamical matrix. See the various kinds of ASR in 
codes

"dynmat" and "matdyn", in particular the "zero-dim" one. Note that  more
sophisticated ASR than "simple" can be surprising slow.

2) My ultimate goal is to perform frequency analysis for adsorbate so 
that
I can both determine transition state structures and apply ZPE 
corrections.
I tried to use ?nat_todo? to fix the surface atoms and only did 
calculation
for adsorbate (CH in my case). I got crazy result (~1 cm-1) when 
using
large tr2_ph, and got improved results when I decrease the 
threshold. But I

still got fairly large translational & rotational frequencies like below



I don't think you will obtain anything sensible by fixing the surface 
atoms

and making the calculation for the adsorbate atoms  only

Paolo




 freq (    1) = -25.618746 [THz] =    -854.549399 [cm-1]
freq (    2) =  -7.333895 [THz] =    -244.632409 [cm-1]
freq (    3) =  -6.696884 [THz] =    -223.383991 [cm-1]
freq (    4) =  -6.248674 [THz] =    -208.433322 [cm-1]
freq (    5) =  -4.947831 [THz] =    -165.041892 [cm-1]
freq (    6) =  -2.014699 [THz] = -67.203109 [cm-1]
freq (   37) =   0.571458 [THz] =  19.061786 [cm-1]
freq (   38) =   5.754719 [THz] = 191.956759 [cm-1]
freq (   39) =  16.488930 [THz] = 550.011494 [cm-1]
freq (   40) =  16.563150 [THz] = 552.487199 [cm-1]
freq (   41) =  

Re: [QE-users] Problem of convergence

2020-03-11 Thread Stefano de Gironcoli
idk about the molecule but I think the bulk input has a very large valuo 
of celldm(3) which should be c/a not c.


also the distance between O and H looks large...

and by the way... Urea should be CO(NH2)2.  that should be 8 atoms

your cell only has 5  !?

stefano

On 11/03/20 14:52, De Gary, Alban wrote:


Dear users and devlopers,




I am trying to determine cohesive Energy of Urea.

I created inputs for a bulk (according to cif file) and isolated 
molecule.


Unfortunately, it is hard to make it converge… I tried to optimize as 
much as possible every factor adapted to my problem, but I’m really stuck


Can you have a look on my inputs and tell me what’s wrong please ?

Best regards,

Thank you in advance for your answers

Alban

STATEMENT OF CONFIDENTIALITY.

This email and any attachments may contain confidential, proprietary, 
privileged and/or private information.


If received in error, please notify us immediately by reply email and 
then delete this email and any attachments from your system. Thank you!


Veuillez trouver notre information relative à la protection des 
données ici 



___
Quantum ESPRESSO is supported by MaX (www.max-centre.eu/quantum-espresso)
users mailing list users@lists.quantum-espresso.org
https://lists.quantum-espresso.org/mailman/listinfo/users
___
Quantum ESPRESSO is supported by MaX (www.max-centre.eu/quantum-espresso)
users mailing list users@lists.quantum-espresso.org
https://lists.quantum-espresso.org/mailman/listinfo/users

Re: [QE-users] Counting Basis Functions

2020-02-19 Thread Stefano de Gironcoli

On 19/02/20 17:58, Ben Comer wrote:

sticks:   dense  smooth PW G-vecs:    dense smooth  PW
 Sum 499 199    55 51837 
13151      1677 


there are 51837 G vectors such that ekin < ecutrho   ... this is used to 
expand the density


there are 131151 G-vectorse such that ekin < ecutwfc*4 ... this is used 
to apply the potential on a wfc


there are  1677 G-vectrose such that ekin < ecutwfc ...  this used to 
expand the wfcs


-

sticks refers to the fact that G-vectors are organized in z-oriented 
sticks. this affects the fft distribution


 499 non empty sticks in the FFT up to ecutrho, 199 non empty sticks in 
the FFT up to ecutwfc*4, 55 non-empty sticks in the FFT up to Ecutwfc



___
Quantum ESPRESSO is supported by MaX (www.max-centre.eu/quantum-espresso)
users mailing list users@lists.quantum-espresso.org
https://lists.quantum-espresso.org/mailman/listinfo/users

Re: [QE-users] Does order of atoms matter in Quantum Espresso?

2019-12-30 Thread Stefano de Gironcoli
the order of the entries in the pseudopotential list or in the atomic 
position list should not affect the result.


however you are not changing only the order.

1) you are also swapping the two atoms... this too should not change 
anything but is not the same as changing the order only :)


2) the first calculation is metallic (with a quite large gaussian 
smearing, 0.05 Ry is ~ 0.7 eV) the second uses the default insulating 
setting. THIS may have a big impact, for two reasons: i) the fundamental 
gap in AlSb, if I remember correctly, is not very large .. comparable to 
the smearing width used; ii) being AlSb a polar material the vibrational 
properties have a non analytic component ruled by the macroscopic 
dielectric properties (Z*, epsilon_infty) that are disabled in the 
phonon calculation of metals (where epsilon_infty diverges).


Additionally, in the phonon calculation step you should make sure you do 
compute Z* and epsilon and include them in the subsequent phonon 
dispersion calculation step.


HTH

stefano

On 30/12/19 23:07, Sheikh Ziauddin Ahmed wrote:
I am trying to calculate the phonon dispersion of AlSb. I am getting 
two different results depending on how I am introducing the atoms in 
the SCF calculation. Will the order of the atom change the symmetry of 
the zinc blende AlSb?


In the first case the input deck is:

    ibrav=2, nat=2, ntyp=2,
    ecutwfc = 50, ecutrho = 500,
    occupations='smearing',
    smearing='gauss',
    degauss=0.05,
    celldm(1)=11.5944146,
 /
 
    conv_thr    = 1.0e-12
    mixing_beta = 0.7
    mixing_mode = 'local-TF'
 /
ATOMIC_SPECIES
 Al 26.98 al_lda_v1.uspp.F.UPF
 Sb 121.76 sb_lda_v1.4.uspp.F.UPF
ATOMIC_POSITIONS (alat)
 Al 0.0   0.0   0.0
 Sb 0.25  0.25  0.25
K_POINTS automatic
6 6 6 0 0 0

In the second case the input deck is:
 
    ibrav=2, nat=2, ntyp=2,
    ecutwfc = 50, ecutrho = 500,
    !occupations='smearing',
    !smearing='gauss',
    !degauss=0.05,
    celldm(1)=11.5944146,
 /
 
    conv_thr    = 1.0e-12
    mixing_beta = 0.7
    mixing_mode = 'local-TF'
 /
ATOMIC_SPECIES
 Sb 121.76 sb_lda_v1.4.uspp.F.UPF
 Al 26.98 al_lda_v1.uspp.F.UPF
ATOMIC_POSITIONS (alat)
 Sb 0.0   0.0   0.0
 Al 0.25  0.25  0.25
K_POINTS automatic
6 6 6 0 0 0

Thanks in advance!
--
*Sheikh Ziauddin Ahmed *
PhD Candidate
Charles L. Brown Department of Electrical and Computer Engineering
University of Virginia
Email:sza9wz_@virginia.edu_ 
LinkedIn: https://www.linkedin.com/in/sheikh-ziauddin-ahmed

___
Quantum ESPRESSO is supported by MaX (www.max-centre.eu/quantum-espresso)
users mailing list users@lists.quantum-espresso.org
https://lists.quantum-espresso.org/mailman/listinfo/users
___
Quantum ESPRESSO is supported by MaX (www.max-centre.eu/quantum-espresso)
users mailing list users@lists.quantum-espresso.org
https://lists.quantum-espresso.org/mailman/listinfo/users

Re: [QE-users] a.u. force conversion

2019-10-19 Thread Stefano de Gironcoli
if the calculation is done with the pwscf code (pw.x) units of energy 
are Ry and unit of forces are Ry/Bohr


default force threshold is 1.d-3 ~ 26 meV/A

stefano

On 19/10/19 02:28, Robert Stanton wrote:

Hello,

    I am seeing a lot of people having numbers of around .2eV/A, or 2 
to some power of 10 for force_conv_thr when reading papers which have 
used Quantum Espresso. I am not seeing how they are getting to this. I 
am taking force in a.u. to be Hartree/Bohr and converting this to eV/A 
as a multiple of ~51. I just want to double check that I am not 
misinterpreting that force convergence is being given in Hartree/Bohr 
or something. It seems strange that many people would be putting in 4* 
something in as a threshold, so I am guessing I'm missing something. 
Any information would be greatly appreciated.


Thanks in advance,
Robert Stanton
Clarkson University

___
Quantum ESPRESSO is supported by MaX (www.max-centre.eu/quantum-espresso)
users mailing list users@lists.quantum-espresso.org
https://lists.quantum-espresso.org/mailman/listinfo/users
___
Quantum ESPRESSO is supported by MaX (www.max-centre.eu/quantum-espresso)
users mailing list users@lists.quantum-espresso.org
https://lists.quantum-espresso.org/mailman/listinfo/users

Re: [QE-users] What Is Estimated SCF Accuracy

2019-10-12 Thread Stefano de Gironcoli

Dear Ben,

what is the E - E' difference you are referring to ?

Is it the difference between the two estimates of the energy ("total 
energy" and "Harris-Foulkes estimate") ?


If so, the first is the expression of the TE from the variational 
principle and is an upper bound to the scf result (not exactly but it 
approaches to it as self consistency gets better), the second is a 
different expression proposed by Harris & Foulkes which is also 
stationary at scf... I don't remember if it is also variational from 
above or just stationary...


anyhow they converge to the same value but are different and their 
difference can give an idea of how far one is from the scf results but 
is not a precise measure of it.


stefano

On 11/10/19 19:01, Ben Comer wrote:


Stefano,

Thanks for the details, I think we have the definitions sorted out. 
But why is the E - E' from the output file not approximately equal to 
the estimated SCF accuracy? It seemed there is always some 
multiplicative factor difference between 1-20.


On 9/28/19 12:51 PM, Stefano de Gironcoli wrote:


Dear Ben,

I think you are right on both accounts... thanks for pointing out the 
problem in eq A7


the equation in reciprocal space equivalent to  A6 and correctly 
coded in PW/src/scf_mod.f90 in function rho_ddot (lines ~450 - 490 )


0.5 4pi e^2 Omega \sum |Delta_rho|^2/|G|^2

it has the correct dimensions of      e^2 x [length^-1]

so A7 should be multiplied by Omega^2/2 to make it correct.


The root of the equivalence between A6 and the (corected) A7 is that

\int exp(+iGr) 1/|r| d3r = 4pi/|G|^2;  rho(r) = \sum_G exp(+iGr) 
rho(G); and


\int exp(+i(G-G')r) d3r = Omega delta(G,G')

where in the last term the integral is done over the unit cell 
because we are looking for a "per cell" quantity.



best regards

stefano


On 27/09/19 21:04, Ben Comer wrote:


I figured the code had to be correct, and are equations A.6 and A.7 
equivalent? It's not trivially obvious that they would be equal as 
one is an integral in real space whereas the other is an integral in 
reciprocal space. If they are the same it looks like A.7 might be 
missing the volume factor and a factor of 1/2.


On 9/27/19 2:28 AM, Paolo Giannozzi wrote:
The code is correct. Eq.A7 likely assumes that rho(G)=\int rho(r) 
exp(-iGr)dr, instead of the more usual definition rho(G)=(1/Omega) 
\int rho(r) exp(-iGr)dr


Paolo

On Thu, Sep 26, 2019 at 11:53 PM Ben Comer <mailto:bcom...@gatech.edu>> wrote:


Dr. Giannozzi,

Per the conversation, I read through the rho_ddot function. In
line 490 of scf_mod.f90 it appears that the volume, omega, is
being multiplied rather than divided as it is in equation A.7
I'm concerned that one of the versions of these equations may
be incorrect. please let me know if I am under a misapprehension.

https://github.com/QEF/q-e/blob/master/PW/src/scf_mod.f90#L490

On 9/26/19 3:57 PM, Paolo Giannozzi wrote:

On Thu, Sep 26, 2019 at 9:44 PM Ben Comer mailto:bcom...@gatech.edu>> wrote:

Just a quick follow up, the text seems to imply that A7 is
used rather than A6. do you know where this is defined in
the source code?

Variable dr2 in PW/src/mix_tho.f90, routine rho_ddot in
PW/src/scf_mix.f90

Paolo

On 9/26/19 10:57 AM, Paolo Giannozzi wrote:

If I remember correctly: Eq.A6 of J.Phys.: Condens.
Matter 21, 395502 (2009)

Paolo

On Thu, Sep 26, 2019 at 4:52 PM Ben Comer
mailto:bcom...@gatech.edu>> wrote:

Hello,

I'm trying to understand what exactly estimated SCF
accuracy is. Does
anyone know where I can find a formal definition of
what it is and how
it is being calculated? The structure of the output
file implies that it
is related to the Harris-Foulkes and the total
energy, but it is clearly
not the trivial difference between these two quantities.

Thanks in advance,
Ben Comer
Georgia Tech

___
Quantum ESPRESSO is supported by MaX
(www.max-centre.eu/quantum-espresso
<http://www.max-centre.eu/quantum-espresso>)
users mailing list users@lists.quantum-espresso.org
<mailto:users@lists.quantum-espresso.org>
https://lists.quantum-espresso.org/mailman/listinfo/users



-- 
Paolo Giannozzi, Dip. Scienze Matematiche Informatiche e

Fisiche,
Univ. Udine, via delle Scienze 208, 33100 Udine, Italy
Phone +39-0432-558216, fax +39-0432-558222


___
Quantum ESPRESSO is supported by MaX (www.max-centre.eu/quantum-espresso  
<http://www.max-centre.eu/quantum-espresso>)
users mailing listus

Re: [QE-users] 回复: unit of dynamical matrix

2019-10-07 Thread Stefano de Gironcoli
The masses written in the dynamical matrix file are?0?2 given in?0?2 
`Ryd-based` atomic unit.


that is

mass_ryd_au?0?2 = mass_amu *?0?2 AMU_SI / ELECTRONMASS_SI * 0.5

where mass_amu in the mass of the atom in the usual atomic mass unit 
(like 12 for Carbon-12 isotope)


AMU_SI?0?2?0?2?0?2?0?2?0?2?0?2?0?2?0?2?0?2?0?2 = 1.660538782E-27_DP?0?2 Kg

ELECTRONMASS_SI?0?2 = 9.10938215E-31_DP?0?2?0?2?0?2 Kg

0.5 is the mass of an electron in Ryd atomic units

as such the frequencies obtained diagonalizing the dynamical matrix are 
such that


\hbar omega are given in rydberg?0?2 (where \hbar = 1 in ryd_au )

stefano


On 05/10/19 11:05,  wrote:

What about the unit of the mass term?
Tong Chen
--  --
*??:*?0?2"Stefano de Gironcoli";
*:*?0?22019??10??1??(??) 5:59
*??:*?0?2"users";
*:*?0?2Re: [QE-users] unit of dynamical matrix

Ryd/bohr^2

stefano


On 01/10/19 10:45,  wrote:

Dear all,
?0?2 ?0?2 What is the unit of the dynamical matrix printed in file *.dyn* ?

Thanks,

Tong Chen

___
Quantum ESPRESSO is supported by MaX (www.max-centre.eu/quantum-espresso)
users mailing listus...@lists.quantum-espresso.org
https://lists.quantum-espresso.org/mailman/listinfo/users


___
Quantum ESPRESSO is supported by MaX (www.max-centre.eu/quantum-espresso)
users mailing list users@lists.quantum-espresso.org
https://lists.quantum-espresso.org/mailman/listinfo/users
___
Quantum ESPRESSO is supported by MaX (www.max-centre.eu/quantum-espresso)
users mailing list users@lists.quantum-espresso.org
https://lists.quantum-espresso.org/mailman/listinfo/users

Re: [QE-users] unit of dynamical matrix

2019-10-01 Thread Stefano de Gironcoli

Ryd/bohr^2

stefano


On 01/10/19 10:45,  wrote:

Dear all,
?0?2 ?0?2 What is the unit of the dynamical matrix printed in file *.dyn* ?

Thanks,

Tong Chen

___
Quantum ESPRESSO is supported by MaX (www.max-centre.eu/quantum-espresso)
users mailing list users@lists.quantum-espresso.org
https://lists.quantum-espresso.org/mailman/listinfo/users
___
Quantum ESPRESSO is supported by MaX (www.max-centre.eu/quantum-espresso)
users mailing list users@lists.quantum-espresso.org
https://lists.quantum-espresso.org/mailman/listinfo/users

Re: [QE-users] Ce19O32 cluster does not convergence

2019-09-29 Thread Stefano de Gironcoli

did you try using  mixing_mode='local_tf' ?

stefano

On 29/09/19 16:08, Andrey Chibisov wrote:

Hello.
I study Ce19O32 cluster, but I can't get the total energy convergence.
My input file is:

calculation ='relax',
restart_mode='from_scratch',
!restart_mode='restart',
wf_collect = .TRUE.,
verbosity='high'
 pseudo_dir='/home/achibisov/CeO2/PBEsol',
 outdir='/home/achibisov/CeO2/Ce19O32/temp',
forc_conv_thr = 0.01
/

ibrav=0,
   celldm(1)=47.2431531141d0,
 nat=51,
   ntyp=2,
  ecutwfc = 80.0,
  ecutrho = 800
  nbnd = 216,
nspin = 2,
tot_magnetization = 0,
!occupations='smearing',
smearing='gaussian', degauss=0.001
!occupations='fixed',
occupations='from_input',
!input_dft='gaup', nqx1=3, nqx2=3, nqx3=3,
!exxdiv_treatment='none'
!x_gamma_extrapolation = .false.
/

conv_thr = 1.D-3,
mixing_beta = 0.1,
/

ion_dynamics = 'bfgs',
/

cell_dynamics = 'bfgs',
cell_dofree = 'all'
/
ATOMIC_SPECIES
  Ce 140.116000d0 Ce.UPF
  O 15.999400d0 O.UPF
ATOMIC_POSITIONS (crystal)
   Ce   0.4868629311d0   0.3815474927d0   0.3558657432d0
 Ce   0.3771040218d0   0.490420d0   0.3549706445d0
   Ce   0.6997754464d0   0.490420d0   0.4702201205d0
 Ce   0.4817073780d0   0.490420d0   0.6789561145d0
   Ce   0.5929365462d0   0.490420d0   0.5766024711d0
 Ce   0.4856620596d0   0.490420d0   0.4648645351d0
   Ce   0.5970259661d0   0.490420d0   0.3568597695d0
 Ce   0.4878552272d0   0.490420d0   0.2520196160d0
   Ce   0.3758834729d0   0.3812732135d0   0.4636105009d0
 Ce   0.5946046648d0   0.5992001660d0   0.4665465423d0
   Ce   0.4834909260d0   0.5989686798d0   0.5739788312d0
 Ce   0.4852252352d0   0.7025437161d0   0.4644374565d0
   Ce   0.3742294968d0   0.490420d0   0.5727685260d0
 Ce   0.4868629311d0   0.5992925073d0   
0.3558657432d0
   Ce   0.3758834729d0   0.5995667865d0   
0.4636105009d0
 Ce   0.2710982326d0   0.490420d0   
0.4624561301d0
   Ce   0.4852252352d0   0.2782962839d0   
0.4644374565d0
 Ce   0.4834909260d0   0.3818713202d0   
0.5739788312d0
   Ce   0.5946046648d0   0.3816398340d0   
0.4665465423d0
  O   0.4284190778d0   0.6603888564d0   
0.4068385834d0
 O   0.4284190778d0   
0.3204511436d0   0.4068385834d0
O   0.4303395760d0   
0.4335412916d0   0.2944091083d0
   O   0.5406516600d0   
0.3208219210d0   0.5230927952d0
  O   0.4268663193d0   
0.3204747807d0   0.5205121157d0
 O   0.4252404411d0   
0.4334727625d0   0.6353234695d0
O   0.4274621721d0  
 0.4342213819d0   0.5206566174d0
   O   
0.4294662094d0   0.4340081656d0   0.4077237855d0
  O   
0.5453391746d0   0.4338342959d0   0.2948938831d0
 O   
0.5436599091d0   0.3208116404d0   0.4083172133d0
O   
0.5435957226d0   0.4343203087d0   0.4082928771d0
   O   
0.6574624760d0   0.4339635776d0   0.4125821189d0
  O 
  0.3132683540d0   0.4337662653d0   0.5196972618d0

 O   0.4303395760d0   0.5472987084d0   0.2944091083d0

O   0.3146749157d0   0.4334990899d0   0.4063334249d0

   O   0.6564105394d0   0.5474457199d0   0.5266229596d0

  O   0.4268663193d0   0.6603652193d0   0.5205121157d0

 O   0.4252404411d0   0.5473672375d0   0.6353234695d0

O   0.3132683540d0   0.5470737347d0   0.5196972618d0

   O   0.4274621721d0   0.5466186181d0   0.5206566174d0
   

Re: [QE-users] What Is Estimated SCF Accuracy

2019-09-28 Thread Stefano de Gironcoli

Dear Ben,

I think you are right on both accounts... thanks for pointing out the 
problem in eq A7


the equation in reciprocal space equivalent to  A6 and correctly coded 
in PW/src/scf_mod.f90 in function rho_ddot (lines ~450 - 490 )


0.5 4pi e^2 Omega \sum |Delta_rho|^2/|G|^2

it has the correct dimensions of      e^2 x [length^-1]

so A7 should be multiplied by Omega^2/2 to make it correct.


The root of the equivalence between A6 and the (corected) A7 is that

\int exp(+iGr) 1/|r| d3r = 4pi/|G|^2;  rho(r) = \sum_G exp(+iGr) rho(G); and

\int exp(+i(G-G')r) d3r = Omega delta(G,G')

where in the last term the integral is done over the unit cell because 
we are looking for a "per cell" quantity.



best regards

stefano


On 27/09/19 21:04, Ben Comer wrote:


I figured the code had to be correct, and are equations A.6 and A.7 
equivalent? It's not trivially obvious that they would be equal as one 
is an integral in real space whereas the other is an integral in 
reciprocal space. If they are the same it looks like A.7 might be 
missing the volume factor and a factor of 1/2.


On 9/27/19 2:28 AM, Paolo Giannozzi wrote:
The code is correct. Eq.A7 likely assumes that rho(G)=\int rho(r) 
exp(-iGr)dr, instead of the more usual definition rho(G)=(1/Omega) 
\int rho(r) exp(-iGr)dr


Paolo

On Thu, Sep 26, 2019 at 11:53 PM Ben Comer > wrote:


Dr. Giannozzi,

Per the conversation, I read through the rho_ddot function. In
line 490 of scf_mod.f90 it appears that the volume, omega, is
being multiplied rather than divided as it is in equation A.7 I'm
concerned that one of the versions of these equations may be
incorrect. please let me know if I am under a misapprehension.

https://github.com/QEF/q-e/blob/master/PW/src/scf_mod.f90#L490

On 9/26/19 3:57 PM, Paolo Giannozzi wrote:

On Thu, Sep 26, 2019 at 9:44 PM Ben Comer mailto:bcom...@gatech.edu>> wrote:

Just a quick follow up, the text seems to imply that A7 is
used rather than A6. do you know where this is defined in
the source code?

Variable dr2 in PW/src/mix_tho.f90, routine rho_ddot in
PW/src/scf_mix.f90

Paolo

On 9/26/19 10:57 AM, Paolo Giannozzi wrote:

If I remember correctly: Eq.A6 of J.Phys.: Condens. Matter
21, 395502 (2009)

Paolo

On Thu, Sep 26, 2019 at 4:52 PM Ben Comer
mailto:bcom...@gatech.edu>> wrote:

Hello,

I'm trying to understand what exactly estimated SCF
accuracy is. Does
anyone know where I can find a formal definition of
what it is and how
it is being calculated? The structure of the output
file implies that it
is related to the Harris-Foulkes and the total energy,
but it is clearly
not the trivial difference between these two quantities.

Thanks in advance,
Ben Comer
Georgia Tech

___
Quantum ESPRESSO is supported by MaX
(www.max-centre.eu/quantum-espresso
)
users mailing list users@lists.quantum-espresso.org

https://lists.quantum-espresso.org/mailman/listinfo/users



-- 
Paolo Giannozzi, Dip. Scienze Matematiche Informatiche e

Fisiche,
Univ. Udine, via delle Scienze 208, 33100 Udine, Italy
Phone +39-0432-558216, fax +39-0432-558222


___
Quantum ESPRESSO is supported by MaX (www.max-centre.eu/quantum-espresso  
)
users mailing listus...@lists.quantum-espresso.org  

https://lists.quantum-espresso.org/mailman/listinfo/users

___
Quantum ESPRESSO is supported by MaX
(www.max-centre.eu/quantum-espresso
)
users mailing list users@lists.quantum-espresso.org

https://lists.quantum-espresso.org/mailman/listinfo/users



-- 
Paolo Giannozzi, Dip. Scienze Matematiche Informatiche e Fisiche,

Univ. Udine, via delle Scienze 208, 33100 Udine, Italy
Phone +39-0432-558216, fax +39-0432-558222


___
Quantum ESPRESSO is supported by MaX (www.max-centre.eu/quantum-espresso  
)
users mailing listus...@lists.quantum-espresso.org  

https://lists.quantum-espresso.org/mailman/listinfo/users




--
Paolo Giannozzi, Dip. Scienze Matematiche Informatiche e Fisiche,
Univ. Udine, via delle Scienze 208, 33100 Udine, 

Re: [QE-users] Some questions about regterg

2019-07-29 Thread Stefano de Gironcoli
g_psi multiplies the correction vector by an approximation of the 
inverse of (H-eS), typically just the inverse of the diagonal .


regterg is the real version of the routine: that is appropriate the one 
for k==Gamma


in this case psi is real in real space which means that the Fourier 
components at G and -G are complex conjugate of each other.


the normalization is as usual

1 = \sum_G psi(G)^* psi(G)   when summing over all G

but only half of them (the "positive" G) are stored and the 
normalization is computed as


1 = psi(0)* psi(0) + 2.0 \sum_G/=0 psi(G)* psi(G)   or rather

1 = 2.0 \sum_G psi(G)* psi(G)   -  psi(0)* psi(0)

the processor with gstart==2 is the one for which the first component is G=0

HTH

stefano

On 29/07/19 15:59, carlossiero siero wrote:

Dear Users,

I have been digging in the regterg.f90 subroutine and I was wondering 
if somebody could tell me what the calling to g_psi (line 286) is doing?


| CALL g_psi( npwx, npw, notcnv, 1, psi(1,nb1), 
ew(nb1) )


I thought the correction vectors, |psi> = (H - e*S) |psi>, were 
already stored in psi, so there is no need to do any inversion or 
anything else.


Also, running a 1processor calculation, the normalization goes through 
line 299:


|      IF ( gstart == 2 ) ew(n) = ew(n) - psi(1,nbn) * 
psi(1,nbn)


What is the purpose of substrating the psi product of the first 
element on each of the new vectors?


Thanks so much for your help!

Carlos

___
Quantum ESPRESSO is supported by MaX (www.max-centre.eu/quantum-espresso)
users mailing list users@lists.quantum-espresso.org
https://lists.quantum-espresso.org/mailman/listinfo/users
___
Quantum ESPRESSO is supported by MaX (www.max-centre.eu/quantum-espresso)
users mailing list users@lists.quantum-espresso.org
https://lists.quantum-espresso.org/mailman/listinfo/users

Re: [QE-users] I need a suggestion

2019-03-21 Thread Stefano de Gironcoli

google-ing  Francium wikipedia...

Bulk francium has never been viewed. Because of the general appearance 
of the other elements in its periodic table column, it is assumed that 
francium would appear as a highly reactive metal, if enough could be 
collected together to be viewed as a bulk solid or liquid. Obtaining 
such a sample is highly improbable, since the extreme heat of decay 
caused by its short half-life would immediately vaporize any viewable 
quantity of the element.


however it also says  (i dont know on the basis of what) : Density (near 
r.t.)    2.8–3.0 g/cm3 (extrapolated)[1]



stefano

On 21/03/19 18:44, yasmin kani wrote:
I am searching for Francium lattice constant for scf calculation and i 
could not find  it. Then I assumed Francium as a body centred cubic 
and calculated its lattice constant as 5.58875 Angstrom. Is it right ?


___
users mailing list
users@lists.quantum-espresso.org
https://lists.quantum-espresso.org/mailman/listinfo/users
___
users mailing list
users@lists.quantum-espresso.org
https://lists.quantum-espresso.org/mailman/listinfo/users

[QE-users] 7 PhD positions at SISSA

2019-02-21 Thread Stefano de Gironcoli

Seven fully-funded PhD positions are available in the Theory and Numerical
Simulation of Condensed Matter curriculum at the International School
for Advances Studies (SISSA), Trieste (Italy), starting in October 2019.

The group of PI of the Condensed Matter (CM) curriculum focuses on the
development and the applications of theoretical methods  - electronic
structure, many-body, non equilibrium statistical mechanics - for
understanding quantum many-body systems in and out of equilibrium as well
as the ab-initio modeling of molecules, solids and nanostructured systems.
For more information, please refer to Condensed Matter SISSA website:
http://cm.sissa.it/

SISSA has been the cradle of the Quantum ESPRESSO project and some of its
historical developers such as Stefano Baroni, Andrea Dal Corso and Stefano
de Gironcoli are CM faculty members at SISSA. Paolo Giannozzi is faculty
member at the University of Udine nearby, and regularly visit SISSA.

Candidates with a degree in physics, chemistry or material science, a
background in physical chemistry/chemical physics, and, more importantly,
strong motivation and enthusiasm for scientific research and software
development are strongly encouraged to apply. Both EU and International
candidates are eligible and can apply following the on-line
procedure at https://pica.cineca.it/sissa/phd-cmp-01-2019/.

Dead line for applications: March 21, 2019.
Candidates with positive academic and scientific qualifications evaluation
will be admitted to the Oral Exam: from April 8 to April 10, 2019.
Candidates will be informed about day and time of the oral exam that,
upon request, could be taken by videoconference, preferably through
Skype. Outcome of the selection will be notified shortly after the
completion of the interviews.

Check the Admission Information Sheet:
https://www.sissa.it/sites/default/files/CMP_1.pdf

___
users mailing list
users@lists.quantum-espresso.org
https://lists.quantum-espresso.org/mailman/listinfo/users

Re: [QE-users] about 'lforcet' flag

2019-01-04 Thread Stefano de Gironcoli
The way I understand the documentation is that if you want to start a 
non collinear magnetic calculation from the result of a collinear one 
you can use the angles defining the magnetic orientation of the first 
atomic species (that has no meaning in the collinear case) to rotate the 
direction of the collinear magnetization (conventionally along z) in the 
desired direction. This applies to the whole space and is just the 
starting guess. The subsequent self-consistency will modify it as 
required by the non collinear energy minimization, but the closer you 
start to the right orientation the better it should be.


stefano


On 03/01/19 11:33, Guido Menichetti wrote:


Dear experts and developers,

I am trying to use 'lforcet' flag.
The documentation says that this flag works
only for the rotation of the spin of the atomic type 1:

"[...] direction described byangle1  
  andangle2  
  variables for_*atomic type 1" 
*_
I was wondering if it works also if I rotate more than 1 atom/atomic 
species at the same time.


Thank you for all your work,
Kind Regards
G.

--
***
Guido Menichetti
Post-Doc researcher in Solid State Physics
Istituto Italiano di Tecnologia
Graphene Labs: Theory and technology of 2D materials
Address: Via Morego, 30, 16163 Genova
Email: guido.meniche...@iit.it 
guido.meniche...@df.unipi.it 
menichetti.gu...@gmail.com 


___
users mailing list
users@lists.quantum-espresso.org
https://lists.quantum-espresso.org/mailman/listinfo/users
___
users mailing list
users@lists.quantum-espresso.org
https://lists.quantum-espresso.org/mailman/listinfo/users

Re: [QE-users] Question about optimizing the lattice constant of iron

2018-12-25 Thread Stefano de Gironcoli

cutoff might be too small for a reliable computation of the stress.

other than that I see no big issue in your input

stefano

On 25/12/18 04:35, Gui Wei wrote:

Hi,
When optimizing the lattice constant of bcc Fe,the result is a=2.830A, 
which is in disagreement with experimentally derived structural 
parameter a=2.863A.Could someone give me some advice?



    calculation = 'vc-relax'
    prefix='Fe1',
    pseudo_dir = '/public/home/duan1/gw2/pseudo/',
outdir='/public/home/duan1/gw2/tempdir/'
    tprnfor     = .true.
    tstress     = .true.
/

    ibrav= 1,
    A=2.8664
    nat= 2
    ntyp= 1
    occupations='smearing',
    smearing='mp',
    degauss=0.02,
    ecutwfc =37
    ecutrho =240
    nspin   = 2
    starting_magnetization(1) = 0.4
/

    conv_thr        = 1.0d-8
    mixing_beta      = 0.7
/

   ion_dynamics = 'bfgs'
/

   cell_dynamics = 'bfgs'
/
ATOMIC_SPECIES
Fe  55.847 Fe.pbe-n-kjpaw_psl.0.2.4.UPF
ATOMIC_POSITIONS crystal
 Fe   0.  0.   0.
 Fe   0.5001  0.5001   0.5001
K_POINTS automatic
8  8  8  0  0  0

The result:
 CELL_PARAMETERS (alat=  5.41671099)
   0.987418499   0.0  0.0
   0.0   0.987418499  0.0
   0.0   0.0  0.987418499



Gui Wei
School of Mechanical Engineering,Chongqing University, China


___
users mailing list
users@lists.quantum-espresso.org
https://lists.quantum-espresso.org/mailman/listinfo/users
___
users mailing list
users@lists.quantum-espresso.org
https://lists.quantum-espresso.org/mailman/listinfo/users

Re: [QE-users] How is the wavefunction supposed to be normalized?

2018-12-11 Thread Stefano de Gironcoli
yes, except the S matrix is never stored anywhere but is applied to the 
wfc by the s_1psi routine,


so more something like


 sum = 0.d0

 call s_1psi( npwx, npw, evc(1,b), spsi )

 DO  i = 1,npw

 sum = sum + conjg(evc(i,b)) * spsi(i)
 END DO



stefano


On 11/12/18 15:36, John McFarland wrote:


Dear all,


As I understand it, the wavefunction for each k-point is given by 
ecv(:,:), with the first index representing a single particle basis 
and the second index representing each band.  I'm guessing that when a 
normalized wavefunction is contracted over the S matrix, the result 
should equal one for each band.  What I think this would look like in 
code is:



 sum = 0

 DO  i = 1,npwx

    DO j = 1,npwx

       sum = sum + evc(i,b) * S(i,j) * evc(j,b)

    END DO

 END DO


and sum should equal 1 for any band b.  Is my understanding correct?


Best regards,

John McFarland


___
users mailing list
users@lists.quantum-espresso.org
https://lists.quantum-espresso.org/mailman/listinfo/users
___
users mailing list
users@lists.quantum-espresso.org
https://lists.quantum-espresso.org/mailman/listinfo/users

Re: [QE-users] Subroutine for renormalization

2018-12-07 Thread Stefano de Gironcoli
not directly ... but the routine  PW/src/s_1psi.f90 computes S*psi for 
one wavefunction


then  should be the quantity needed to normalize the 
wavefunction psi


stefano

On 07/12/18 14:42, John McFarland wrote:

Is there a subroutine in QE that normalizes the wavefunction?

Get Outlook for Android 


*From:* John McFarland
*Sent:* Friday, December 7, 2018 8:37:51 AM
*To:* Quantum Espresso users Forum
*Subject:* Re: [QE-users] Subroutine for renormalization
Sorry that's what I meant.

Get Outlook for Android 


*From:* users  on behalf of 
Paolo Giannozzi 

*Sent:* Friday, December 7, 2018 5:00:12 AM
*To:* Quantum Espresso users Forum
*Subject:* Re: [QE-users] Subroutine for renormalization
What do you mean by "renormalize the wave function"? "normalize"?

Paolo

On Thu, Dec 6, 2018 at 10:50 PM John McFarland > wrote:


Dear all,


Is there a subroutine in Quantum ESPRESSO that renormalizes the
wave function?


Thanks,


John McFarland

___
users mailing list
users@lists.quantum-espresso.org

https://lists.quantum-espresso.org/mailman/listinfo/users





--
Paolo Giannozzi, Dip. Scienze Matematiche Informatiche e Fisiche,
Univ. Udine, via delle Scienze 208, 33100 Udine, Italy
Phone +39-0432-558216, fax +39-0432-558222


___
users mailing list
users@lists.quantum-espresso.org
https://lists.quantum-espresso.org/mailman/listinfo/users
___
users mailing list
users@lists.quantum-espresso.org
https://lists.quantum-espresso.org/mailman/listinfo/users

Re: [QE-users] Adsorption of Cations and Anions on Metal-organic frameworks (MOF)

2018-11-30 Thread Stefano de Gironcoli
assume_isolated is an option that applies to a molecule or a cluster 
surrounded by vacuum.


Does not work for a defect inside a solid.

one could estimate the correction in  a way similar to the Makov-Payne  
method scaling the MP correction with the MOF dielectric constant. MP is 
a correction only to the energy so the density and potential are not 
affected.


stefano

On 30/11/18 19:15, Mohamed Safy wrote:

Dear QE users
I am trying to study the adsorption if some Cations and Anions on the 
pore of Metal-organic frameworks (MOF). I relaxed the cell which 
contains the ion and added tot_charge=1 or tot_charge=-1 for the cell 
containing cation and anion. I took the optimized coordinates of the 
complex, MOF and ion and performed scf calculations with higher 
/ecutwfc /and denser K-points. I added the tot_charge keyword as follow:

tot_charge = 1 for the complex
tot_charge =1 for the ion *i add **assume_isolated = 'mt'*
removed the keyword for the MOF.
I calculate the adsorption energy as follow:
E_Adsorbtion= EComplex -EMOF- Eadsorbate
but I found an overestimated binding energy for my calculations.
any advice or suggestions, please
Thanks in Advance



___
users mailing list
users@lists.quantum-espresso.org
https://lists.quantum-espresso.org/mailman/listinfo/users
___
users mailing list
users@lists.quantum-espresso.org
https://lists.quantum-espresso.org/mailman/listinfo/users

Re: [QE-users] Lowdin charge not equal to the total number of electrons

2018-08-15 Thread Stefano de Gironcoli

0.0244 * 166 = 4.0504 ... i guess this is what it is.

stefano


On 15/08/2018 22:56, Fernando Soto wrote:


I want to calculate the Lowdin charges in my system (see input info. 
below) using espresso-5.4.0. The total number of electrons in my 
system is 166 |e| but according to the Lowdin charges the number of 
electrons in my system is close to 162 |e|. That is, I am missing 
approximately 4 electrons. So, why is the sum of partial Lowdin 
charges not equal to the total charge?


I am aware that the missing charge may be delocalized (see link 
below). But since my spilling parameter is 0.0244 I was expecting the 
Lowdin charge to be close to 166 |e|, not missing 4 |e|.

https://www.quantum-espresso.org/resources/faq/self-consistency#6.6

Any help with this issue is much appreciated.
Thanks,
Fernando A. Soto
Postdoctoral Research Associate
Texas A University

Pseudopotentials:
Li.pbe-s-kjpaw_psl.0.2.1.UPF
F.pbe-n-kjpaw_psl.0.1.UPF
N.pbe-n-kjpaw_psl.0.1.UPF
O.pbe-n-kjpaw_psl.0.1.UPF
S.pbe-n-kjpaw_psl.0.1.UPF


 calculation = 'scf',
 disk_io='low'
 pseudo_dir = 'xxx',
 outdir = 'xxx',
 tefield = .true.,
 dipfield = .true.,
/

 ibrav=0,
 nat=46, ntyp=5,
 ecutwfc = 80,
 ecutrho = 800,
 occupations = 'smearing',
 smearing = 'gaussian',
 degauss = 0.014,
 nosym = .true.,
 edir = 3,
 eamp = 0.001,
 emaxpos = 0.90,
 eopreg = 0.05,
/

  conv_thr    = 7.35E-05,
  mixing_beta = 0.3D0,
/

/
ATOMIC_SPECIES
  Li 6.941d0  Li.pbe-s-kjpaw_psl.0.2.1.UPF
  F  18.998d0 F.pbe-n-kjpaw_psl.0.1.UPF
  O  8.0d0    O.pbe-n-kjpaw_psl.0.1.UPF
  S  30.973d0 S.pbe-n-kjpaw_psl.0.1.UPF
  N  14.0067d0 N.pbe-n-kjpaw_psl.0.1.UPF

CELL_PARAMETERS angstrom
10.5276002884         0.00         0.00
0.00        10.5276002884         0.00
0.00         0.00        32.0183982849

ATOMIC_POSITIONS angstrom
Li       5.265484338   4.719733776  22.007205940    0  0   0
Li       5.264957870   8.188788801  21.994077740    0  0   0
Li       1.772531993   1.202041387  21.983833210    0  0   0
Li       8.757805267   1.201620260  21.981911400    0  0   0
Li       8.741592708   4.694046373  21.975506660    0  0   0
Li       1.788955076   4.694362316  21.974546710    0  0   0
Li       8.755489187   8.227424854  21.972945530    0  0   0
Li       1.775479802   8.227319435  21.972306200    0  0   0
Li       5.264432031   1.212674270  21.968143880    0  0   0
Li       7.015908623   6.457525041  20.272768900    0  0   0
Li       3.515060366   6.457103365  20.272129570    0  0   0
Li       6.998117289   2.968783337  20.245873180    0  0   0
Li       3.532009917   2.969098966  20.245233850    0  0   0
Li       3.525588145   9.962899725  20.240750910    0  0   0
Li       7.003696337   9.962689515  20.240750910    0  0   0
Li       0.002000244   2.927304423  20.236267980    0  0   0
Li       0.000842208   6.459209234  20.234666790    0  0   0
Li       0.002631900  10.005746890  20.228263960    0  0   0
Li       8.560136239   4.727283938  18.193254470
Li       2.409090639   5.247078089  18.135019876
Li       9.338356878   1.232441496  18.315151690
Li       9.301720965   8.404424023  18.064318334
Li       1.900450217   1.462959364  18.278036192
Li       1.569486464   8.247262181  18.418232045
Li       5.126232747   8.513706494  18.113903613
Li       5.540135528   1.547095495  18.020268176
Li       5.422547465   4.705484378  18.608713439
Li       0.132063306  10.636744925  16.055070194
Li       0.136769188   3.411757549  16.928096549
Li       6.773427219   6.433991188  16.656220542
Li       3.496746817   6.764292996  15.782209259
Li      10.548369731   6.741043353  16.078376653
Li       3.368232699   9.954133073  16.503248574
Li       7.273469732   9.978166702  15.942689502
Li       7.936998222   2.460042680  16.467257451
Li       3.168125695   2.695451737  16.284299947
Li       5.492631088   3.473792016  15.756491939
F        7.067605427   2.777657263  11.877296164
F        4.032991602   6.088199959  14.258992850
O        3.936147062   3.181492237  14.493789613
O        6.634146778   3.412806352  14.312769529
O        7.499204696   5.058340086  12.671275898
O        2.588481881   4.134972702  12.524475689
S        6.580790466   3.980834432  12.891680874
S        3.823265586   4.235478859  13.273917402
N        5.156903210   4.088811602  12.210249283

K_POINTS (gamma)



    prefix = 'pwscf'
    outdir = 'xxx'
    ngauss = 0,
    degauss = 0.01,
    DeltaE = 0.01,
 /




___
users mailing list
users@lists.quantum-espresso.org
https://lists.quantum-espresso.org/mailman/listinfo/users


___
users mailing list
users@lists.quantum-espresso.org
https://lists.quantum-espresso.org/mailman/listinfo/users

Re: [QE-users] Difference in phonon frequencies produced by matdyn.x

2018-08-15 Thread Stefano de Gironcoli
I think the reason of the difference is the presence of macroscopic 
electric fields that makes the phonon dispersion close to gamma non 
-analytic (it depends on the direction  of approach to Gamma point). 
Accounting for this non-anayticity is the reason why one computes the 
effective charges and the dielectric matrix.


when you select the path (with matdyn.x) the code knows which direction 
you are approaching Gamma and includes the non analytic terms 
accordingly. you get therefore the LO-TO splitting of modes that would 
be otherwise degenerate (174.65). The TO mode stays there while the LO 
is shifted up to 322). The other modes are not changed because they 
don't have a macroscopic polarization.


When calculation the Dos the code does not know which direction to adopt 
and does not include the LO-TO splitting .. the TO matrix is 
diagonalized that have additional symmetry. This is in principle wrong 
but the gamma point is just a single point in the DoS. neighboring 
points and the rest of the points in the BZ should be computed correctly.


HTH

stefano



On 15/08/2018 23:24, Jie Peng wrote:

Dear QE users:

I have been running phonon calculations of HfS2 using quantum 
espresso. I followed the steps of relaxing to the equilibrium atomic 
configuration using pw.x, computing dynamical matrices on a k point 
mesh grid using ph.x, producing interatomic force constant matrix 
using q2r.x.


Now I want to plot phonon dispersion along high symmetry direction 
Gamma-M-K-Gmma in the HCP (hexagonal closed pack) HfS2 lattice. So I 
used the following input file for matdyn.x:


/ /
/    asr='crystal'/
/    amass(1)=178.49,/
/    amass(2)=32.065,/
/    ntyp=2/
/    flfrc='HfS2.fc', flfrq='HfS2.freq', q_in_band_form=.true.,/
/ //
/ 4/
/ 0 0 0 40/
/0.5 -0.2887 0 40/
/0.6667 0 0 40/
/0 0 0 1/

which gives me phonon frequencies at Gamma point as:

/   diagonalizing the dynamical matrix .../
/
/
/ q =       0.      0.      0./
/ **/
/     freq (    1) =      -0.00 [THz] = -0.01 [cm-1]/
/ (  0.545987  -0.00    -0.167781  -0.00 -0.084152  -0.00   )/
/ (  0.545987  -0.00    -0.167781  -0.00 -0.084152  -0.00   )/
/ (  0.545987  -0.00    -0.167781  -0.00 -0.084152   0.00   )/
/     freq (    2) =       0.00 [THz] =  0.04 [cm-1]/
/ (  0.166188   0.00     0.552429   0.00 -0.023176  -0.00   )/
/ (  0.166188   0.00     0.552429   0.00 -0.023176  -0.00   )/
/ (  0.166188   0.00     0.552429   0.00 -0.023176   0.00   )/
/     freq (    3) =       0.00 [THz] =  0.08 [cm-1]/
/ (  0.087255   0.00    -0.002306   0.00  0.570714   0.00   )/
/ (  0.087255   0.00    -0.002306   0.00  0.570714   0.00   )/
/ (  0.087255   0.00    -0.002306   0.00  0.570714   0.00   )/
/     freq (    4) =       5.235913 [THz] =  174.651242 [cm-1]/
/ (  0.123126   0.00     0.213241   0.00 -0.00  -0.00   )/
/ ( -0.342690  -0.00    -0.593505  -0.00  0.00   0.00   )/
/ ( -0.342690  -0.00    -0.593505  -0.00  0.00   0.00   )/
/     freq (    5) =       7.799484 [THz] =  260.162777 [cm-1]/
/ ( -0.00   0.00     0.00  -0.00  0.00   0.00   )/
/ (  0.707104  -0.13     0.001900  -0.00 -0.00  -0.00   )/
/ ( -0.707104   0.13    -0.001900   0.00 -0.00   0.00   )/
/     freq (    6) =       7.799484 [THz] =  260.162777 [cm-1]/
/ ( -0.00  -0.00    -0.00  -0.00 -0.00   0.00   )/
/ (  0.001900   0.00    -0.707104  -0.00  0.00  -0.00   )/
/ ( -0.001900  -0.00     0.707104   0.00  0.00   0.00   )/
/     freq (    7) =       9.162102 [THz] =  305.614822 [cm-1]/
/ ( -0.00   0.00     0.00   0.00 -0.246235   0.00   )/
/ (  0.00  -0.00    -0.00   0.00  0.685335  -0.00   )/
/ (  0.00   0.00    -0.00  -0.00  0.685335   0.00   )/
/     freq (    8) =       9.170687 [THz] =  305.901189 [cm-1]/
/ ( -0.213241   0.00     0.123126  -0.00  0.00  -0.00   )/
/ (  0.593505  -0.00    -0.342690   0.00 -0.00   0.00   )/
/ (  0.593505  -0.00    -0.342690   0.00 -0.00   0.00   )/
/     freq (    9) =      10.162710 [THz] =  338.991522 [cm-1]/
/ (  0.00   0.00    -0.00  -0.00 -0.00  -0.00   )/
/ ( -0.00  -0.00     0.00   0.00 -0.707107   0.00   )/
/ ( -0.00  -0.00    -0.00   0.00  0.707107   0.00   )/
/ **/

However, when I tried to compute phonon DOS in which a mesh grid 
rather than a list of high symmetry kpoints was declared, a different 
set of Gamma point phonon frequencies appeared. The input file for DOS 
calculation is:


/ /
/    asr='crystal'/
/    amass(1)=178.49,/
/    

Re: [QE-users] Potential issue with charge density cutoff convergence and K-point grid convergence

2018-04-25 Thread Stefano de Gironcoli

Dear Martina Lessio,

  the 8*ecutwfc is a rule of thumb that is indeed often suggested for 
US pseudopotential.


  There is nothing special in the factor 8, it simply reflects the 
expectation that augmentation charges typically contain more Fourier 
components than the 4*ecutwfc coming from the square of the wavefuncitons.


   As you did the converge test and found that 55/280 Ry are good in 
your case it should be ok to use these values.


   best,

stefano


On 25/04/2018 22:11, Martina Lessio wrote:

Dear Stefano,

I have one more question regarding the convergence tests. I have 
performed the tests in the order that you recommended and found that 
if I set ecutrho=280 Ry I can get away with ecutwfc=55 Ry without 
negatively impacting the convergence. So I am thinking of using these 
parameters for my future calculations. However, I have read in many QE 
resources that if you use ultrasoft pseudopotentials like I do, 
ecutrho should be at least equal to 8*ecutwfc. So I am now wondering 
whether it is an issue that in my case ecutrho is only equal to about 
5*ecutwfc or given that I performed the recommended convergence tests 
it is okay to have this setup even for pseudopotentials.


I hope my question is clear.
Thanks so much!

All the best,
Martina

Martina Lessio
Postdoctoral Research Scientist
Department of Chemistry
Columbia University


On Tue, Apr 24, 2018 at 10:53 AM, Martina Lessio <ml4...@columbia.edu 
<mailto:ml4...@columbia.edu>> wrote:


Dear Stefano,

Thank you very much for your prompt response, that helps a lot!
I will repeat the tests as you suggested but it's good to know
that my system is behaving normally and I can then proceed with
more complex calculations.

All the best,
Martina

On Tue, Apr 24, 2018 at 3:23 AM, Stefano de Gironcoli
<degir...@sissa.it <mailto:degir...@sissa.it>> wrote:

Dear Martina Lessio,

   first of all I would say that a convergence error of 1.d-5
Ry in a 6 atom cell looks pretty much converged to me. I think
that even your ecutrho = 240 Ry calculation (~1.d-5 Ry per
atom) looks quite good.

   coming to the way things converge:

   - total energy convergence with respect to ecutwfc is
expected to be from above because of variational principle:
the higher ecutwfc is the more planewaves are included in the
wavefunction expansion, hence the lower the energy. However in
the ultrasoft pseudopotential definition the energy is not
only a function of the wavefunctions but also includes a
dependence on augmentation charges, that are localized and may
contain higher Fourier components with respect to 4*ecutwfc (
= 240 in your case). Failing to include enough Fourier
components in the augmentation charges will affect a number of
integrals but not in a variational way... integrals would
simply be inaccurate and the inaccuracy can be both from above
or from below.

   - I would perform cutoff convergence test in a slightly
different order: 1) I would check convergence of total energy
(and stress, and forces) as a function of ecutwfc using the
default value for ecutho=4*ecutwfc (that is without specifying
ecutrho in the input). When this procedure converges (and it
can initially converge from below due to augmentation charge
Fourier components being missing) this means that wavefunction
expansion AND augmentation-charge expansion are both
converged. 2) I would then fix ecutrho=4*converged_ecutwfc,
which takes care of augmentation charge convergence, and I
would check whether I can get away with a lower ecutwfc for
the wavefunction expansion.

    - as for k-point sampling convergence, there is no
variational principle w.r.t. number of k-points: it's again a
matter of convergence of an integral. The denser the grid the
better the integral but there is no variational principle with
respect to which k-point you include and which you dont.

  hope this helps

stefano


On 24/04/2018 05:56, Martina Lessio wrote:

Dear Quantum Espresso community,

I am new to Quantum Espresso and I am trying to run some
simple simulations on MoTe2 bulk. Unfortunately I seem to be
having some issues with some preliminary convergence tests
for charge density cutoff and K-point grid and I am hoping to
get some help from you on this.
Here is a graph with the results of the charge density cutoff
convergence test I performed while setting the kinetic energy
cutoff equal to 60 Ry (I performed a test to set this as well):

I am worried about these results because I would expect the
total energy to go down rather than going up when I increase

Re: [QE-users] Potential issue with charge density cutoff convergence and K-point grid convergence

2018-04-24 Thread Stefano de Gironcoli

Dear Martina Lessio,

   first of all I would say that a convergence error of 1.d-5 Ry in a 6 
atom cell looks pretty much converged to me. I think that even your 
ecutrho = 240 Ry calculation (~1.d-5 Ry per atom) looks quite good.


   coming to the way things converge:

   - total energy convergence with respect to ecutwfc is expected to be 
from above because of variational principle: the higher ecutwfc is the 
more planewaves are included in the wavefunction expansion, hence the 
lower the energy. However in the ultrasoft pseudopotential definition 
the energy is not only a function of the wavefunctions but also includes 
a dependence on augmentation charges, that are localized and may contain 
higher Fourier components with respect to 4*ecutwfc ( = 240 in your 
case). Failing to include enough Fourier components in the augmentation 
charges will affect a number of integrals but not in a variational 
way... integrals would simply be inaccurate and the inaccuracy can be 
both from above or from below.


   - I would perform cutoff convergence test in a slightly different 
order: 1) I would check convergence of total energy (and stress, and 
forces) as a function of ecutwfc using the default value for 
ecutho=4*ecutwfc (that is without specifying ecutrho in the input). When 
this procedure converges (and it can initially converge from below due 
to augmentation charge Fourier components being missing) this means that 
wavefunction expansion AND augmentation-charge expansion are both 
converged. 2) I would then fix ecutrho=4*converged_ecutwfc, which takes 
care of augmentation charge convergence, and I would check whether I can 
get away with a lower ecutwfc for the wavefunction expansion.


    - as for k-point sampling convergence, there is no variational 
principle w.r.t. number of k-points: it's again a matter of convergence 
of an integral. The denser the grid the better the integral but there is 
no variational principle with respect to which k-point you include and 
which you dont.


  hope this helps

stefano


On 24/04/2018 05:56, Martina Lessio wrote:

Dear Quantum Espresso community,

I am new to Quantum Espresso and I am trying to run some simple 
simulations on MoTe2 bulk. Unfortunately I seem to be having some 
issues with some preliminary convergence tests for charge density 
cutoff and K-point grid and I am hoping to get some help from you on this.
Here is a graph with the results of the charge density cutoff 
convergence test I performed while setting the kinetic energy cutoff 
equal to 60 Ry (I performed a test to set this as well):


I am worried about these results because I would expect the total 
energy to go down rather than going up when I increase ecutrho. I also 
observe a similar energy trend when I increase the k-point grid, which 
also seems unusual and possibly wrong to me.
I am copying below the input I have used for these calculations and I 
would greatly appreciate any help with figuring our whether I am doing 
something wrong.


Thank you so much!

Kind Regards,
Martina Lessio

Postdoctoral Research Scientist
Department of Chemistry
Columbia University

 

  calculation = 'scf'

  restart_mode='from_scratch',

  prefix='MoTe2_ecutwfc',

  pseudo_dir = '/home/mlessio/espresso-5.4.0/pseudo/',

  outdir='/home/mlessio/espresso-5.4.0/tempdir/'

 /

 

  ibrav= 4, A=3.530, B=3.530, C=13.882, cosAB=-0.5, cosAC=0, cosBC=0,

  nat= 6, ntyp= 2,

  ecutwfc =60.0 ecutrho=300.

  nspin =4, lspinorb =.true., noncolin=.true.

 /

 

  mixing_mode = 'plain'

  mixing_beta = 0.7

  conv_thr =  1.0d-8

 /

ATOMIC_SPECIES

 Mo 95.96 Mo.rel-pbe-spn-rrkjus_psl.1.0.0.UPF

 Te 127.6 Te.rel-pbe-n-rrkjus_psl.1.0.0.UPF

ATOMIC_POSITIONS {crystal}

Te     0.4         0.66643 0.62534

Te     0.66641         0.33282 0.37500

Te     0.66641         0.33282 0.12500

Te     0.4         0.66643 0.87466

Mo     0.4         0.66643 0.25000

Mo     0.66641         0.33282 0.75000


K_POINTS {automatic}

8 8 2 0 0 0



___
users mailing list
users@lists.quantum-espresso.org
https://lists.quantum-espresso.org/mailman/listinfo/users


___
users mailing list
users@lists.quantum-espresso.org
https://lists.quantum-espresso.org/mailman/listinfo/users

Re: [QE-users] NVE simulation in cp.x - large fluctuation in temperature

2018-04-14 Thread Stefano de Gironcoli

Dear Jie Peng,

 suppose you were running a model harmonic system in 1 dimension.

  M a = - K x

 at fixed energy  E.

 The kinetic energy would fluctuate harmonically between 0 (at 
maximum/minimum elongation) and E at the equilibrium distance .


 On average the Kinetic energy would be E/2 and its fluctuation some 
big fraction of E^2


 Something like sigma^2 = 1/T \int_0^T (E cos^2(2pi t/T) -E/2)^2 dt = 
E^2 1/T \int_0^T (cos(4pi t/T)/2)^2 dt = (E/2)^2 1/2pi \int_0^2pi 
cos^2(x) dx = (E/2)^2 / 2


  or

  sigma =  1/sqrt(2) * E/2 = 1/sqrt(2) avg EKin

  with 1 degree of freedom the mean square fluctuation of the kinetic 
energy is 70% of its average !


  you have 3 atoms in your cell hence 9 degrees of freedom. Assuming 
each contributes independently to the average this goes down by a factor 
1/sqrt(9)=1/3


  actually more likely just 1/sqrt(6) as the total momentum is 
conserved so only 6 modes at Gamma are actually excited...


  If you perform your simulation in a bigger supercell with more atoms 
(more degrees of freedom) the average will be more stable ( 
proportionally to  1/sqrt(#deg.of.freedom-3 )  ... moreover the thermal 
excitations of vibrational modes will be sampled more faithfully.


  best

stefano


On 13/04/2018 21:39, Jie Peng wrote:

Dear all

I have been running MD simulations on HfS2 using cp.x code in Quantum 
espresso. I start from initial configuration obtained from pwscf 
vc-relax, and relax the system using cp.x by consecutive steps of: 
electron relaxation->ionic relaxation->cell relaxation. Then, I just 
directly start a NVE simulation starting from the equilibrium 
configuration. I expect the system to almost stay stationary or the 
temperature should be very small since I am allowing dynamics in a 
system that is already in equilibrium. However, what I see is a huge 
fluctuation in the /tmpp/ output of cp.x, as I attach a figure showing 
variation of tmpp (Ionic temperature) with simulation time


I did this because it is suggested in the user guide you should apply 
an initial displacement to the atoms in your system after the 
relaxation since otherwise there will not be any dynamics. But what I 
see here is a large fluctuation of the system temperature.


The thinking or questions here are

1.Does the tmpp represents the physical temperature of the system 
here? I think it should be since it is the temperature corresponding 
to kinetic energy of the ions.


2.It above point is true, why is the temperature varying so fiercely? 
Am I setting incorrect parameters, for instance the timestep or the 
fictitious mass? But I took those from previous simulation steps where 
I did the relaxation, and they all worked well since they successfully 
drived my system to equilibrium, satisfying the convergence threshold 
on total energy, forces acting on atoms, and the fictitious electron 
kinetic energy. I am confused at this point.


The input file for NVE simulation is attached here:

//
/    calculation='cp',/
/    title='Halfnium disulfide'/
/    restart_mode='restart',/
/    ndr=53,/
/    ndw=54,/
/    nstep=5,/
/    iprint=10/
/    isave=100,/
/    tstress = .true./
/    tprnfor = .true./
/    dt=10,/
/    wf_collect=.true./
/    etot_conv_thr=1e-6/
/    forc_conv_thr=1e-3/
/    ekin_conv_thr=1e-5/
/    prefix='HfS2',/
/    pseudo_dir='/home/jpeng/HfS2/potential'/
/    outdir='./tmp/',/
/ //
/ /
/    ibrav= 4,/
/    a=3.6529/
/   c=5.6544/
/    nat=  3, ntyp= 2,/
/    ecutwfc =50/
/    vdw_corr='DFT-D',/
/ !   lspinorb=.true./
/ !   noncolin=.true./
/ !   ecutrho=300/
/ !   nbnd=14/
/!    occupations='smearing'/
/!    smearing='gaussian'/
/!   degauss=0.01/
/ !  nspin=2/
/ !   starting_magnetization(1)=0.1/
/! Hf  95.94  Hf.pbe-mt_fhi.UPF/
/! S  32.065  S.pbe-mt_fhi.UPF/
///
/ /
/    electron_dynamics='verlet'/
/    electron_velocities='zero'/
/    emass=400/
/    emass_cutoff=1/
///
/ /
/    ion_dynamics = 'verlet'/
/    ion_damping=0.1/
/!    ion_nstepe=10/
/ //
/ /
/    cell_dynamics = 'none'/
//
///
/ATOMIC_SPECIES/
/ Hf  95.94  Hf.pbe-mt_fhi.UPF/
/ S  32.065  S.pbe-mt_fhi.UPF/
/ATOMIC_POSITIONS (crystal)/
/Hf      -0.0  -0.0  -0.0/
/S        0.7   0.3   0.257234636/
/S        0.3   0.7  -0.257234636/

Anyone could help me on it? Thank you very much.

Best
Jie
--

Jie Peng
PhD student
2134 Glenn Martin Hall, Mechanical Engineering, University of Maryland
College Park, Maryland, USA
Phone:(+1) 240-495-9445
Email: jiep...@umd.edu 



___
users mailing list
users@lists.quantum-espresso.org
https://lists.quantum-espresso.org/mailman/listinfo/users


___
users mailing list
users@lists.quantum-espresso.org
https://lists.quantum-espresso.org/mailman/listinfo/users

Re: [QE-users] Wrong adsorption energy

2018-03-08 Thread Stefano de Gironcoli

two things I think might be the origin of strange results.

your ethyl-anion is charged. the code introduce a compensating 
background that should cancel out when you calculate the adsortion 
energy but you are using different cells in the two charged 
configurations. so something does not cancel out as it should.


there might be some vdW interaction contributing to the physisorption. I 
see no inclusion of vdW corrections or use of vdW-aware functionals in 
your calculations.



best


stefano


On 08/03/2018 15:50, Laurens Siemons wrote:


Dear all,


I'm calculating parameters like adsorption energy, geometric 
parameters, NMR chemical shifts of adsorbates on a surface. In this 
specific case I calculated the adsorption energy of an ethyl anion on 
an anatase 001 surface of titania. I got the following energies:



E_surface + adsorbate = -4301.05337244

E_surface = -4258.9387474

E_adsorbate = -42.37757742


So E_ads = E_surface + adsorbate - (E_surface + E_adsorbate ) = 0.26


Now this seems wrong. If E_ads is positive, so adsorption is not 
favourable, then the ethyl-anion should migrate away from the surface 
in the relax calculation of a001cti. correct? I executed the three 
calculations with the exact same parameters as the other ones so I 
don't see how this is possible. I attached my input files of the 
calculations. I hope someone can guide me through this.



Thanks in advance,

Laurens Siemons

Master student chemistry

University of Antwerp



___
users mailing list
users@lists.quantum-espresso.org
https://lists.quantum-espresso.org/mailman/listinfo/users


___
users mailing list
users@lists.quantum-espresso.org
https://lists.quantum-espresso.org/mailman/listinfo/users

Re: [QE-users] Wrong adsorption energy

2018-03-08 Thread Stefano de Gironcoli

two things I think might be the origin of strange results.

your ethyl-anion is charged. the code introduce a compensating 
background that should cancel out when you calculate the adsortion 
energy but you are using different cells in the two charged 
configurations. so something does not cancel out as it should.


there might be some vdW interaction contributing to the physisorption. I 
see no inclusion of vdW corrections or use of vdW-aware functionals in 
your calculations.



best


stefano


On 08/03/2018 15:50, Laurens Siemons wrote:


Dear all,


I'm calculating parameters like adsorption energy, geometric 
parameters, NMR chemical shifts of adsorbates on a surface. In this 
specific case I calculated the adsorption energy of an ethyl anion on 
an anatase 001 surface of titania. I got the following energies:



E_surface + adsorbate = -4301.05337244

E_surface = -4258.9387474

E_adsorbate = -42.37757742


So E_ads = E_surface + adsorbate - (E_surface + E_adsorbate ) = 0.26


Now this seems wrong. If E_ads is positive, so adsorption is not 
favourable, then the ethyl-anion should migrate away from the surface 
in the relax calculation of a001cti. correct? I executed the three 
calculations with the exact same parameters as the other ones so I 
don't see how this is possible. I attached my input files of the 
calculations. I hope someone can guide me through this.



Thanks in advance,

Laurens Siemons

Master student chemistry

University of Antwerp



___
users mailing list
users@lists.quantum-espresso.org
https://lists.quantum-espresso.org/mailman/listinfo/users


___
users mailing list
users@lists.quantum-espresso.org
https://lists.quantum-espresso.org/mailman/listinfo/users

Re: [Pw_forum] Total forces of relaxing metallic system decreases then increases gradually

2018-02-17 Thread Stefano de Gironcoli

is the energy decreasing ? if so the relaxation is not yet finished.

stefano


On 18/02/2018 02:55, Hind Almisbahi wrote:


Dear experts of quantum espresso,

I am doing relaxation and the problem is that the total force is 
decreasing until the 4^th scf, then start to increase gradually. The 
system does not converge. Actually, I need to calculate the band 
structure and see the band gap of Armchair graphene ribbon. The ribbon 
width is composed of 17 carbon atoms (N=3p+2 family), so it is 
metallic. I have one unit cell of the ribbon. I read in this mailing 
list many useful discussions about suitable input for metallic systems 
and I followed them as I understand. The version of quantum espresso 
is 6.1.


I tried many options of the input script but all the runs have the 
same problem, total force is decreasing until the 4^th scf then start 
to increase gradually. This is a complete version of my input:




calculation='relax',

outdir='.\Work',

prefix='Arm17',

pseudo_dir='C:\qe-6.1\pseudo',

verbosity='high',

etot_conv_thr= 1.0D-3,

forc_conv_thr=1.0d-2,

/



ibrav=8,

a=41.1089d0,

b=4.26258d0,

c=20.0d0,

nat=34,

ntyp=1,

ecutwfc=65,

ecutrho=520,

nbnd=76

occupations='smearing',

smearing='marzari-vanderbilt',

degauss=0.001d0,

/



conv_thr=1d-06,

mixing_beta=0.7d0,

mixing_mode='local-TF'

mixing_ndim=12

diagonalization='cg'

/



/

ATOMIC_SPECIES

C 12.010700d0 C.pbe-rrkjus.UPF

ATOMIC_POSITIONS {crystal}

C0.0336555229d00.666956d00.50d0

C0.0919486003d00.833913d00.50d0

C0.0336555229d00.333044d00.50d0

C0.1502416777d00.666956d00.50d0

C0.2085347551d00.833913d00.50d0

C0.0919486003d00.166087d00.50d0

C0.1502416777d00.333044d00.50d0

C0.2668277852d00.666956d00.50d0

C0.3251206731d00.833913d00.50d0

C0.2085347551d00.166087d00.50d0

C0.2668277852d00.333044d00.50d0

C0.3834141768d00.666956d00.50d0

C0.4417072068d00.833913d00.50d0

C0.3251206731d00.166087d00.50d0

C0.3834141768d00.333044d00.50d0

C0.502369d00.666956d00.50d0

C0.5582932669d00.833913d00.50d0

C0.4417072068d00.166087d00.50d0

C0.502369d00.333044d00.50d0

C0.6165862969d00.666956d00.50d0

C0.6748793269d00.833913d00.50d0

C0.5582932669d00.166087d00.50d0

C0.6165862969d00.333044d00.50d0

C0.7331723570d00.666956d00.50d0

C0.7914653870d00.833913d00.50d0

C0.6748793269d00.166087d00.50d0

C0.7331723570d00.333044d00.50d0

C0.8497584170d00.666956d00.50d0

C0.9080514470d00.833913d00.50d0

C0.7914653870d00.166087d00.50d0

C0.8497584170d00.333044d00.50d0

C0.9663444771d00.666956d00.50d0

C0.9080514470d00.166087d00.50d0

C0.9663444771d00.333044d00.50d0

K_POINTS {automatic}

150 1 0 0 0

These are some of the many options that I have explored:

The above potential is ultrasoft, I used also norm conserving 
(C.pbe-mt_gipaw.UPF).


Smearing: I used methfessel-paxton and marzari-vanderbilt

Degause: I used 0.01 and 0.001

mixing_beta: I used 0.1 and 0.7

mixing_mode: I used local-TF and TF

mixing_ndim: I used 8 and 12

diagonalization: I used cg and david

ecutwfc: I used 65 and 75

ecutrho: with norm conserving (ecutwfc*4 and ecutwfc*10), with 
ultrasoft (ecutwfc*8)


nbnd=76 (I have 34 atoms, so 136 valence electrons which need 68 
bands, because it is metallic I added 8 extra bands)



Previously I have successfully calculated the band structure and band 
gap of semiconductor ribbon but I could not do the calculation of 
metallic ribbon. I appreciate any help to solve my problem, why the 
forces always increase after the 4^th scf?



​

Best Regards,

Hind

Lecturer, IT department

King Abdulaziz University



Disclaimer: The information contained in this message is intended for 
the addressee only and may contain classified information. If you are 
not the addressee, please delete this message and notify the sender; 
you should not copy or distribute this message or disclose its 
contents to anyone. Any views or opinions expressed in this message 
are those of the individual(s) and not necessarily of the university. 
No reliance may be placed on this message without written confirmation 
from an authorized representative of its contents. No guarantee is 
implied that this message or any attachment is virus free or has not 
been intercepted and amended.


إخلاء مسؤليه:  ان جميع المعلومات المتضمنة في هذه الرسالة تخص المستلم، 
وربما تتضمن معلومات سرية، واذا لم تكن انت المستلم فالرجاء الغاء 
الرسالة واشعار المرسل، كما يمنع نسخ او توزيع هذه الرسالة او افشاء 
محتوياتها لاخرين. ويجدر بالعلم ان جميع الآراء ووجهات النظر الواردة في 
هذه الرسالة تعتبر شخصية وليست بالضرورة منسوبة للجامعة. كما لايمكن ان 
يعتمد على هذه الرسالة ما لم تحصل على 

Re: [Pw_forum] Help With Bandstructure Calculation

2017-09-21 Thread Stefano de Gironcoli

On 22/09/2017 01:07, Aaditya Hambarde wrote:


 namelist  no longer valid: please use  instead*


some time ago the  namelist has been renamed 

stefano
___
Pw_forum mailing list
Pw_forum@pwscf.org
http://pwscf.org/mailman/listinfo/pw_forum

Re: [Pw_forum] Error in routine bfgs (1): dE0s is positive which should never happen

2017-09-14 Thread Stefano de Gironcoli
Are you close to solution ? 
Does that comment seams to apply to your case ? You are the only one that can 
tell this.

stefano 
(sent from my phone)

> On 14 Sep 2017, at 19:52, Madhurya Chandel  wrote:
> 
> Respected user/admin, 
> 
> After running one input file for 8 days I have received on error
> Error in routine bfgs (1):
>  dE0s is positive which should never happen.
> Regarding this, on Forum someone has answered  that 
> 
> "This kind of errors invariably happens when you are very close to the 
> minimum and you have some numerical noise on forces. It is useless in my 
> opinion to insist: your system is sufficiently relaxed".
> Is there any solution or have to continue with the above mention statement 
> only?
> I have tried twice with same input file but getting same error. 
> The input file is 
> 
>title = CuF ,
>  calculation = 'vc-relax' ,
> restart_mode = 'from_scratch' ,
>   wf_collect = .false. ,
>   outdir = '/home/f2013877/QE/Madhurya/' ,
>   wfcdir = '/home/f2013877/QE/Madhurya/' ,
>   pseudo_dir = 
> '/home/f2013877/QE/pseudo/pslibrary.1.0.0/pbe/PSEUDOPOTENTIALS/' ,
>   prefix = 'pwscf' ,
>verbosity = 'low' ,
>nstep = 500 ,
>  /
>  
>ibrav = 2,
>celldm(1) = 15.8170076626d0,
>  nat = 3,
> ntyp = 3,
>  ecutwfc = 50 ,
>  ecutrho = 500 ,
> nbnd = 100,
>input_dft = pbe ,
>  occupations = 'smearing' ,
>  degauss = 0.005d0 ,
> smearing = 'methfessel-paxton' ,
>nspin = 2 ,
>starting_magnetization(1) = 0.1,
>starting_magnetization(2) = 0.5,
> vdw_corr = 'grimme-d2' ,
>  /
>  
> electron_maxstep = 800,
>scf_must_converge = .false. ,
> conv_thr = 1d-06 ,
>  mixing_mode = 'local-TF' ,
>  mixing_beta = 0.07d0 ,
>  diagonalization = 'cg' ,
>  /
>  
> ion_dynamics = 'bfgs' ,
>  /
>  
>cell_dynamics = 'bfgs' ,
>  cell_factor = 1.2D0 ,
>  /
> ATOMIC_SPECIES
>Cu   63.54600  Cu.pbe-dn-rrkjus_psl.1.0.0.UPF 
>Fe   55.84500  Fe.pbe-n-rrkjus_psl.1.0.0.UPF 
> O   15.99940  O.pbe-nl-rrkjus_psl.1.0.0.UPF 
> ATOMIC_POSITIONS crystal 
>Cu  0.00.00.0
>Fe  0.625000.625000.62500
> O  0.387000.387000.38700
> K_POINTS automatic 
>   8 8 8   0 0 0 
> 
> waiting for the reply. 
> Thanking you
> 
> With regards 
> Madhurya Chandel
> Research Scholar 
> Department of Chemistry 
> BITS PILANI, GOA campus
> +91-7507546773
> ___
> Pw_forum mailing list
> Pw_forum@pwscf.org
> http://pwscf.org/mailman/listinfo/pw_forum
___
Pw_forum mailing list
Pw_forum@pwscf.org
http://pwscf.org/mailman/listinfo/pw_forum

Re: [Pw_forum] Supercell size in phonon calculations

2017-09-06 Thread stefano de gironcoli

On 06/09/2017 11:22, Charlie Ruffman wrote:


Hi all,


I have recently become interested in how the size of the unit cell in 
phonon calculations effects the results produced by the calculation 
(Raman spectra in particular). It seems that increasing the number of 
atoms explicitly simulated should also increase the number of normal 
modes calcuated?



It does. these are hovewer modes that would correspond to a q/=0 in the 
original primitive cell... as such they should be raman-inactive.
The calculation in the supercell, if properly done (same cutoff, 
equivalent k-points, sufficient scf convergence) should be exactly 
comparable to the superposition of the results in the primitive cell for 
the q-vectors that map into the Gamma point of the supercell.


stefano

Does anyone know if using the primitive cell will produce more or less 
accurate results than a larger supercell?



Thanks


--
Charlie Ruffman | BSc
BSc(Hons) Student

Chemistry Department
University of Otago
Tel 64 3 479 7930
PO Box 56, Dunedin 9054
New Zealand

Website _http://neon.otago.ac.nz/research/alg/ 
_



___
Pw_forum mailing list
Pw_forum@pwscf.org
http://pwscf.org/mailman/listinfo/pw_forum



___
Pw_forum mailing list
Pw_forum@pwscf.org
http://pwscf.org/mailman/listinfo/pw_forum

Re: [Pw_forum] Require help in vc-relaxing of pure antimony

2017-08-17 Thread stefano de gironcoli

ecutwfc=50 ecutrho=1000 seriously ?
if ibrav=4 then celldm2 and celldm4 are not used... and luckily because 
celldm4 is the cosine of an angle NOT an angle in degrees

what is not converging is the first scf calculation not the vc-relaxation.
check your input.. possibly atoms are too close or you are computing too 
few bands.

or some other silly error.
stefano


On 16/08/2017 13:47, sayan chaudhuri wrote:

Dear QE Experts,

  I am running a variable cell relax code for pure Antimony. But even 
after 500 iteration the code is not converging. It will be very 
helpful if you kindly suggest something regarding the issue.

 I am attaching my input and output file herewith.

Thanking you,
Sayan Chaudhuri
Research Scholar
IIT Indore


___
Pw_forum mailing list
Pw_forum@pwscf.org
http://pwscf.org/mailman/listinfo/pw_forum



___
Pw_forum mailing list
Pw_forum@pwscf.org
http://pwscf.org/mailman/listinfo/pw_forum

Re: [Pw_forum] equation of state fitting

2017-08-16 Thread Stefano de Gironcoli


stefano 
(sent from my phone)

> On 16 Aug 2017, at 18:36, Sunetra Das  wrote:
> 
> Dear QE users,
> 
> I would like to know if there are any other equation of state that we can use 
> to fit the 'etot vs alat' graph to find out the equilibrium lattice constant. 
> I have been following the QE tutorials to learn the basics where it has been 
> demonstrated how to fit a etot vs alat graph using the four given equations 
> of state. But none of the obtained equilibrium bulk modulus values from those 
> equation fitting match the given experimental value, not even close. 
> How to improve the fitting of the equations or is there any other equation I 
> can use to fit the obtained data?
> 
> I am new to QE, any suggestion is highly appreciated. 
> 
> Thank you.
> 
> Regards,
> Sunetra Das
> Assistant Professor, 
> MMC College, Kolkata, WB,
> India.
> 
> ___
> Pw_forum mailing list
> Pw_forum@pwscf.org
> http://pwscf.org/mailman/listinfo/pw_forum


___
Pw_forum mailing list
Pw_forum@pwscf.org
http://pwscf.org/mailman/listinfo/pw_forum


Re: [Pw_forum] Help me to solve a problem in Charge Density Difference calculation using Quantum Espresso

2017-08-12 Thread stefano de gironcoli
you take the final vc-relaxed cell and assume the same for the two 
separate fragments.
And you need to redo the scf calculation (only the scf calculation) of 
the combined system so as to have the same parameters for all three.


stefano



On 12/08/2017 17:04, Bibhas Manna wrote:

Dear Matthieu,

Since, I have done the geometry relaxation using *vc-relax,* how can I 
maintain the same *Cell_Parameters* for all the three cases.


Thanks.
Bibhas


On Sat, Aug 12, 2017 at 8:22 PM, Matthieu Fortin-Deschênes 
<matthieu.fortin-desche...@polymtl.ca 
<mailto:matthieu.fortin-desche...@polymtl.ca>> wrote:


I'm no expert, but I think you need to make the scf calculations with
the same parameters, especially ecutrho and CELL_PARAMETERS.

Matthieu

Bibhas Manna <mannabib...@gmail.com
<mailto:mannabib...@gmail.com>> a écrit :

> Dear Sir,
>
> Thanking you for your quick reply.
>
> I have used the same parameter for three charges. Following are
the input
> pp.x files:
>
>
> *For Substrate + molecules:*
> prefix  = 'PG_4_HCHO_PBE'
> outdir = './tmp/'
> filplot = 'PG_4_HCHO_charge'
> plot_num= 0
>  /
>  
> nfile = 1
> filepp(1) = 'PG_4_HCHO_charge'
> weight(1) = 1.0
> iflag = 2
> output_format = 3
> fileout = 'PG4_HCHO.rho.dat'
> e1(1) =1.0, e1(2)=0, e1(3) = 0.0,
> e2(1) =0.0, e2(2)=0.0, e2(3) = 1.0,
> nx=56, ny=40
>  /
>
> *For Substrate only:*
> prefix  = 'PG_4_PBE'
> outdir = './tmp/'
> filplot = 'PG_4_charge'
> plot_num= 0
>  /
>  
> nfile = 1
> filepp(1) = 'PG_4_charge'
> weight(1) = 1.0
> iflag = 2
> output_format = 3
> fileout = 'PG4.rho.dat'
> e1(1) =1.0, e1(2)=0.0, e1(3) = 0.0,
> e2(1) =0.0, e2(2)=0.0, e2(3) = 1.0,
> nx=56, ny=40,
>  /
>
>
> *For Gas molecule only:*
> prefix  = 'HCHO_PBE'
> outdir = './tmp/'
> filplot = 'HCHO_charge'
> plot_num= 0
>  /
>  
> nfile = 1
> filepp(1) = 'HCHO_charge'
> weight(1) = 1.0
> iflag = 2
> output_format = 3
> fileout = 'HCHO.rho.dat'
> e1(1) =1.0, e1(2)=0.0, e1(3) = 0.0,
> e2(1) =0.0, e2(2)=0.0, e2(3) = 1.0,
>  nx=56, ny=40,
> /
>  I am actually getting  error in the reading of the filepp(2) as
follows:
>
> Reading header from file  PG_4_HCHO_charge
>
>Info: using nr1, nr2, nr3 values from input
>  Reading data from file  PG_4_HCHO_charge
>  Reading data from file  PG_4_charge
>
>
>

%%
>  Error in routine chdens (1):
>  incompatible gcutm or dual or ecut
>
>

%%
>
>  stopping ...
>
> Thanks.
> Bibhas
>
> On Sat, Aug 12, 2017 at 7:59 PM, stefano de gironcoli
<degir...@sissa.it <mailto:degir...@sissa.it>>
> wrote:
>
>> you need to compute the three charges with the same parameters
>>
>> stefano
>>
>>
>> On 12/08/2017 16:20, Bibhas Manna wrote:
>>
>> Dear All,
>>
>> I am very new to Quantum Espresso. I want to get a charge density
>> difference plot for a gas molecule adsorbed on the single layer
graphene
>> surface. Presently I am using pp.x to compute the same in QE v.5.1
>> following an equation:
>>
>> ∆ρ=ρAB – ρA-ρB
>>
>> Where, ρAB, ρA and ρB are the charge densities of graphene
-molecule
>> complex, graphene and molecule respectively. I have
successfully computed
>> all of these three charge densities using pp.x. Now, I am
trying to find
>> out the charge density difference using pp.x with nfile = 3,
while all of
>> these three charge files are available to me as inputs.
>>
>> Unfortunately, I got an error in the output log:
>> *Error in routine chdens (1):
>>incompatible gcutm or dual or ecut*
>>
>> Since, I have different number of atoms in each of the three
files, how
>> can I make the charge files having same number of atomic
coordinates?
>>
>> I am sharing my input and output files for your kind
considerations.
>>
&g

Re: [Pw_forum] Help me to solve a problem in Charge Density Difference calculation using Quantum Espresso

2017-08-12 Thread stefano de gironcoli
On 12/08/2017 16:52, Matthieu Fortin-Deschênes wrote:
> I'm no expert, but I think you need to make the scf calculations with
> the same parameters, especially ecutrho and CELL_PARAMETERS.
>
> Matthieu
exactly. stefano

>
> Bibhas Manna <mannabib...@gmail.com> a écrit :
>
>> Dear Sir,
>>
>> Thanking you for your quick reply.
>>
>> I have used the same parameter for three charges. Following are the input
>> pp.x files:
>>
>>
>> *For Substrate + molecules:*
>>  prefix  = 'PG_4_HCHO_PBE'
>>  outdir = './tmp/'
>>  filplot = 'PG_4_HCHO_charge'
>>  plot_num= 0
>>   /
>>   
>>  nfile = 1
>>  filepp(1) = 'PG_4_HCHO_charge'
>>  weight(1) = 1.0
>>  iflag = 2
>>  output_format = 3
>>  fileout = 'PG4_HCHO.rho.dat'
>>  e1(1) =1.0, e1(2)=0, e1(3) = 0.0,
>>  e2(1) =0.0, e2(2)=0.0, e2(3) = 1.0,
>>  nx=56, ny=40
>>   /
>>
>> *For Substrate only:*
>>  prefix  = 'PG_4_PBE'
>>  outdir = './tmp/'
>>  filplot = 'PG_4_charge'
>>  plot_num= 0
>>   /
>>   
>>  nfile = 1
>>  filepp(1) = 'PG_4_charge'
>>  weight(1) = 1.0
>>  iflag = 2
>>  output_format = 3
>>  fileout = 'PG4.rho.dat'
>>  e1(1) =1.0, e1(2)=0.0, e1(3) = 0.0,
>>  e2(1) =0.0, e2(2)=0.0, e2(3) = 1.0,
>>  nx=56, ny=40,
>>   /
>>
>>
>> *For Gas molecule only:*
>>  prefix  = 'HCHO_PBE'
>>  outdir = './tmp/'
>>  filplot = 'HCHO_charge'
>>  plot_num= 0
>>   /
>>   
>>  nfile = 1
>>  filepp(1) = 'HCHO_charge'
>>  weight(1) = 1.0
>>  iflag = 2
>>  output_format = 3
>>  fileout = 'HCHO.rho.dat'
>>  e1(1) =1.0, e1(2)=0.0, e1(3) = 0.0,
>>  e2(1) =0.0, e2(2)=0.0, e2(3) = 1.0,
>>   nx=56, ny=40,
>> /
>>   I am actually getting  error in the reading of the filepp(2) as follows:
>>
>> Reading header from file  PG_4_HCHO_charge
>>
>> Info: using nr1, nr2, nr3 values from input
>>   Reading data from file  PG_4_HCHO_charge
>>   Reading data from file  PG_4_charge
>>
>>
>> %%
>>   Error in routine chdens (1):
>>   incompatible gcutm or dual or ecut
>>
>> %%
>>
>>   stopping ...
>>
>> Thanks.
>> Bibhas
>>
>> On Sat, Aug 12, 2017 at 7:59 PM, stefano de gironcoli <degir...@sissa.it>
>> wrote:
>>
>>> you need to compute the three charges with the same parameters
>>>
>>> stefano
>>>
>>>
>>> On 12/08/2017 16:20, Bibhas Manna wrote:
>>>
>>> Dear All,
>>>
>>> I am very new to Quantum Espresso. I want to get a charge density
>>> difference plot for a gas molecule adsorbed on the single layer graphene
>>> surface. Presently I am using pp.x to compute the same in QE v.5.1
>>> following an equation:
>>>
>>>  ∆ρ=ρAB – ρA-ρB
>>>
>>> Where, ρAB, ρA and ρB are the charge densities of graphene -molecule
>>> complex, graphene and molecule respectively. I have successfully computed
>>> all of these three charge densities using pp.x. Now, I am trying to find
>>> out the charge density difference using pp.x with nfile = 3, while all of
>>> these three charge files are available to me as inputs.
>>>
>>> Unfortunately, I got an error in the output log:
>>> *Error in routine chdens (1):
>>> incompatible gcutm or dual or ecut*
>>>
>>> Since, I have different number of atoms in each of the three files, how
>>> can I make the charge files having same number of atomic coordinates?
>>>
>>> I am sharing my input and output files for your kind considerations.
>>>
>>> Can you help me to solve this problem?
>>>
>>> Thanks with regards,
>>> Bibhas Manna
>>> Research Scholar,
>>> IIT Kharagpur, India
>>>
>>>
>>>
>>> ___
>>> Pw_forum mailing
>>> listPw_forum@pwscf.orghttp://pwscf.org/mailman/listinfo/pw_forum
>>>
>>>
>>>
>>> ___
>>> Pw_forum mailing list
>>> Pw_forum@pwscf.org
>>> http://pwscf.org/mailman/listinfo/pw_forum
>>>
>
>
> ___
> Pw_forum mailing list
> Pw_forum@pwscf.org
> http://pwscf.org/mailman/listinfo/pw_forum


___
Pw_forum mailing list
Pw_forum@pwscf.org
http://pwscf.org/mailman/listinfo/pw_forum

Re: [Pw_forum] Help me to solve a problem in Charge Density Difference calculation using Quantum Espresso

2017-08-12 Thread stefano de gironcoli

you need to compute the three charges with the same parameters

stefano

On 12/08/2017 16:20, Bibhas Manna wrote:

Dear All,

I am very new to Quantum Espresso. I want to get a charge density 
difference plot for a gas molecule adsorbed on the single layer 
graphene surface. Presently I am using pp.x to compute the same in QE 
v.5.1 following an equation:


∆ρ=ρAB – ρA-ρB

Where, ρAB, ρA and ρB are the charge densities of graphene -molecule 
complex, graphene and molecule respectively. I have successfully 
computed all of these three charge densities using pp.x. Now, I am 
trying to find out the charge density difference using pp.x with nfile 
= 3, while all of these three charge files are available to me as inputs.


Unfortunately, I got an error in the output log: *Error in routine 
chdens (1):

 incompatible gcutm or dual or ecut*

Since, I have different number of atoms in each of the three files, 
how can I make the charge files having same number of atomic coordinates?


I am sharing my input and output files for your kind considerations.

Can you help me to solve this problem?

Thanks with regards,
Bibhas Manna
Research Scholar,
IIT Kharagpur, India



___
Pw_forum mailing list
Pw_forum@pwscf.org
http://pwscf.org/mailman/listinfo/pw_forum



___
Pw_forum mailing list
Pw_forum@pwscf.org
http://pwscf.org/mailman/listinfo/pw_forum

Re: [Pw_forum] Open shell system calculations

2017-08-10 Thread stefano de gironcoli
* Martyna-Tuckerman option for assume_isolated variable in the  
namelist.


stefano


On 10/08/2017 09:00, stefano de gironcoli wrote:
I think spin unpolarized calculations in that case miss a large 
exchange correlation contribution coming from spin alignment.
Isolated Mn2+ should have total magnetization equal to 5 (i think). 
You can set this from input. Or you can just let the code find the 
minimum by itself.


you can use martina-tuckerman option to decouple periodic images.
It's very important for charged systems.
That should reduce the size of supercell needed to have stable numbers.

stefano


On 10/08/2017 08:50, Iryna Zaporozhets wrote:
I have a single Mn2+  ion in the isolated system with vacuum between 
periodic images. Spin-unpolarized calculations run without any errors 
so I wonder whether spin-unpolarized calculations produce any useful 
results.


Best, Iryna Zaporozhets

2017-08-09 23:39 GMT-07:00 stefano de gironcoli <degir...@sissa.it 
<mailto:degir...@sissa.it>>:


depends on your system, is it magnetic ?
metallic aluminum or copper have spin compensated densities and
you can use nspin=1.
nickel or iron are magnetic and nspin=2 is needed.
stefano


On 10/08/2017 08:29, Iryna Zaporozhets wrote:

Dear PW community,

I am working with an open-shell system which requires a lot of
computational efforts. Is it possible to use other options than
spin-polarized calculations (nspin = 2) to reduce computational
cost, for example, restricted open-shell  DFT?  Is there any
physical meaning to the results obtained with
spin-unpolarized (nspin = 1) calculations in case of the
open-shell system?

Thank you,
Iryna Zaporozhets,
Master student in Chemistry
Kharkiv National University, Ukraine


___
Pw_forum mailing list
Pw_forum@pwscf.org <mailto:Pw_forum@pwscf.org>
http://pwscf.org/mailman/listinfo/pw_forum
<http://pwscf.org/mailman/listinfo/pw_forum>




___
Pw_forum mailing list
Pw_forum@pwscf.org <mailto:Pw_forum@pwscf.org>
http://pwscf.org/mailman/listinfo/pw_forum
<http://pwscf.org/mailman/listinfo/pw_forum>




___
Pw_forum mailing list
Pw_forum@pwscf.org
http://pwscf.org/mailman/listinfo/pw_forum





___
Pw_forum mailing list
Pw_forum@pwscf.org
http://pwscf.org/mailman/listinfo/pw_forum



___
Pw_forum mailing list
Pw_forum@pwscf.org
http://pwscf.org/mailman/listinfo/pw_forum

Re: [Pw_forum] Open shell system calculations

2017-08-10 Thread stefano de gironcoli
I think spin unpolarized calculations in that case miss a large exchange 
correlation contribution coming from spin alignment.
Isolated Mn2+ should have total magnetization equal to 5 (i think). You 
can set this from input. Or you can just let the code find the minimum 
by itself.


you can use martina-tuckerman option to decouple periodic images.
It's very important for charged systems.
That should reduce the size of supercell needed to have stable numbers.

stefano


On 10/08/2017 08:50, Iryna Zaporozhets wrote:
I have a single Mn2+  ion in the isolated system with vacuum between 
periodic images. Spin-unpolarized calculations run without any errors 
so I wonder whether spin-unpolarized calculations produce any useful 
results.


Best, Iryna Zaporozhets

2017-08-09 23:39 GMT-07:00 stefano de gironcoli <degir...@sissa.it 
<mailto:degir...@sissa.it>>:


depends on your system, is it magnetic ?
metallic aluminum or copper have spin compensated densities and
you can use nspin=1.
nickel or iron are magnetic and nspin=2 is needed.
stefano


On 10/08/2017 08:29, Iryna Zaporozhets wrote:

Dear PW community,

I am working with an open-shell system which requires a lot of
computational efforts. Is it possible to use other options than
spin-polarized calculations (nspin = 2) to reduce computational
cost, for example, restricted open-shell  DFT?  Is there any
physical meaning to the results obtained with
spin-unpolarized (nspin = 1) calculations in case of the
open-shell system?

Thank you,
Iryna Zaporozhets,
Master student in Chemistry
Kharkiv National University, Ukraine


___
Pw_forum mailing list
Pw_forum@pwscf.org <mailto:Pw_forum@pwscf.org>
http://pwscf.org/mailman/listinfo/pw_forum
<http://pwscf.org/mailman/listinfo/pw_forum>




___
Pw_forum mailing list
Pw_forum@pwscf.org <mailto:Pw_forum@pwscf.org>
http://pwscf.org/mailman/listinfo/pw_forum
<http://pwscf.org/mailman/listinfo/pw_forum>




___
Pw_forum mailing list
Pw_forum@pwscf.org
http://pwscf.org/mailman/listinfo/pw_forum



___
Pw_forum mailing list
Pw_forum@pwscf.org
http://pwscf.org/mailman/listinfo/pw_forum

Re: [Pw_forum] Open shell system calculations

2017-08-10 Thread stefano de gironcoli

depends on your system, is it magnetic ?
metallic aluminum or copper have spin compensated densities and you can 
use nspin=1.

nickel or iron are magnetic and nspin=2 is needed.
stefano

On 10/08/2017 08:29, Iryna Zaporozhets wrote:

Dear PW community,

I am working with an open-shell system which requires a lot of 
computational efforts. Is it possible to use other options than 
spin-polarized calculations (nspin = 2) to reduce computational cost, 
for example, restricted open-shell  DFT? Is there any physical meaning 
to the results obtained with spin-unpolarized (nspin = 1) calculations 
in case of the open-shell system?


Thank you,
Iryna Zaporozhets,
Master student in Chemistry
Kharkiv National University, Ukraine


___
Pw_forum mailing list
Pw_forum@pwscf.org
http://pwscf.org/mailman/listinfo/pw_forum



___
Pw_forum mailing list
Pw_forum@pwscf.org
http://pwscf.org/mailman/listinfo/pw_forum

Re: [Pw_forum] vc-relax convergence

2017-08-06 Thread stefano de gironcoli

yes
start from the perfect relaxed geometry (as obtained from vc-relax)
remove an atom and relax the internal structure.
at the end also perform a vc-relaxation (from the fixed-cell relaxed 
geometry)

stefano

On 06/08/2017 17:33, Manu Hegde wrote:

Hello Stephano,
Thanks for your reply. Just to double check what you meant before 
starting the calculations, you mean take the equilibrium values from 
the vc-relax calculation (perfect cell,  only co-ordinates) and do the 
'relax' calculation with defect?.

Manu
(University of Waterloo).


On Sun, Aug 6, 2017 at 11:15 AM, stefano de gironcoli 
<degir...@sissa.it <mailto:degir...@sissa.it>> wrote:


does the energy (enthalpy) decreases as the vc-relaxation
progresses ?
The local structure around the defect may be quite different. It
may just take long.

I would start by performing a fixed cell relaxation with the cell
parameters fixed at the undefected equilibrium values. And only
after that I would start the vc-relax calculation from the so
relaxed configuration to evaluate the vacancy formation volume.
The local arrangement around the defect should be already more or
less ok (if the supercell is large enough).

HTH
stefano



On 06/08/2017 16:40, Manu Hegde wrote:

Hello,

I am trying to optimize anatase TiO2 supercell with 48 atoms. The
supercell converges in 15 BFGS cycles for a perfect cell. But
when I introduce defect (by removing one of the oxygen atom), the
vc-relax is not t all converging. I am using the same parameters
in both the cases. Can anyone give some feedback on this.?.  Here
is my input,



 calculation = 'vc-relax' ,
restart_mode = 'from_scratch' ,
outdir = '/work/mhegde/tmp_tiodef/',
pseudo_dir = '/home/mhegde/tio_project/def_tio/',
prefix = 'tioan',
 verbosity = 'high',

 /
 
 ibrav = 0,
 nat = 47,
  ntyp = 2,
 ecutwfc =40,
 ecutrho=400,
 occupations='smearing',
degauss=0.003

 /
 
conv_thr = 1.D-6,
mixing_beta=0.1
/

ion_dynamics='bfgs'

/

cell_dynamics='bfgs'

/
ATOMIC_SPECIES
   Ti  47.867000  Ti.pw91-nsp-van.UPF
   O   15.99900 O.pw91-van_ak.UPF

ATOMIC_POSITIONS angstrom
Ti 0.   2.84182500 1.2097
Ti 1.89455000   2.84182500 3.6291
O  0.   0.94727500 1.99939216
O  0.   2.84182500 4.41879216
O  1.89455000   0.94727500 2.83940784
Ti  -1.89455000   0.94727500  -3.6291
Ti 0.   0.94727500  -1.2097
O -1.89455000  -0.94727500  -2.83940784
O -1.89455000   0.94727500  -0.42000784
O  0.  -0.94727500  -1.99939216
O  0.   0.94727500  -4.41879216
Ti 0.  -0.94727500 1.2097
Ti 1.89455000  -0.94727500 3.6291
O  0.  -2.84182500 1.99939216
O  0.  -0.94727500 4.41879216
O  1.89455000  -2.84182500 2.83940784
O  1.89455000  -0.94727500 0.42000784
Ti  -1.89455000  -2.84182500  -3.6291
Ti 0.  -2.84182500  -1.2097
O -1.89455000   2.84182500  -2.83940784
O -1.89455000  -2.84182500  -0.42000784
O  0.   2.84182500  -1.99939216
O  0.  -2.84182500  -4.41879216
Ti  -3.7891   2.84182500 1.2097
Ti  -1.89455000   2.84182500 3.6291
O -3.7891   0.94727500 1.99939216
O -3.7891   2.84182500 4.41879216
O -1.89455000   0.94727500 2.83940784
O -1.89455000   2.84182500 0.42000784
Ti 1.89455000   0.94727500  -3.6291
Ti  -3.7891   0.94727500  -1.2097
O  1.89455000  -0.94727500  -2.83940784
O  1.89455000   0.94727500  -0.42000784
O -3.7891  -0.94727500  -1.99939216
O -3.7891   0.94727500  -4.41879216
Ti  -3.7891  -0.94727500 1.2097
Ti  -1.89455000  -0.94727500 3.6291
O -3.7891  -2.84182500 1.99939216
O -3.7891  -0.94727500 4.41879216
O -1.89455000  -2.84182500 2.83940784
O -1.89455000  -0.94727500 0.42000784
Ti 1.89455000  -2.84182500  -3.6291
Ti  -3.7891  -2.84182500  -1.2097
O  1.89455000   2.84182500  -2.83940784
O  1.89455000  -2.84182500  -0.42000784
O -3.7891   2.84182500  -1.99939216
O -3.7891  -2.84182500  -4.41879216

K_POINTS {automatic}

 2 2 2   0 0 0

CELL_PARAMETERS angstrom

 7.5782   0.   0.
 0.   7.5782   0.
 0.   0.   9.6776

Thanks,
Manu
(University of Waterloo)






___
Pw_forum mailing list
Pw_forum@pwscf.org <mailto:Pw_forum@pwscf.org>
http://pwscf.org/mailman/listinfo/pw_forum
<htt

Re: [Pw_forum] vc-relax convergence

2017-08-06 Thread stefano de gironcoli

does the energy (enthalpy) decreases as the vc-relaxation progresses ?
The local structure around the defect may be quite different. It may 
just take long.


I would start by performing a fixed cell relaxation with the cell 
parameters fixed at the undefected equilibrium values. And only after 
that I would start the vc-relax calculation from the so relaxed 
configuration to evaluate the vacancy formation volume. The local 
arrangement around the defect should be already more or less ok (if the 
supercell is large enough).


HTH
stefano


On 06/08/2017 16:40, Manu Hegde wrote:

Hello,

I am trying to optimize anatase TiO2 supercell with 48 atoms. The 
supercell converges in 15 BFGS cycles for a perfect cell. But when I 
introduce defect (by removing one of the oxygen atom), the vc-relax is 
not t all converging. I am using the same parameters in both the 
cases. Can anyone give some feedback on this.?.  Here is my input,




 calculation = 'vc-relax' ,
restart_mode = 'from_scratch' ,
  outdir = '/work/mhegde/tmp_tiodef/',
  pseudo_dir = '/home/mhegde/tio_project/def_tio/',
  prefix = 'tioan',
   verbosity = 'high',

 /
 
   ibrav = 0,
   nat = 47,
ntyp = 2,
 ecutwfc =40,
 ecutrho=400,
 occupations='smearing',
  degauss=0.003

 /
 
conv_thr = 1.D-6,
mixing_beta=0.1
/

ion_dynamics='bfgs'

/

cell_dynamics='bfgs'

/
ATOMIC_SPECIES
   Ti  47.867000  Ti.pw91-nsp-van.UPF
   O   15.99900   O.pw91-van_ak.UPF

ATOMIC_POSITIONS angstrom
Ti   0. 2.84182500   1.2097
Ti   1.89455000 2.84182500   3.6291
O0. 0.94727500   1.99939216
O0. 2.84182500   4.41879216
O1.89455000 0.94727500   2.83940784
Ti  -1.89455000 0.94727500  -3.6291
Ti   0. 0.94727500  -1.2097
O   -1.89455000  -0.94727500  -2.83940784
O   -1.89455000 0.94727500  -0.42000784
O0.  -0.94727500  -1.99939216
O0. 0.94727500  -4.41879216
Ti   0.  -0.94727500   1.2097
Ti   1.89455000  -0.94727500   3.6291
O0.  -2.84182500   1.99939216
O0.  -0.94727500   4.41879216
O1.89455000  -2.84182500   2.83940784
O1.89455000  -0.94727500   0.42000784
Ti  -1.89455000  -2.84182500  -3.6291
Ti   0.  -2.84182500  -1.2097
O   -1.89455000 2.84182500  -2.83940784
O   -1.89455000  -2.84182500  -0.42000784
O0. 2.84182500  -1.99939216
O0.  -2.84182500  -4.41879216
Ti  -3.7891 2.84182500   1.2097
Ti  -1.89455000 2.84182500   3.6291
O   -3.7891 0.94727500   1.99939216
O   -3.7891 2.84182500   4.41879216
O   -1.89455000 0.94727500   2.83940784
O   -1.89455000 2.84182500   0.42000784
Ti   1.89455000 0.94727500  -3.6291
Ti  -3.7891 0.94727500  -1.2097
O1.89455000  -0.94727500  -2.83940784
O1.89455000 0.94727500  -0.42000784
O   -3.7891  -0.94727500  -1.99939216
O   -3.7891 0.94727500  -4.41879216
Ti  -3.7891  -0.94727500   1.2097
Ti  -1.89455000  -0.94727500   3.6291
O   -3.7891  -2.84182500   1.99939216
O   -3.7891  -0.94727500   4.41879216
O   -1.89455000  -2.84182500   2.83940784
O   -1.89455000  -0.94727500   0.42000784
Ti   1.89455000  -2.84182500  -3.6291
Ti  -3.7891  -2.84182500  -1.2097
O1.89455000 2.84182500  -2.83940784
O1.89455000  -2.84182500  -0.42000784
O   -3.7891 2.84182500  -1.99939216
O   -3.7891  -2.84182500  -4.41879216

K_POINTS {automatic}

 2 2 2   0 0 0

CELL_PARAMETERS angstrom

 7.5782   0.   0.
 0.   7.5782   0.
 0.   0.   9.6776

Thanks,
Manu
(University of Waterloo)






___
Pw_forum mailing list
Pw_forum@pwscf.org
http://pwscf.org/mailman/listinfo/pw_forum



___
Pw_forum mailing list
Pw_forum@pwscf.org
http://pwscf.org/mailman/listinfo/pw_forum

Re: [Pw_forum] GPU Version

2017-08-03 Thread stefano de gironcoli

Dear Kopinjol Baishya,

there is an ongoing effort, in collaboration with NVIDIA, to enable QE 
on GPU using cuda fortran.


  A post on pw_forum appeared some months ago.

  The project is hosted here https://github.com/RSE-Cambridge/qe-gpu
  For the most recent pre-production tagged version, v0.3, see 
https://github.com/RSE-Cambridge/qe-gpu/releases/tag/v0.3


  Please refer to README.md for additional information

  Notice that the cuda version developed by Filippo Spiga is no longer 
maintained.
  The last release, aligned to QE-5.4.0 version, can still be 
downloaded from the QE download page 
http://www.qe-forge.org/gf/project/q-e/frs/?action=FrsReleaseBrowse_package_id=18


best regards,

Stefano

Kopinjol Baishya wrote:

Hello,

How do I get the GPU enabled version of Quantum Espresso?

Thank you,
Kopinjol Baishya


___
Pw_forum mailing list
Pw_forum@pwscf.org
http://pwscf.org/mailman/listinfo/pw_forum



___
Pw_forum mailing list
Pw_forum@pwscf.org
http://pwscf.org/mailman/listinfo/pw_forum

Re: [Pw_forum] Magnetic moment of Hydrogen

2017-07-22 Thread stefano de gironcoli

dear Rajesh,

nspin =2 (meaning two independent spin components in the density)

metallic calculation or rather define the total magnetization.
nband > 1. 2-3 should be ok. default values are usually fine.

starting_magnetization(1)=anything different from zero, in this case 
0.7-0.9 should be ok, even 1.0.


An Hydrogen isolated atom may have some issues due to the fact that one 
of the two spin channels should be completely empty and I'm not sure how 
the code deal with zero density.


maybe you can do calculations for total_magnetization 0.7, 0.8, 0.9, 
0.95, 0.99 and see where it goes.


stefano

On 23/07/2017 05:40, Rajesh wrote:

Dear users
I am very new to quantum expresso. I want to calculate energy of a 
hydrogen atom. For that I need nspin, nbnd and starting magnetisation. 
From where I can get these values?


Thank you.

Rajesh


___
Pw_forum mailing list
Pw_forum@pwscf.org
http://pwscf.org/mailman/listinfo/pw_forum



___
Pw_forum mailing list
Pw_forum@pwscf.org
http://pwscf.org/mailman/listinfo/pw_forum

Re: [Pw_forum] Periodic Boundary Condition in PW

2017-07-22 Thread stefano de gironcoli

from the input description that you can find at
http://www.quantum-espresso.org/wp-content/uploads/Doc/INPUT_PW.html
i would say bc1 is probably what you want.
stefano

esm_bc CHARACTER
Default: 'pbc'
See: assume_isolated
If assume_isolated = 'esm', determines the boundary
conditions used for either side of the slab.

Currently available choices:
'pbc' :
 (default): regular periodic calculation (no ESM).
'bc1' :
 Vacuum-slab-vacuum (open boundary conditions).
'bc2' :
Metal-slab-metal (dual electrode configuration).
See also esm_efield.
'bc3' :
Vacuum-slab-metal


On 22/07/2017 10:51, Rajesh wrote:

Dear Stefano,

Thank you for clarification. What boundary conditions are good for 2D 
materials? pbd or bc1?


Thank you.

Rajesh

On Sat, Jul 22, 2017 at 2:16 PM, stefano de gironcoli 
<degir...@sissa.it <mailto:degir...@sissa.it>> wrote:


Dear Rajesh,

quantum espresso is a periodic code. PBC are the default.
you'll find options to remove (or account for) interaction with
periodic replicas.

stefano



On 22/07/2017 10:34, Rajesh wrote:

Dear users

How can I employ periodic boundary conditions in quamtum espresso?
I dont find any option in pwgui for pbc.

Thank you.

Rajesh


___
Pw_forum mailing list
Pw_forum@pwscf.org <mailto:Pw_forum@pwscf.org>
http://pwscf.org/mailman/listinfo/pw_forum
<http://pwscf.org/mailman/listinfo/pw_forum>




___
Pw_forum mailing list
Pw_forum@pwscf.org <mailto:Pw_forum@pwscf.org>
http://pwscf.org/mailman/listinfo/pw_forum
<http://pwscf.org/mailman/listinfo/pw_forum>




___
Pw_forum mailing list
Pw_forum@pwscf.org
http://pwscf.org/mailman/listinfo/pw_forum



___
Pw_forum mailing list
Pw_forum@pwscf.org
http://pwscf.org/mailman/listinfo/pw_forum

Re: [Pw_forum] Periodic Boundary Condition in PW

2017-07-22 Thread stefano de gironcoli

Dear  Rajesh,

quantum espresso is a periodic code. PBC are the default.
you'll find options to remove (or account for) interaction with periodic 
replicas.


stefano


On 22/07/2017 10:34, Rajesh wrote:

Dear users

How can I employ periodic boundary conditions in quamtum espresso?
I dont find any option in pwgui for pbc.

Thank you.

Rajesh


___
Pw_forum mailing list
Pw_forum@pwscf.org
http://pwscf.org/mailman/listinfo/pw_forum



___
Pw_forum mailing list
Pw_forum@pwscf.org
http://pwscf.org/mailman/listinfo/pw_forum

Re: [Pw_forum] computation time of addusdens subroutine

2017-07-11 Thread stefano de gironcoli
You can try to see if the postprocess code accepts the calculation of 
augmentation charge in real space (tqr=.true. in some namelist , I guess 
electrons)
If it works it should be faster
stefano

On 11/07/2017 16:04, Junfeng Qiao wrote:
> Dear QE users,
>
> I am currently trying to extract wavefunction |psi|^2 of some bands and
> kpoints, which can be achieved with plot_num = 7 of pp.x.
>
> However, if I extract large amount of wavefunctions, for example, assigning
> kpoint(1) = 1, kpoint(2) = 20*20, kband(1) = 1, kband(2) = 300,
> the pp.x will run for an unbearable long time.
>
> On my cluster (10 intel cpus, total 60 cores), each wavefunction will take
> about 8 seconds, and do a simple prediction, in total 8 * (20*20) * 300 / 3600
> / 24 = 11 days ! It is much more longer than the scf calculations.
>
> After playing with the code(QE v6.1), I found in local_dos.f90 line 392, the
> call to addusdens() takes up almost all the computation time, other parts are
> negligible.
>
> While, in this situation, I totally have no ideas how to get out of this
> trouble, so I am seeking for help in this community: is it possible to speed
> up this addusdens() part?
>
> Thanks for your time of reading this email, any suggestions will be truly
> appreciated.
>
> Sincerely,
> Junfeng Qiao
> Beihang University (BUAA)
> Beijing, China
> ___
> Pw_forum mailing list
> Pw_forum@pwscf.org
> http://pwscf.org/mailman/listinfo/pw_forum


___
Pw_forum mailing list
Pw_forum@pwscf.org
http://pwscf.org/mailman/listinfo/pw_forum


Re: [Pw_forum] Confusion about silicon phonon dispersion along K-X path

2017-07-09 Thread stefano de gironcoli

the confusion comes from the fact that in the picture you linked
U and K are defined as different points but they are actually the same 
point.

stefano

On 09/07/2017 17:42, stefano de gironcoli wrote:

dear balabi,
if you take the sigma line, (xi,xi,0), when xi=0.75 you are at the K point
if you keep going along (xi,xi,0) until xi=1.0 you get to (1,1,0) 
which is the same as

X=(0,0,1) because (1,1,-1) is a G vector for the fcc
HTH

stefano


On 09/07/2017 17:23, balabi wrote:

Dear nicola,
Thank you so much for your reply. But I don't understand.
According to wikipedia's picture, the only Γ - K - X path that I 
can see is like this

https://pasteboard.co/GA9vm6Q.png
I can not see any other possibility. Would you please show me 
what is the other possible path for  Γ - K - X in a picture? Thank 
you so much.
And I do the phonon dispersion calculation using q_in_band_form 
for matdyn.x, k path labels are default. I thought this is standard.


best regards



___
Pw_forum mailing list
Pw_forum@pwscf.org
http://pwscf.org/mailman/listinfo/pw_forum





___
Pw_forum mailing list
Pw_forum@pwscf.org
http://pwscf.org/mailman/listinfo/pw_forum



___
Pw_forum mailing list
Pw_forum@pwscf.org
http://pwscf.org/mailman/listinfo/pw_forum

Re: [Pw_forum] Confusion about silicon phonon dispersion along K-X path

2017-07-09 Thread stefano de gironcoli

dear balabi,
if you take the sigma line, (xi,xi,0), when xi=0.75 you are at the K point
if you keep going along (xi,xi,0) until xi=1.0 you get to (1,1,0) which 
is the same as

X=(0,0,1) because (1,1,-1) is a G vector for the fcc
HTH

stefano


On 09/07/2017 17:23, balabi wrote:

Dear nicola,
Thank you so much for your reply. But I don't understand.
According to wikipedia's picture, the only Γ - K - X path that I 
can see is like this

https://pasteboard.co/GA9vm6Q.png
I can not see any other possibility. Would you please show me what 
is the other possible path for  Γ - K - X in a picture? Thank you so much.
And I do the phonon dispersion calculation using q_in_band_form 
for matdyn.x, k path labels are default. I thought this is standard.


best regards



___
Pw_forum mailing list
Pw_forum@pwscf.org
http://pwscf.org/mailman/listinfo/pw_forum



___
Pw_forum mailing list
Pw_forum@pwscf.org
http://pwscf.org/mailman/listinfo/pw_forum

Re: [Pw_forum] Dielectric constant at RPA level with PHONON

2017-06-29 Thread stefano de gironcoli
because somewhere in phq_readin
trans = trans .OR. ldisp
lrpa is only valid for the calculation of the macroscopic dielectric 
constant, not for the vibrational parts.

stefano

On 30/06/2017 01:06, Vahid Askarpour wrote:
> Dear QE Users,
>
> After a self consistent calculation, I used the following input to the ph.x 
> code (qe-6.1) to calculate the dielectric constant at the RPA level using the 
> lrpa tag.
>
> SnSe
> 
>tr2_ph   =  1.0d-16
>prefix   = 'SnSe'
>amass(1) = 118.71
>amass(2) = 78.96
>ldisp = .true.
>epsil=.true.
>trans = .false.
>lrpa = .true.
>qplot = .true.
>outdir   = './'
>alpha_mix(1)=0.3
>fildyn   = 'SnSe.dyn'
>fildvscf = 'dvscf'
>   /
>   48
> 0.000   0.000   0.000   0.0138889
> 0.000   0.000   0.167   0.028
> 0.000   0.000   0.333   0.028
> 0.000   0.000  -0.500   0.0138889
> 0.000   0.167   0.000   0.028
> 0.000   0.167   0.167   0.056
> 0.000   0.167   0.333   0.056
> 0.000   0.167  -0.500   0.028
> 0.000   0.333   0.000   0.028
> 0.000   0.333   0.167   0.056
> 0.000   0.333   0.333   0.056
> 0.000   0.333  -0.500   0.028
> 0.000  -0.500   0.000   0.0138889
> 0.000  -0.500   0.167   0.028
> 0.000  -0.500   0.333   0.028
> 0.000  -0.500  -0.500   0.0138889
> 0.250   0.000   0.000   0.028
> 0.250   0.000   0.167   0.056
> 0.250   0.000   0.333   0.056
> 0.250   0.000  -0.500   0.028
> 0.250   0.167   0.000   0.056
> 0.250   0.167   0.167   0.111
> 0.250   0.167   0.333   0.111
> 0.250   0.167  -0.500   0.056
> 0.250   0.333   0.000   0.056
> 0.250   0.333   0.167   0.111
> 0.250   0.333   0.333   0.111
> 0.250   0.333  -0.500   0.056
> 0.250  -0.500   0.000   0.028
> 0.250  -0.500   0.167   0.056
> 0.250  -0.500   0.333   0.056
> 0.250  -0.500  -0.500   0.028
>-0.500   0.000   0.000   0.0138889
>-0.500   0.000   0.167   0.028
>-0.500   0.000   0.333   0.028
>-0.500   0.000  -0.500   0.0138889
>-0.500   0.167   0.000   0.028
>-0.500   0.167   0.167   0.056
>-0.500   0.167   0.333   0.056
>-0.500   0.167  -0.500   0.028
>-0.500   0.333   0.000   0.028
>-0.500   0.333   0.167   0.056
>-0.500   0.333   0.333   0.056
>-0.500   0.333  -0.500   0.028
>-0.500  -0.500   0.000   0.0138889
>-0.500  -0.500   0.167   0.028
>-0.500  -0.500   0.333   0.028
>-0.500  -0.500  -0.500   0.0138889
>
> I get the following error:
>
>   
> %
>
>   task # 6  from phq_readin : error # 1
>   only dielectric constant with lrpa or lnoloc
>   
> %
>
> I checked the phq_redin.f90 code and the error arises from the following line:
>
>IF (trans.AND.(lrpa.OR.lnoloc)) CALL errore('phq_readin', &
>  'only dielectric constant with lrpa or lnoloc',1)
>
> Since I have set trans=.false., why am I getting this error? There must be 
> something wrong with the input but no such error was reported to the Forum.
>
> Any suggestions would be appreciated.
>
> Thank you,
>
> Vahid
>
> Vahid Askarpour
> Department of Physics and Atmospheric Science
> Dalhousie University,
> Halifax, NS, Canada
> ___
> Pw_forum mailing list
> Pw_forum@pwscf.org
> http://pwscf.org/mailman/listinfo/pw_forum


___
Pw_forum mailing list
Pw_forum@pwscf.org
http://pwscf.org/mailman/listinfo/pw_forum


Re: [Pw_forum] vc-relax cell parameters are not changing

2017-06-23 Thread stefano de gironcoli

defining A=1 should be the same as defining celldm(1)=1.889...=1/0.529177

stefano

On 23/06/2017 08:30, Isaiah Moses wrote:

Hi Paolo,
But how does one set CELL_PARAMETERS (angstrom)?
Because I observe that the output I obtain from vc-relax is 
automatically set to

CELL_PARAMETERS (alat=  8.1140).
Does that has to do with the unit of celldm?

I must be missing something on theses units' usage.

I appreciate your usual prompt response,
Isaiah

On Fri, Jun 23, 2017 at 7:16 AM, Paolo Giannozzi 
<p.gianno...@gmail.com <mailto:p.gianno...@gmail.com>> wrote:


And, by the way, it is a bad habit to set the lattice parameter to the
conversion factor between a.u. and A. One should use instead
CELL_PARAMETERS (angstrom)
to specify lattice vectors in A

Paolo

On Fri, Jun 23, 2017 at 4:55 AM, Stefano de Gironcoli
<degir...@sissa.it <mailto:degir...@sissa.it>> wrote:
> The lattice parameter does not change, it is not updated along
vc-relax, the
> lattice vectors do.
> Using bfgs you don't need to define dt
>
> stefano
> (sent from my phone)
>
> On 23 Jun 2017, at 01:55, Brendan Smith <mrbsmit...@gmail.com
<mailto:mrbsmit...@gmail.com>> wrote:
>
> Hello all,
>
> I am doing a the following vc-relax calculation and the unit
cell paramters
> are not changing. I have looked at other posts which descirbes
similar
> problems, but it does not remedy my problem. Below is my input.
> Best,
> Brendan
>
>
> 
>   calculation = 'vc-relax',
>   dt = 20.67055,
>   nstep = 1000,
>   pseudo_dir = './'
>   outdir = './',
>   prefix = 'x',
>   disk_io = 'low',
> /
>
> 
>   ibrav = 0,
>   celldm(1) = 1.89,
>   nat = 16,
>   ntyp = 3,
>   nspin = 1,
>   nbnd = 120,
>   ecutwfc = 70,
>   ecutrho = 560,
>   tot_charge = 0.0,
>   occupations = 'smearing',
>   smearing = 'gaussian',
>   degauss = 0.005,
>   nosym = .true.,
> /
>
> 
>   electron_maxstep = 1000,
>   conv_thr = 1.D-5,
>   mixing_beta = 0.45,
> /
>
> 
>   ion_dynamics = 'bfgs',
>   ion_temperature = 'andersen',
>   tempw = 300.00 ,
>   nraise = 1,
> /
>
> 
>   cell_dynamics = 'bfgs',
>   press_conv_thr = 0.5,
>   cell_factor = 2.0,
>   cell_dofree = 'all',
> /
>
> ATOMIC_SPECIES
>  O  15.9994  O_pbe_v1.2.uspp.F.UPF
>  V  50.9415  V_pbe_v1.uspp.F.UPF
>  Pb 207.2Pb.pbe-dn-rrkjus_psl.0.2.2.UPF
>
> K_POINTS automatic
>  3 3 3 0 0 0
>
> CELL_PARAMETERS (alat)
>11.5439996719 0.00  0.00
> 0.00 3.571992  0.00
> 0.00 0.00  4.3829998970
>
> ATOMIC_POSITIONS (alat)
>  V6.9405990.8927500.475994
>  V4.6034002.6782503.907006
>  V   10.3754010.8927500.475994
>  V1.1685992.6782503.907006
>  O4.9743090.8927504.370728
>  O6.5696912.6782500.012272
>  O0.7976900.8927504.370728
>  O   10.7463092.6782500.012272
>  O6.9783480.8927502.058695
>  O4.5656522.6782502.324305
>  O   10.3376520.8927502.058695
>  O1.2063492.6782502.324305
>  O8.6580000.8927500.003506
>  O2.8860002.6782504.379493
>  Pb   8.502.501.50
>  Pb   3.001.503.00
>
>
>
>
> ___
> Pw_forum mailing list
> Pw_forum@pwscf.org <mailto:Pw_forum@pwscf.org>
> http://pwscf.org/mailman/listinfo/pw_forum
<http://pwscf.org/mailman/listinfo/pw_forum>
>
>
> ___
> Pw_forum mailing list
> Pw_forum@pwscf.org <mailto:Pw_forum@pwscf.org>
> http://pwscf.org/mailman/listinfo/pw_forum
<http://pwscf.org/mailman/listinfo/pw_forum>



--
Paolo Giannozzi, Dip. Scienze Matematiche Informatiche e Fisiche,
Univ. Udine, via delle Scienze 208, 33100 Udine, Italy
Phone +39-0432-558216, fax +39-0432-558222
___
Pw_forum mailing list
Pw_forum@pwscf.org <mailto:Pw_forum@pwscf.org>
http://pwscf.org/mailman/listinfo/pw_forum
<http://pwscf.org/mailman/listinfo/pw_forum>




--
Isaiah Abu Moses
Graduate Student,
Physics Department,
University of Ibadan,
Nigeria


___
Pw_forum mailing list
Pw_forum@pwscf.org
http://pwscf.org/mailman/listinfo/pw_forum



___
Pw_forum mailing list
Pw_forum@pwscf.org
http://pwscf.org/mailman/listinfo/pw_forum

Re: [Pw_forum] vc-relax cell parameters are not changing

2017-06-22 Thread Stefano de Gironcoli
The lattice parameter does not change, it is not updated along vc-relax, the 
lattice vectors do.
Using bfgs you don't need to define dt

stefano 
(sent from my phone)

> On 23 Jun 2017, at 01:55, Brendan Smith  wrote:
> 
> Hello all,
> 
> I am doing a the following vc-relax calculation and the unit cell paramters 
> are not changing. I have looked at other posts which descirbes similar 
> problems, but it does not remedy my problem. Below is my input.
> Best,
> Brendan
> 
> 
> 
>   calculation = 'vc-relax',
>   dt = 20.67055,
>   nstep = 1000,
>   pseudo_dir = './'
>   outdir = './',
>   prefix = 'x',
>   disk_io = 'low',
> /
> 
> 
>   ibrav = 0,
>   celldm(1) = 1.89,
>   nat = 16,
>   ntyp = 3,
>   nspin = 1,
>   nbnd = 120,
>   ecutwfc = 70,
>   ecutrho = 560,
>   tot_charge = 0.0,
>   occupations = 'smearing',
>   smearing = 'gaussian',
>   degauss = 0.005,
>   nosym = .true.,
> /
> 
> 
>   electron_maxstep = 1000,
>   conv_thr = 1.D-5,
>   mixing_beta = 0.45,
> /
> 
> 
>   ion_dynamics = 'bfgs',
>   ion_temperature = 'andersen',
>   tempw = 300.00 ,
>   nraise = 1,
> /
> 
> 
>   cell_dynamics = 'bfgs',
>   press_conv_thr = 0.5,
>   cell_factor = 2.0,
>   cell_dofree = 'all',
> /  
> 
> ATOMIC_SPECIES
>  O  15.9994  O_pbe_v1.2.uspp.F.UPF
>  V  50.9415  V_pbe_v1.uspp.F.UPF
>  Pb 207.2Pb.pbe-dn-rrkjus_psl.0.2.2.UPF
> 
> K_POINTS automatic
>  3 3 3 0 0 0
> 
> CELL_PARAMETERS (alat)
>11.5439996719 0.00 0.00
> 0.00 3.571992 0.00
> 0.00 0.00 4.3829998970
> 
> ATOMIC_POSITIONS (alat)
>  V6.9405990.8927500.475994
>  V4.6034002.6782503.907006
>  V   10.3754010.8927500.475994
>  V1.1685992.6782503.907006
>  O4.9743090.8927504.370728
>  O6.5696912.6782500.012272
>  O0.7976900.8927504.370728
>  O   10.7463092.6782500.012272
>  O6.9783480.8927502.058695
>  O4.5656522.6782502.324305
>  O   10.3376520.8927502.058695
>  O1.2063492.6782502.324305
>  O8.6580000.8927500.003506
>  O2.8860002.6782504.379493
>  Pb   8.502.501.50
>  Pb   3.001.503.00
> 
> 
> 
> 
> ___
> Pw_forum mailing list
> Pw_forum@pwscf.org
> http://pwscf.org/mailman/listinfo/pw_forum
___
Pw_forum mailing list
Pw_forum@pwscf.org
http://pwscf.org/mailman/listinfo/pw_forum

Re: [Pw_forum] LSDA error

2017-06-21 Thread stefano de gironcoli
It means you started with a magnetic trial solution and at the end of 
the vc-relaxation the final configuration is not magnetic. Is this 
correct or an artifact of the relaxation history ?
To understand this the code performs a new scf calculation in the final 
configuration starting from scratch with the original 
starting_magnetization.
Does the magnetization vanish again and the relaxation was correct or is 
it finite and one should continue the relaxation ? only you knows the 
answer to this question.


stefano

On 21/06/2017 22:10, Manu Hegde wrote:
Okay, thanks. How to avoid that statement?. Which means structure is 
non-magnetic?




On Wed, Jun 21, 2017 at 4:04 PM, stefano de gironcoli 
<degir...@sissa.it <mailto:degir...@sissa.it>> wrote:


this is not an error, it's a statement of what the code does and why.
stefano


On 21/06/2017 22:01, Manu Hegde wrote:


Hello,

I was performing 'vc-relax' calculation (LSDA) i found this
message after few cycles. After performing scf calculaion run
stopped. (using QE 5.4)


 lsda relaxation :  a final configuration with zero
absolute magnetization has been found

 the program is checking if it is really the minimum energy
structure
 by performing a new scf iteration without any "electronic"
history


Thanks
Manu Hegde

(PDF, University of Waterloo)




___
Pw_forum mailing list
Pw_forum@pwscf.org <mailto:Pw_forum@pwscf.org>
http://pwscf.org/mailman/listinfo/pw_forum
<http://pwscf.org/mailman/listinfo/pw_forum>




___
Pw_forum mailing list
Pw_forum@pwscf.org <mailto:Pw_forum@pwscf.org>
http://pwscf.org/mailman/listinfo/pw_forum
<http://pwscf.org/mailman/listinfo/pw_forum>




___
Pw_forum mailing list
Pw_forum@pwscf.org
http://pwscf.org/mailman/listinfo/pw_forum



___
Pw_forum mailing list
Pw_forum@pwscf.org
http://pwscf.org/mailman/listinfo/pw_forum

Re: [Pw_forum] LSDA error

2017-06-21 Thread stefano de gironcoli

this is not an error, it's a statement of what the code does and why.
stefano

On 21/06/2017 22:01, Manu Hegde wrote:


Hello,

I was performing 'vc-relax' calculation (LSDA) i found this message 
after few cycles. After performing scf calculaion run stopped. (using 
QE 5.4)



 lsda relaxation :  a final configuration with zero
absolute magnetization has been found

 the program is checking if it is really the minimum energy structure
 by performing a new scf iteration without any "electronic" history


Thanks
Manu Hegde

(PDF, University of Waterloo)




___
Pw_forum mailing list
Pw_forum@pwscf.org
http://pwscf.org/mailman/listinfo/pw_forum



___
Pw_forum mailing list
Pw_forum@pwscf.org
http://pwscf.org/mailman/listinfo/pw_forum

Re: [Pw_forum] (no subject)

2017-06-14 Thread stefano de gironcoli
what do you want to chose as reference energy ?
Na atom or Na bulk ?
if Na atom, pay attention to the spin state

stefano

On 14/06/2017 17:50, nipesh dulal wrote:
> Hello Everyone,
>
> For the calculation of adsorption energy of sodium atom in graphene, i am 
> using the ground state energy of sodium -95.88087129Ry. Is it correct one? 
> And i have calculated this value removing all cabon atoms in Na adsorbed 
> graphene iput file.
>
> With regards
> NIpesh Dulal
> Kathmandu, Nepal
> ___
> Pw_forum mailing list
> Pw_forum@pwscf.org
> http://pwscf.org/mailman/listinfo/pw_forum


___
Pw_forum mailing list
Pw_forum@pwscf.org
http://pwscf.org/mailman/listinfo/pw_forum


Re: [Pw_forum] how to choose ecutwfc

2017-06-11 Thread stefano de gironcoli

hello Ali Mehdizadeh,
  please sign with your affiliation.
  converged values for ecutwfc and ecutrho do not depend on the 
particle size.

  they depend on the psudopotential used.
stefano
---
Stefano de Gironcoli - SISSA and DEMOCRITOS - Trieste
On 11/06/2017 11:18, ali mehdizadeh wrote:

hello
ecutwfc are dependent  to  the particle size for example the 
difference between quantum dots 3 nm and bulk or ecutwfc just 
dependent to pseudopotentials .

thanks


___
Pw_forum mailing list
Pw_forum@pwscf.org
http://pwscf.org/mailman/listinfo/pw_forum



___
Pw_forum mailing list
Pw_forum@pwscf.org
http://pwscf.org/mailman/listinfo/pw_forum

Re: [Pw_forum] The recalculated pressure is beyond the range specified by press and press_conv_thr

2017-06-07 Thread stefano de gironcoli
the last two pressure values printed
total   stress  (Ry/bohr**3)   (kbar) P= 0.08
total   stress  (Ry/bohr**3)   (kbar) P= 3.17
are the pressure as computed by the code at the end of the vc-relaxation 
with the cutoff sphere deformed according to the cell evolution and the 
pressure computed for the SAME ionic configuration but with the cutoff 
sphere reset to the proper shape.
If the cutoff is high enough (or if the deformation of the cell) is 
small enough these two values should be close.
Increasing the cutoff should make things better.
Alternatively (especially if the cell contracts wrt the initial value) 
one can experiment with the ecfixed,qcutz,q2sigma variables.

stefano

On 07/06/2017 16:45, Tsung-Lung Li wrote:
> Dear QE Friends:
>
> Below is an excerpt of my recent PW simulation,
>
> 
> 
> :
> :
> calculation = 'vc-relax'
> restart_mode = 'from_scratch'
> etot_conv_thr = 1.0D-4
> forc_conv_thr = 1.0D-3
> /
> 
> ion_dynamics = 'bfgs'
> /
> 
> press = 0.001D0  ! kbar
> press_conv_thr = 0.2D0  ! kbar
> /
> 
>
> The simulation converges.
>
> A final scf calculation at the relaxed structure.
> The G-vectors are recalculated for the final unit cell.
>
> The last few lines containing "P=" are
>
> grep "P=" t.log
>
> total   stress  (Ry/bohr**3)   (kbar) P= 0.13
> total   stress  (Ry/bohr**3)   (kbar) P= 0.19
> total   stress  (Ry/bohr**3)   (kbar) P= 0.08
> total   stress  (Ry/bohr**3)   (kbar) P= 3.17
>
> The recalculation suggests a final pressure of 3.17 kbar, which is
> BEYOND the specified range of
> [0.001D0-0.2D0, 0.001D0+0.2D0] = [-0.19900, 0.20100].
>
> Does this mean that the simulation does NOT converge to condition specified
> in  section?  Or, do I make any mistake?
>
> Thank you in advance.
>
>
> Best Regards,
> Tsung-Lung Li
>
>
> ___
> Pw_forum mailing list
> Pw_forum@pwscf.org
> http://pwscf.org/mailman/listinfo/pw_forum


___
Pw_forum mailing list
Pw_forum@pwscf.org
http://pwscf.org/mailman/listinfo/pw_forum


Re: [Pw_forum] relax vs multiple SCF calculations inconsistency

2017-04-19 Thread stefano de gironcoli
then the relaxation found a local minimum which is all it is guaranteed 
to find and by exploring the angular dependence you found a better 
minimum or at least a configuration oh lower energy. If the other 
coordinates were not optimized, starting the relaxation from this 
configuration should lead to a better (local) minimum.
I have no idea whether the 5 mRy=70meV barrier is real or an artifact of 
a noisy landscape due to low cutoff/k-points .

stefano


On 19/04/2017 20:16, sarashs wrote:
> Yes! there is a tiny barrier of 0.005 Ry
>
> Arash
>
> On 2017-04-19 00:20, Stefano de Gironcoli wrote:
>> Is there an energy barrier between the minimum found by relax and the
>> one you find varying the angle ?
>>
>> stefano
>> (sent from my phone)
>>
>>> On 18 Apr 2017, at 21:54, sarashs <sara...@ece.ubc.ca> wrote:
>>>
>>> They use exactly the same cuttoffs, K-points and everything
>>> (unfortunately.) but the near equilibrium energies are slightly less
>>> than equilibrium one. For instance for SiOZr angle at equilibrium I
>>> get:
>>>
>>> !total energy  =-245.22924923 Ry
>>>   Harris-Foulkes estimate   =-245.22924923 Ry
>>>   estimated scf accuracy<  3.9E-12 Ry
>>>
>>>   The total energy is the sum of the following terms:
>>>
>>>   one-electron contribution =-715.79581276 Ry
>>>   hartree contribution  = 369.08543112 Ry
>>>   xc contribution   = -57.16353294 Ry
>>>   ewald contribution= 158.66552815 Ry
>>>   Dispersion Correction =  -0.02086281 Ry
>>>
>>> And for SiOZr angle at equiliberium-25 I get:
>>>
>>> !total energy  =-245.23454839 Ry
>>>   Harris-Foulkes estimate   =-245.23454839 Ry
>>>   estimated scf accuracy<  1.7E-12 Ry
>>>
>>>   The total energy is the sum of the following terms:
>>>
>>>   one-electron contribution =-739.45020087 Ry
>>>   hartree contribution  = 380.81843421 Ry
>>>   xc contribution   = -57.19470926 Ry
>>>   ewald contribution= 170.61520075 Ry
>>>   Dispersion Correction =  -0.02327322 Ry
>>>
>>> Which is similar in terms of total energy but slightly lower and
>>> that's
>>> weird. Is there anything I can do to force QE to use the same basis
>>> set
>>> throughout SCF calculations?
>>>
>>>
>>>>> On Tuesday, 18 April 2017 17:10:25 CEST sarashs wrote:
>>>>> the other SCF's not have higher energy than the
>>>>> equilibrium angle regardless of them being optimized with a
>>>>> constraint?
>>>>> I mean if the structure is originally relax (which it is) then one
>>>>> expects other near equilibrium structures to have higher energies.
>>>>> Am
>>>>> I
>>>>> wrong there?
>>>> You are right.
>>>>
>>>> They should be higher, which usually means less negative. As long as
>>>> the same
>>>> pseudopotentials, cutoffs, k-points and everything else is used.
>>>>
>>>> Do they?
>>> ___
>>> Pw_forum mailing list
>>> Pw_forum@pwscf.org
>>> http://pwscf.org/mailman/listinfo/pw_forum
>>
>> ___
>> Pw_forum mailing list
>> Pw_forum@pwscf.org
>> http://pwscf.org/mailman/listinfo/pw_forum
> ___
> Pw_forum mailing list
> Pw_forum@pwscf.org
> http://pwscf.org/mailman/listinfo/pw_forum


___
Pw_forum mailing list
Pw_forum@pwscf.org
http://pwscf.org/mailman/listinfo/pw_forum


Re: [Pw_forum] Conserving the same Wyckoff multiplicity in the input and in the output

2017-04-19 Thread stefano de gironcoli
a way to avoid the rejection of symmetries with fractionary translation 
without the use_all_frac=.true. flag is to set the nr1,nr2,nr3 
dimensions in such a way that all fractionary translation are such that 
they correspond to translation of an integer number of grid points.

for instance if the FT in crystal coordinates is (1/3, 1/2, 0) then nr1 
must be multiple of 3, nr2 multiple of 2, nr3 is not constrained.
The nr1,nr2,nr3 choice must fulfill the constraints imposed by all the 
FT ... and be compatible with the desired ecutwfc/ecutrho... that is 
larger than the ones that the code would have chosen by default.

stefano

On 19/04/2017 11:47, hqtst42 wrote:
> Dear Paolo,
>
> Many thanks for you reply. It seems like "use_all_frac=.true. " solved
> my problem.
> I have one final question: assuming you use one of the options
> that would prevent you from using (hybrid functionals / phonon
> calculations), how could I change the input parameters (especially
> ecutwfc) so all of the symmetry elements are present ?
>
> Many thanks indeed,
>
> Henri Colaux
>
>
>
> Le 2017/04/19 à 18:46, Henri Colaux a écrit :
>> Le 2017/04/12 à 19:36, Paolo Giannozzi a écrit :
>>> The symmetry the code finds may differ from the actual symmetry of the
>>> system. If so, only a reduced symmetry will be enforced. Note the last
>>> point in this excerpt from the user manual. It holds also for Wyckoff
>>> positions and space groups.
>>>
>>> Paolo
>>>
>>> ===
>>> 5.0.0.19 pw.x does not find all the symmetries you expected
>>>
>>> pw.x determines first the symmetry operations (rotations) of the
>>> Bravais lattice; then checks which of these are symmetry operations of
>>> the system (including if needed fractional translations). This is done
>>> by rotating (and translating if needed) the atoms in the unit cell and
>>> verifying if the rotated unit cell coincides with the original one.
>>>
>>> Assuming that your coordinates are correct (please carefully check!),
>>> you may not find all the symmetries you expect because:
>>>
>>> the number of significant figures in the atomic positions is not large
>>> enough. In file PW/eqvect.f90, the variable accep is used to decide
>>> whether a rotation is a symmetry operation. Its current value (10-5 )
>>> is quite strict: a rotated atom must coincide with another atom to 5
>>> significant digits. You may change the value of accep and recompile.
>>> they are not acceptable symmetry operations of the Bravais lattice.
>>> This is the case for C60 , for instance: the Ih icosahedral group of
>>> C60 contains 5-fold rotations that are incompatible with translation
>>> symmetry.
>>> the system is rotated with respect to symmetry axis. For instance: a
>>> C60 molecule in the fcc lattice will have 24 symmetry operations (Th
>>> group) only if the double bond is aligned along one of the crystal
>>> axis; if C60 is rotated in some arbitrary way, pw.x may not find any
>>> symmetry, apart from inversion.
>>> they contain a fractional translation that is incompatible with the
>>> FFT grid (see next paragraph). Note that if you change cutoff or unit
>>> cell volume, the automatically computed FFT grid changes, and this may
>>> explain changes in symmetry (and in the number of k-points as a
>>> consequence) for no apparent good reason (only if you have fractional
>>> translations in the system, though).
>>> a fractional translation, without rotation, is a symmetry operation of
>>> the system. This means that the cell is actually a supercell. In this
>>> case, all symmetry operations containing fractional translations are
>>> disabled. The reason is that in this rather exotic case there is no
>>> simple way to select those symmetry operations forming a true group,
>>> in the mathematical sense of the term.
>>>
>>> 5.0.0.20 Warning: symmetry operation # N not allowed
>>>
>>> This is not an error. If a symmetry operation contains a fractional
>>> translation that is incompatible with the FFT grid, it is discarded in
>>> order to prevent problems with symmetrization. Typical fractional
>>> translations are 1/2 or 1/3 of a lattice vector. If the FFT grid
>>> dimension along that direction is not divisible respectively by 2 or
>>> by 3, the symmetry operation will not transform the FFT grid into
>>> itself. Solution: you can either force your FFT grid to be
>>> commensurate with fractional translation (set variables nr1, nr2, nr3
>>> to suitable values), or set variable use_all_frac to .true., in
>>> namelist  Note however that the latter is incompatible with
>>> hybrid functionals and with phonon calculations.
>>> ===
>>>
>>>
>>> On Wed, Apr 12, 2017 at 12:03 PM, hqtst42  wrote:
 Hi Paolo,

 Many thanks for your reply ; maybe the problem may be something
 different ; I see a symmetry break from the gipaw simulation. Because of
 the symmetry, I expect, for example, 4 carbons with identical 

Re: [Pw_forum] relax vs multiple SCF calculations inconsistency

2017-04-19 Thread Stefano de Gironcoli
Is there an energy barrier between the minimum found by relax and the one you 
find varying the angle ?

stefano 
(sent from my phone)

> On 18 Apr 2017, at 21:54, sarashs  wrote:
> 
> They use exactly the same cuttoffs, K-points and everything 
> (unfortunately.) but the near equilibrium energies are slightly less 
> than equilibrium one. For instance for SiOZr angle at equilibrium I get:
> 
> !total energy  =-245.22924923 Ry
>  Harris-Foulkes estimate   =-245.22924923 Ry
>  estimated scf accuracy<  3.9E-12 Ry
> 
>  The total energy is the sum of the following terms:
> 
>  one-electron contribution =-715.79581276 Ry
>  hartree contribution  = 369.08543112 Ry
>  xc contribution   = -57.16353294 Ry
>  ewald contribution= 158.66552815 Ry
>  Dispersion Correction =  -0.02086281 Ry
> 
> And for SiOZr angle at equiliberium-25 I get:
> 
> !total energy  =-245.23454839 Ry
>  Harris-Foulkes estimate   =-245.23454839 Ry
>  estimated scf accuracy<  1.7E-12 Ry
> 
>  The total energy is the sum of the following terms:
> 
>  one-electron contribution =-739.45020087 Ry
>  hartree contribution  = 380.81843421 Ry
>  xc contribution   = -57.19470926 Ry
>  ewald contribution= 170.61520075 Ry
>  Dispersion Correction =  -0.02327322 Ry
> 
> Which is similar in terms of total energy but slightly lower and that's 
> weird. Is there anything I can do to force QE to use the same basis set 
> throughout SCF calculations?
> 
> 
>>> On Tuesday, 18 April 2017 17:10:25 CEST sarashs wrote:
>>> the other SCF's not have higher energy than the
>>> equilibrium angle regardless of them being optimized with a 
>>> constraint?
>>> I mean if the structure is originally relax (which it is) then one
>>> expects other near equilibrium structures to have higher energies. Am 
>>> I
>>> wrong there?
>> 
>> You are right.
>> 
>> They should be higher, which usually means less negative. As long as 
>> the same
>> pseudopotentials, cutoffs, k-points and everything else is used.
>> 
>> Do they?
> ___
> Pw_forum mailing list
> Pw_forum@pwscf.org
> http://pwscf.org/mailman/listinfo/pw_forum


___
Pw_forum mailing list
Pw_forum@pwscf.org
http://pwscf.org/mailman/listinfo/pw_forum


Re: [Pw_forum] Pulay Stress

2017-03-22 Thread stefano de gironcoli

yes and no.

By default the computed stress is the derivative of the energy at 
constant number of PW and the Pulay stress is not estimated.
You should check the PW basis is large enough that the stress is 
converged (hence Pulay stress is negligible).
An alternative is to use the smooth-cutoff trick implemented by the 
variables

ECFIXED, QCUTZ,Q2SIGMA (in the  namelist)
See M. Bernasconi et al, J. Phys. Chem. Solids 56, 501 (1995),
doi:10.1016/0022-3697(94)00228-2 for an explanation

stefano


On 22/03/2017 12:30, Umesh Roy wrote:

Dear Q.E users,

   Does the Q.E code calculate Pulay stress?


Thank you all.
/*




-

Umesh Chandra Roy*
Research Scholar, School of Physical Sciences
Jawaharlal Nehru University, New Delhi-110067,
/
/India.
/
/Email:*umesh24...@gmail.com* 
/
/Mobile:*+919868022722*/


___
Pw_forum mailing list
Pw_forum@pwscf.org
http://pwscf.org/mailman/listinfo/pw_forum



___
Pw_forum mailing list
Pw_forum@pwscf.org
http://pwscf.org/mailman/listinfo/pw_forum

Re: [Pw_forum] gk_sort igk(ig) versus igk_k(ig, ik) possible bug in old versions of wfck2r.f90?

2017-03-02 Thread Stefano de Gironcoli
What is passed is the address of the first element of the array. The routine 
expects a vector and will take three elements starting from there which is 
correct

stefano 
(sent from my phone)

> On 2 Mar 2017, at 12:46, aritz leonardo  wrote:
> 
> Dear all,
> 
> I have just successfully installed qe-6.0 and I realized this version, the 
> k+G to G correspondence has changed. The old igk(ig) has been changed for 
> igk_k.
> My question is regarding the program "wfck2r.f90" of the PP section which 
> transforms the wave-function from k-space to r.
>  wfck2r.f90 from  qe-5.3.0 (and previous versions) reads:
>   DO ik = 1,nks
>  !
>  !prepare the indices of this k point
>  !
>  CALL gk_sort (xk (1, ik), ngm, g, ecutwfc / tpiba2, npw, &
>   igk, g2kin)
>  
>  CALL davcio (evc, 2*nwordwfc, iunwfc, ik, - 1)
> 
>  do ibnd=1,nbnd 
>  !
>  ! I perform fourier transform
>  !
> evc_r = cmplx(0.d0, 0.d0) 
> do ig = 1, npw
>evc_r (nls (igk (ig) ),1 ) = evc (ig,ibnd)
> enddo
> CALL invfft ('Wave', evc_r(:,1), dffts)
> 
> 
> 
> Shouldn't one pass  to gk_sort the hole k point instead of the first 
> coordinate? i.e. xk(:,ik) instead of xk(1,ik) 
> 
> (I am aware that in qe-6.0 this part has been replaced with igk_k.)
> 
> Thanks in advance!
> 
> -- 
> 
> 
> Aritz Leonardo Liceranzu
> Department of Applied Physics II,
> Faculty of Science and Technology,
> University of the Basque Country (UPV/EHU)
> Bº Sarriena s/n, 48940 Leioa, Spain
> 
> http://fisica.ehu.es/~aritz   Fax: +34-946013500
> Mail: aritz.leona...@ehu.es   Phone: +34-946015338
>  
> ___
> Pw_forum mailing list
> Pw_forum@pwscf.org
> http://pwscf.org/mailman/listinfo/pw_forum
___
Pw_forum mailing list
Pw_forum@pwscf.org
http://pwscf.org/mailman/listinfo/pw_forum

Re: [Pw_forum] Units in Vc-relax

2017-02-18 Thread stefano de gironcoli

On 18/02/2017 16:47, Manu Hegde wrote:

Hello All,

I have performed some vc-relax calculations at the end I am getting,

(criteria: energy <  1.0E-04, force <  1.0E-03, cell <  5.0E-01)

ryd , ryd/a.u., kbar

HTH

stefano


What are the units of above quantities?. Hope I will get some answer.

Thanks and Regards,
Manu

(University of Waterloo)


___
Pw_forum mailing list
Pw_forum@pwscf.org
http://pwscf.org/mailman/listinfo/pw_forum



___
Pw_forum mailing list
Pw_forum@pwscf.org
http://pwscf.org/mailman/listinfo/pw_forum

Re: [Pw_forum] Kind of Stress tensor

2017-02-17 Thread stefano de gironcoli
the stress in the code is defined as the derivative of the energy per 
unit volume with respect to infinitesimal deformations

sigma_alpha,beta = 1/Omega partial E / partial epsilon_alpha,beta
for a deformation such that
r_alpha -> r'_alpha = r_alpha + sum_beta epsilon_alpha,beta r_beta
it is symmetric in alpha,beta because the antisymmetric components of 
the deformation (describing rotations) do not change the energy.


If I understand correctly the stress definitions 
(https://en.wikipedia.org/wiki/Stress_measures) for infinitesimal 
deformations they are all the same

because J=1, F=I, dn_0=dn, ... etc

If you are considering finite deformations the stress is the derivative 
taken w.r.t. the present deformed configuration so I think must be the 
Cauchy stress because the code does not know you want to take another 
configuration as reference.

Therefore, should be something like  S = J (F^-1) sigma  (F^-1)^T

HTH

stefano



On 17/02/2017 16:27, Mahdi Faghihnasiri wrote:

Dear all,

I know If I set tstress=.true. the stress tensor will be computed and
printed. but What's the kind of the Stress Tensor QE dose print? for example 
Cauchy,
  true, Kirchhoff, PK2, ...
I am trying to calculate second Piola–Kirchhoff (PK2) stresses with QE. Any 
suggestions or comments are appreciated.

Sincerely
Mahdi
*Mahdi FaghihNasiri*
Department of Physics
Shahrood University of Technology
Shahrood,Iran**


___
Pw_forum mailing list
Pw_forum@pwscf.org
http://pwscf.org/mailman/listinfo/pw_forum



___
Pw_forum mailing list
Pw_forum@pwscf.org
http://pwscf.org/mailman/listinfo/pw_forum

[Pw_forum] Summer School on Atomistic Simulation Techniques, Trieste, 14-30 June 2017

2017-02-08 Thread Stefano de Gironcoli
Dear Collegues,

we are pleased to announce the 7th edition of Summer School on
Atomistic Simulation Techniques to be held in Trieste, on 14-30 June
2017

The International School for Advanced Studies (SISSA) and CNR-IOM
DEMOCRITOS Simulation Centre, Trieste (Italy), organize a CECAM Summer
School on Atomistic Simulation Techniques, which this year will be
dedicated to quantum mechanical methods and chemical and biochemical
applications.

The School is mainly directed to undergraduate and graduate students,
with little or no experience in computer simulations.

The purpose of the School will be threefold: (i) providing students
with a basic but detailed overview of the theoretical foundations and
numerical methods of quantum mechanical (QM) simulations of molecular
and extended systems; (ii) training them to solve quantum mechanical
problems in practice, either implementing simple numerical codes from
scratch or using existing QM codes for selected applications; (iii)
giving an overview of the domains of current interesting re
search problems in material and biochemical science.

At the end of the school the students should have a clear idea of the
importance of quantum mechanical molecular simulations; they should be
aware of the problems that are still open and are at the center of
current research efforts and should have the capability of developing
their own simple code for performing a simulation or an analysis.

For interested students with no previous experience in computational
science it is possible to attend a three-day pre-school (14 - 16 June)
devoted to the introduction to programming in C++/Fortran, with few
numerical exercises.

In 2017 the School will be entirely focused on quantum mechanics and
in particular will be devoted to electronic properties of materials,
chemical- and bio-molecules. The duration of the School is two weeks
(plus the pre-school)

To apply visit the School web site  http://democritos.sissa.it/school2017/
and fill the application form http://www.democritos.it/school2017/appform.php

Deadline for Applications
The closing date for receipt of requests for participation is 31 March 2017

A registration fee (200 Euro) will be required to the accepted
candidates and reimbursed to students who will have fully attended the
school.
The organizers will cover accommodation (14 nights), lunches and
dinners at SISSA (Monday to Friday), public transportation and social
dinner. Lodging expenses will be covered also during the pre-school.
Limited support toward travel expenses can be provided in exceptional
cases, based on fund availability.

The organisers
Stefano de Gironcoli (SISSA)
Andrea Dal Corso (SISSA)
Alessandra Magistrato (CNR-IOM @ SISSA)


___
Pw_forum mailing list
Pw_forum@pwscf.org
http://pwscf.org/mailman/listinfo/pw_forum

Re: [Pw_forum] A question about tot_charge flag with DFT+U calculations

2017-01-13 Thread stefano de gironcoli

Dear Kosuke Nakano,
   the total charge option acts on deciding the number of states used 
and how to fix the fermi energy/chemical potential.
   the occupation matrix return the projection of the occupied manifold 
(determined by the previous recipe) on the localized states so it should 
be what one expects it to be.
   so yes it looks like it's Ni ~ +2 (d8) but with partial occupation 
of the eg orbitals and NOT spin polarized... Is this what you 
expect/desire ?.
Did you suggest a starting magnetization to see it the system does not 
prefer being d5_up d3_dw ?


stefano

On 13/01/2017 05:26, kosuke.nak...@agc.com wrote:


Hi, all:

I have a question about tot_charge flag with DFT+U calculations

I would like to calculate d state of transition metal with tot_charge 
flag.


I know that tot_charge flag with DFT+U calculations is valid as 
written in




I am wondering if occupation matrices are also valid in tot_charge

flag with DFT+U calculations.

For example, I obtained the following occupation matrices with a 
finite tot_charge(+4)


The system has 24 atoms and 4 of them are Ni atoms.

--The final result--

atom   1   Tr[ns(na)] (up, down, total) =   4.26150  4.08319  8.34469 
<- Ni (+2, d8?)


   spin  1

eigenvalues:

  0.658  0.658  0.981 0.982  0.982

eigenvectors:

  0.000  0.000  0.996 0.003  0.002

  0.588  0.074  0.000 0.291  0.046

  0.074  0.588  0.001 0.046  0.291

  0.038  0.300  0.002 0.089  0.571

  0.300  0.038  0.001 0.571  0.090

occupations:

  0.981 -0.000 -0.000 -0.000 -0.000

-0.000  0.768  0.000 -0.000  0.153

-0.000  0.000  0.768 0.153 -0.000

-0.000 -0.000  0.153 0.873 -0.000

-0.000  0.153 -0.000 -0.000  0.873

   spin  2

eigenvalues:

  0.649  0.649  0.923 0.931  0.931

eigenvectors:

  0.000  0.000  1.000 0.000  0.000

  0.649  0.018  0.000 0.078  0.255

  0.018  0.649  0.000 0.255  0.078

  0.009  0.324  0.000 0.510  0.157

  0.324  0.009  0.000 0.157  0.510

occupations:

  0.923  0.000  0.000 0.000  0.000

  0.000  0.743  0.000 0.000  0.133

  0.000  0.000  0.743 0.133  0.000

  0.000  0.000  0.133 0.837  0.000

  0.000  0.133  0.000 0.000  0.837

atomic mag. moment = 0.178309

I am wondering if it means Ni(+2, d8) or not.

Best regards

Kosuke Nakano

Asahi Glass Co., Japan.



___
Pw_forum mailing list
Pw_forum@pwscf.org
http://pwscf.org/mailman/listinfo/pw_forum



___
Pw_forum mailing list
Pw_forum@pwscf.org
http://pwscf.org/mailman/listinfo/pw_forum

Re: [Pw_forum] URGENT

2017-01-08 Thread stefano de gironcoli
there are a few options. from the simplest makov-payne energy correction 
to more elaborated martyna-tuckerman or ems corrections.
you can check the assume_isolated keyword in the  namelist 
descritpion

http://www.quantum-espresso.org/wp-content/uploads/Doc/INPUT_PW.html

stefano


On 08/01/2017 22:20, ashkan shekaari wrote:

Dear experts,

In dealing with charged unit cells, what kind of scheme is applied in 
quantum espresso to reduce periodic-image errors?



Regards,
Ashkan/

/
*Ashkan Shekaari*
Plasma Physics Research Center
Science and Research Branch
I A U, 14778-93855 Tehran, Iran.


___
Pw_forum mailing list
Pw_forum@pwscf.org
http://pwscf.org/mailman/listinfo/pw_forum



___
Pw_forum mailing list
Pw_forum@pwscf.org
http://pwscf.org/mailman/listinfo/pw_forum

Re: [Pw_forum] Problem: different output with identical input

2016-12-28 Thread Stefano de Gironcoli
Dear Pablo Garcia Risueño

I'm not sure I understand the problem.
the two final positions for ecut=30 are identical to my eyes.
the two final positions for ecut=60 differ by about 10-5 A. !
If the property you are interested in depends so strongly on the  
atomic positions you better learn how to live with it. Even assuming  
that you are using perfect xc functional, perfect pseudopotentials and  
converged cutoff and k-points (which likely you don't), your  
calculation is neglecting zero point energy, thermal expansion,  
quantum nature of H motion, ...

As for your specific question. In principle if you tighten enough  
the convergence the minima should tend to converge toward a common  
value. However the stopping criteria will make the two calculations  
stop as soon they are satisfied. In your case as soon as each force  
component is lower than 10-6 Ry/au and the energy does not change more  
that 10-8. Did the calculation complete ? 10-8 for the energy is  
easily reachable but 10-6 for the forces looks to me very demanding.

  Anyhow... even if the relaxation did complete successfully two  
equivalent calculations may still differ. If you run your calculations  
with exactly the same input on exactly the same machine with exactly  
the same mpi setup the results should be identical ..but why doing  
twice exactly the same calculation ?. If however you change anything  
in the input (Davidson vs CG, starting wavefunction options, mixing  
mode, BFGS-related parameters) or parallel computational setup the  
relaxation history will be slightly different and the final  
configurations will differ by a certain amount allowed by the  
tolerance defined by your stopping criteria.

  HTH

stefano

Quoting Pablo García Risueño :

> Dear professor
>
> Thank you very much for your reply. The differences are important in this
> case, my final calculated quantities are very sensitive to these optimized
> positions. Should I change any of the ***conv_thr variables, or other
> variable, to have the same result for same inputs?
>
> Is there any random number in the algorithm of relax calculations, can we
> be sure that different outputs with the same input are not due to any bug?
>
> Thank you very much. Best regards.
>
> 2016-12-28 14:10 GMT+01:00 Paolo Giannozzi :
>
>> The differences you find are very small. Nothing to worry about in my
>> opinion.
>>
>> By the way: Davidson diagonalization is typically faster than CG; do not
>> specify incompatible options in K_POINTS (gamma or automatic, not both;
>> gamma should be used unless you have a good reason not to)
>>
>> Paolo
>>
>> Il 28/dic/2016 01:46 PM, "Pablo García Risueño" 
>> ha scritto:
>> >
>> > Dear Espresso community
>> >
>> > I have one problem that is important for me; it is somewhat surprising.
>> I run geometry optimization (relax) calculations with pw.x with identical
>> input files, and I obtain rather different final coordinates. The problem
>> does not happen if ecutwfc is 30, but it does appear for cutoffs of 60, 80
>> or 90. Below one can see the exact input file, and examples of the
>> difference between the final coordinates for both runs (both run with
>> identical input) for given cutoffs.
>> >
>> > Could anybody give me a clue on the origin of the problem, and how to
>> solve it?
>> >
>> > Thank you very much. Best regards.
>> >
>> >
>> >
>> > Input file:
>> >
>> >
>> >
>> > 
>> >
>> > calculation = 'relax',
>> >
>> > restart_mode = 'from_scratch',
>> >
>> > prefix='',
>> >
>> > outdir = './',
>> >
>> > pseudo_dir = '/path_xxx/PP/',
>> >
>> > forc_conv_thr = 1.0D-6 ,
>> >
>> > etot_conv_thr = 1.0D-8 ,
>> >
>> >  /
>> >
>> > 
>> >
>> > ibrav = 0, a=18.0,
>> >
>> > nat= 26, ntyp= 2,
>> >
>> > ecutwfc = 30d0,
>> >
>> > nbnd = 100,
>> >
>> > /
>> >
>> > 
>> >
>> > conv_thr = 1.0e-9,
>> >
>> > mixing_beta = 0.7,
>> >
>> > mixing_mode = 'plain',
>> >
>> > diagonalization = 'cg'
>> >
>> > /
>> >
>> > 
>> >
>> > /
>> >
>> >
>> >
>> > ATOMIC_SPECIES
>> >
>> > C   12.0107   C.pz-vbc.UPF
>> >
>> > H   1.007825035  H.pz-vbc.UPF
>> >
>> >
>> >
>> > ATOMIC_POSITIONS { angstrom }
>> >
>> > C  8.891700e+00 8.891700e+00 8.891700e+00
>> >
>> > C  9.783400e+00 9.783400e+00 8.00e+00
>> >
>> > C  9.783400e+00 8.00e+00 9.783400e+00
>> >
>> > C  8.00e+00 9.783400e+00 9.783400e+00
>> >
>> > C  8.891700e+00 1.067510e+01 1.067510e+01
>> >
>> > C  1.067510e+01 1.067510e+01 8.891700e+00
>> >
>> > C  1.067510e+01 8.891700e+00 1.067510e+01
>> >
>> > C  9.783400e+00 1.156680e+01 9.783400e+00
>> >
>> > C  9.783400e+00 9.783400e+00 1.156680e+01
>> >
>> > C  1.156680e+01 9.783400e+00 9.783400e+00
>> >
>> > H  8.391700e+00 8.391700e+00 8.391700e+00
>> >
>> > H  7.50e+00 1.028340e+01 9.283400e+00
>> >
>> > H  

Re: [Pw_forum] ​propagation direction of LO-TO splitting for dynmat.in file phonon calculation

2016-12-24 Thread stefano de gironcoli

Dear Wei Li,

  from the header of the dynmat.f90 source code
!
!  This program
!  - reads a dynamical matrix file produced by the phonon code
!  - adds the nonanalytical part (if Z* and epsilon are read from file),
!applies the chosen Acoustic Sum Rule (if q=0)
!  - diagonalise the dynamical matrix
!  - calculates IR and Raman cross sections (if Z* and Raman tensors
!are read from file, respectively)
!  - writes the results to files, both for inspection and for plotting
!
!  Input data (namelist "input")
!
!  fildyn  character input file containing the dynamical matrix
!(default: fildyn='matdyn')
!  q(3)  realcalculate LO modes (add nonanalytic terms) along
!the direction q (cartesian axis, default: q=(0,0,0) )

the value of the vector q (when different from zero) is interpreted as 
the direction of propagation of the phonons.


stefano

On 25/12/2016 02:20, Wei Li wrote:

​​
​Hi Friends,

I plan to calculate the infrared spectrum of anatase TiO2 bulk using 
pw.x (scf), ph.x calculates phonon frequencies and effective charge 
etc..  Then the dynmat.x is used to calculate LO-TO splitting.


The ph.x calculation is performed at single q, i.e., q=0.0,0.0,0.0 in 
ph.in  file


​
The question is how to define the
​​
propagation direction of LO-TO splitting, q(i), i=1,2,3, in the 
dynmat.in  file?


For instance, how to set q(1),q(2),q(3)?​






--
*Wei Li*
PhD student
State Key Lab of Theoretical & Computational Chemistry
Institute of Theoretical Chemistry
Jilin University
Changchun, 130023
P.R. China
Email: liwei0...@gmail.com 


___
Pw_forum mailing list
Pw_forum@pwscf.org
http://pwscf.org/mailman/listinfo/pw_forum



___
Pw_forum mailing list
Pw_forum@pwscf.org
http://pwscf.org/mailman/listinfo/pw_forum

Re: [Pw_forum] Run using a relaxed structure as initial one NOT converge in fir st few ionic steps.

2016-12-20 Thread stefano de gironcoli
dear Tsung-Lung Li,

Your expectations are correct.
If this is what you experience you probably are not doing what you think 
you are doing but it is difficult to make any suggestion without further 
information

stefano


On 20/12/2016 10:26, quantum wrote:
> Dear QE Friends:
>
> I did a "relax" calculation with PW (version 5.3.0)
> and obtained the relaxed atomic positions.
>
> Then, I took the atomic positions
> in the last "ATOMIC_POSITIONS (crystal)"
> section of the output file to replace the
> atomic positions in the original input file
> without changing any other things except new
> prefix and outdir.
>
> The atoms in the relaxed structure should
> all satisfy etot_conv_thr and forc_conv_thr
> thresholds.  With this in mind, I started
> the new run.
>
> I expected the run to
> converge after the first or two ionic steps.
> But it did not stop, and continued running
> for many ionic steps without convergence.
>
> Is my expectation correct? or it is a problem
> of QE?
>
> Sincerely,
> Tsung-Lung Li
>
>
> Tsung-Lung Li, Ph. D.
> Professor
> Department of Applied Physics
> National Chia-Yi University
> 300 Hsueh-Fu Road, Chiayi 60004, Taiwan
> Phone: 886-5-2717904.  FAX: 886-5-2717909.
> E-mail:quan...@mail.ncyu.edu.tw
> URL:http://web.ncyu.edu.tw/~quantum
>
> ___
> Pw_forum mailing list
> Pw_forum@pwscf.org
> http://pwscf.org/mailman/listinfo/pw_forum


___
Pw_forum mailing list
Pw_forum@pwscf.org
http://pwscf.org/mailman/listinfo/pw_forum


Re: [Pw_forum] 0

2016-12-09 Thread Stefano de Gironcoli
With ibrav=0 you are on your own. Which usually is not too bad if you know what 
you're doing 

stefano 
(sent from my phone)

> On 09 Dec 2016, at 18:44, ashkan shekaari  wrote:
> 
> Dear experts, 
> 
> For the case of ibrav=0, how one could name high-symmetry points in the BZ?
> --
> Regards,
> Ashkan Shekaari
> Plasma Physics Research Center
> Science and Research Branch
> I A U, 14778-93855 Tehran, Iran.
> ___
> Pw_forum mailing list
> Pw_forum@pwscf.org
> http://pwscf.org/mailman/listinfo/pw_forum
___
Pw_forum mailing list
Pw_forum@pwscf.org
http://pwscf.org/mailman/listinfo/pw_forum

Re: [Pw_forum] VC-relax collapsing unit cell

2016-11-18 Thread Stefano de Gironcoli
I dont know if this is the problem but the last two oxygens are 4c not 8d

stefano 
(sent from my phone)

> On 19 Nov 2016, at 00:25, John Bilgerman  wrote:
> 
> Hi,
> 
> I've been banging my head against this and cannot find what is likely a silly 
> mistake despite many tests and lots of reading.
> 
> I'm trying to optimize the (known) structure of NaFePO4 as a test. I'm 
> starting from the experimental crystal structure, so the drastic collapse of 
> the unit cell to < 1/2 suggests an issue.
> 
> I know the common problem is inputing the structure wrong, but I've done my 
> best (and sanity-checked the input/output files with Xcrysden).
> 
> I'm new to QE, any help would be appreciated.
> 
> Input file:
>  
>  calculation = 'vc-relax' ,
> restart_mode = 'from_scratch' ,
>   outdir = './' ,
>   wfcdir = './scratch' ,
>   pseudo_dir = './pseudo' ,
>  disk_io = 'default' ,
>verbosity = 'high' ,
>  /
>  
>ibrav = 8,
>  space_group = 62 ,
>A = 9.001 ,
>B = 6.874 ,
>C = 5.052 ,
>cosAB = 0 ,
>cosAC = 0 ,
>cosBC = 0 ,
>  nat = 6,
> ntyp = 4,
>  ecutwfc = 35 ,
>  ecutrho = 140 ,
>  occupations = 'smearing' ,
>  degauss = 0.02 ,
> smearing = 'gaussian' ,
>nspin = 2 ,
>starting_magnetization(1) = 0.7,
>starting_magnetization(2) = 0,
>starting_magnetization(3) = 0,
>starting_magnetization(4) = 0,
> noncolin = .false. ,
>  /
>  
>  diagonalization = 'david' ,
>  /
>  
>  /
>  
>  /
> ATOMIC_SPECIES
>Fe   55.0  Fe.pbe-sp-hgh.upf 
> P   30.0  P.pbe-hgh.upf 
>Na   22.0  Na.pbe-sp-hgh.upf 
> O   16.0  O.pbe-hgh.upf 
> ATOMIC_POSITIONS crystal_sg 
> Fe 4a 
> P 4c 0.17585  0.46447
> Na 4c  0.34999  0.9702
> O 8d 0.1212 0.0682 0.3177
> O 8d 0.3486 0.25 0.4561
> O 8d 0.1154 0.25 0.7507
> K_POINTS automatic 
>   2 3 4   1 1 1 
> 
> 
> The relevant parts of the CIF file for the structure are:
> ...
> _cell_length_a 9.001(8)
> _cell_length_b 6.874(3)
> _cell_length_c 5.052(4)
> _cell_angle_alpha 90.
> _cell_angle_beta 90.
> _cell_angle_gamma 90.
> _cell_volume 312.58
> _cell_formula_units_Z 4
> _symmetry_space_group_name_H-M 'P n m a'
> _symmetry_Int_Tables_number 62
> ...
> Fe1 Fe2+ 4 a 0 0 0 . 1. 0
> P1 P5+ 4 c 0.17585(4) 0.25 0.46447(8) . 1. 0
> Na1 Na1+ 4 c 0.34999(9) 0.25 0.9702(2) . 1. 0
> O1 O2- 8 d 0.1212(1) 0.0682(1) 0.3177(2) . 1. 0
> O2 O2- 4 c 0.3486(1) 0.25 0.4561(2) . 1. 0
> O3 O2- 4 c 0.1154(1) 0.25 0.7507(2) . 1. 0
> ...
> 
> John
> ___
> Pw_forum mailing list
> Pw_forum@pwscf.org
> http://pwscf.org/mailman/listinfo/pw_forum


___
Pw_forum mailing list
Pw_forum@pwscf.org
http://pwscf.org/mailman/listinfo/pw_forum


Re: [Pw_forum] The coordinate of X-point in GaAs band structure calculation

2016-10-31 Thread stefano de gironcoli

Dear Evan,

On 31/10/2016 08:51, evan wrote:


Dear Stefano

Thank you for your reply, but I do not understand what’s you meaning 
in the last letter.


What I want to know is that when the lattice structure of GaAs (fcc) 
is different from the simulation cell (sc, ibrav =1 in my 
calculations), the coordinates of high symmetry points in Brillioun 
zone of which lattice structure (fcc or those of sc) I should specify 
in the PWscf input file?


I am not familiar with this issue, but it is important for my study. 
Thank you for your patience.




then I would suggest that you get familiar with the concepts of real and 
reciprocal space, supercells and refolding of points in the Brillouin zone.
In your calculation 16 valence bands are computed. Did you wonder why 
and what they are ?

The coordinates of the X point are not your problem.

regards,

stefano
---
On two occasions I have been asked [by members of Parliament!], "Pray, Mr.
Babbage, if you put into the machine wrong figures, will the right answers
come out?"  I am not able rightly to apprehend the kind of confusion of
ideas that could provoke such a question.
-- Charles Babbage



Regards

Evan

USC, China





At 2016-10-31 14:32:53, "Stefano de Gironcoli" <degir...@sissa.it> wrote:

The one you are using: ibrav=1.
Do not expect to see a band structure similar to the usual one tho

stefano
(sent from my phone)

On 31 Oct 2016, at 03:03, evan <ewa...@126.com
<mailto:ewa...@126.com>> wrote:


Dear Stefano and Andrew

Thank you for your kind replies. Let me ask a more general
question which I think about.

I want to calculate the band structure of GaAs (FCC structure) in
a simple cubic lattice cell with eight atoms (ibrav =1). If I
specify the k points in units of 2pi/a (K_POINTS tpiba_b), the
high symmetry points in Brillioun zone of which cell I should
specify in the input file, FCC (ibrav =2) or those of SC
(ibrav=1) lattice structure?

If I specify the k points in in crystal coordinates (K_POINTS
crystal_b), then of which cell I should specify the k points in
the input files, FCC or SC lattice structure.

Thank you in advance.

Regards

Evan

USC, China


    At 2016-10-30 02:17:33, "stefano de gironcoli" <degir...@sissa.it
<mailto:degir...@sissa.it>> wrote:

Dear Evan,
   GaAS structure is zincblend (2 atoms per cell). its X
point is indeed 2pi/a(1,0,0).
   if you choose to compute GaAs in a SC cell (8 atoms pr
cell) then that point is indeed equivalent to Gamma. The zone
boundary of the SC cell is pi/a(1,0,0) but this of course
does not correspond to the X point of the zincblend structure.
  HTH
 stefano

On 29/10/2016 16:00, evan wrote:


Hello, everyone

I calculated the band structure of GaAs in a simple cubic
lattice cell (ibrav=1), I searched from the internet the
coordinates of X-point in the first Brillouin zone are
2pi/a(0,0,1) or 2pi/a(1,0,0), but both results show that
band gap occurs at G and X points simultaneously, it seems
to me that the X is equivalent to G, can you tell me the
correct X point coordinates which give me the reasonable
result. The input files and results are attached, you can
show what is wrong in the input file.

Your suggestions are appreciated.

Yours sincerely

Evan

USC, China







___
Pw_forum mailing list
Pw_forum@pwscf.org
http://pwscf.org/mailman/listinfo/pw_forum





___
Pw_forum mailing list
Pw_forum@pwscf.org <mailto:Pw_forum@pwscf.org>
http://pwscf.org/mailman/listinfo/pw_forum






___
Pw_forum mailing list
Pw_forum@pwscf.org
http://pwscf.org/mailman/listinfo/pw_forum



___
Pw_forum mailing list
Pw_forum@pwscf.org
http://pwscf.org/mailman/listinfo/pw_forum

Re: [Pw_forum] The coordinate of X-point in GaAs band structure calculation

2016-10-31 Thread Stefano de Gironcoli
The one you are using: ibrav=1.
Do not expect to see a band structure similar to the usual one tho

stefano 
(sent from my phone)

> On 31 Oct 2016, at 03:03, evan <ewa...@126.com> wrote:
> 
> Dear Stefano and Andrew
> 
> Thank you for your kind replies. Let me ask a more general question which I 
> think about.
> 
>  I want to calculate the band structure of GaAs (FCC structure) in a simple 
> cubic lattice cell with eight atoms (ibrav =1). If I specify the k points in 
> units of 2pi/a (K_POINTS tpiba_b), the high symmetry points in Brillioun zone 
> of which cell I should specify in the input file, FCC (ibrav =2) or those of 
> SC (ibrav=1) lattice structure?
> 
> If I specify the k points in in crystal coordinates (K_POINTS crystal_b), 
> then of which cell I should specify the k points in the input files, FCC or 
> SC lattice structure.
> 
> Thank you in advance.
> 
> Regards
> 
> Evan
> 
> USC, China
> 
> 
> At 2016-10-30 02:17:33, "stefano de gironcoli" <degir...@sissa.it> wrote:
> Dear Evan,
>GaAS structure is zincblend (2 atoms per cell). its X point is indeed 
> 2pi/a(1,0,0).
>if you choose to compute GaAs in a SC cell (8 atoms pr cell) then that 
> point is indeed equivalent to Gamma. The zone boundary of the SC cell is 
> pi/a(1,0,0) but this of course does not correspond to the X point of the 
> zincblend structure.
>   HTH
>  stefano
> 
>> On 29/10/2016 16:00, evan wrote:
>> Hello, everyone
>> 
>> I calculated the band structure of GaAs in a simple cubic lattice cell 
>> (ibrav=1), I searched from the internet the coordinates of X-point in the 
>> first Brillouin zone are 2pi/a(0,0,1) or 2pi/a(1,0,0), but both results show 
>> that band gap occurs at G and X points simultaneously, it seems to me that 
>> the X is equivalent to G, can you tell me the correct X point coordinates 
>> which give me the reasonable result. The input files and results are 
>> attached, you can show what is wrong in the input file.
>> 
>> Your suggestions are appreciated.
>> 
>> Yours sincerely
>> 
>> Evan
>> 
>> USC, China 
>> 
>> 
>> 
>>  
>> 
>> 
>> 
>>  
>> 
>> 
>> 
>> ___
>> Pw_forum mailing list
>> Pw_forum@pwscf.org
>> http://pwscf.org/mailman/listinfo/pw_forum
> 
> 
> 
>  
> 
> ___
> Pw_forum mailing list
> Pw_forum@pwscf.org
> http://pwscf.org/mailman/listinfo/pw_forum
___
Pw_forum mailing list
Pw_forum@pwscf.org
http://pwscf.org/mailman/listinfo/pw_forum

Re: [Pw_forum] Error related to "radial FFT during vc-relax" calculations

2016-10-28 Thread stefano de gironcoli

what type of vc-relax are you doing ?
I mean: it's a CNT... if you don't fix the xy lattice spacing the cell 
will shrink significantly and the code will beg for a larger 
cell_factor... which you could try to give it if this is really what you 
want to do ... but is it ?

HTH
stefano

On 28/10/2016 08:51, Sai Phani wrote:

Dear all,
I am trying to perform "vc-relax" pwscf calculations for the 54 atomed 
Carbon Nanotube (CNT) molecule. I had performed these calculations at 
various k-points of 5*5*1, 9*9*1, 11*11*1 and 20*20*1 using 
Monkhorst-Pack grid scheme.


But, every time the calculations were stopped in a midway with 
generating the CRASH file.

The CRASH file consists of the following error:

%%
 task # 0
 from scale_h : error # 1
 Not enough space allocated for radial FFT: try restarting with a 
larger cell_factor.

 %%

 %%
 task # 3
 from scale_h : error # 1
 Not enough space allocated for radial FFT: try restarting with a 
larger cell_factor.

 %%

 %%
 task # 1
 from scale_h : error # 1
 Not enough space allocated for radial FFT: try restarting with a 
larger cell_factor.

 %%

I am currently using Ecutwfc = 30 Ry and rhocut_wfc = 300 Ry for this 
system.



Could any of you tell me the reason for the error and what k-points 
and Ecut_wfc and rho_wfc values should I use in my input file?





Thanks & Regards,
Sai Phani Kumar,
Research Scholar (PhD),
Quantum and Molecular Engineering Laboratory,
Department of Chemical Engineering,
Indian Institute of Technology Kharagpur.
West Bengal, India-721302.




___
Pw_forum mailing list
Pw_forum@pwscf.org
http://pwscf.org/mailman/listinfo/pw_forum



___
Pw_forum mailing list
Pw_forum@pwscf.org
http://pwscf.org/mailman/listinfo/pw_forum

Re: [Pw_forum] Errors with structure optimization

2016-10-26 Thread stefano de gironcoli
I think this happens because the cell shrinks very much and the 
effective cutoff grows above beyond a certain fraction (set with the 
cell_factor) of the input cutoff.

restarting after setting a larger cell_factor should work.
restarting from scratch from the latest volume should also work.

stefano


On 26/10/2016 10:38, Oanh Nguyen wrote:

Dear users!
I am optimising "vc_relax" for PdxTaSe2. I have a problem when I am 
running.
Not enough space allocated for radial FFT: try restarting with a 
larger cell_factor.

Running with "relax" is ok

Here is my inputfile.
I already use lattice constant from experimental result.

How to the way to fix it?


___
Pw_forum mailing list
Pw_forum@pwscf.org
http://pwscf.org/mailman/listinfo/pw_forum



___
Pw_forum mailing list
Pw_forum@pwscf.org
http://pwscf.org/mailman/listinfo/pw_forum

Re: [Pw_forum] Phonon frequency: nat_todo option

2016-10-21 Thread stefano de gironcoli

Dear Vikan,
I dont have direct experience on this problem. here a few things i would 
look into:


from your NEB calculation around the TS you should be able to see what 
the unstable mode is expected to look like and how many atoms are involved.
If you compute interatomic force constants (IFC) only around a subset of 
the atoms there is the problem of how you impose the acoustic sum rule.
One way would be to complete your DynMat using the IFC of the bulk 
material (that you could compute separately), and impose ASR on this model.
To stay on the safe side you could set the mass of the atoms that you 
did not compute to a very large number so they do not move.


stefano


On 21/10/2016 17:38, Vikan Manmathan wrote:

Dear Developers and Experts,

I am interested in studying diffusion of metal clusters on oxide 
support. As a first step, I took a simplest case (adatom hopping) and 
obtained the transition state using CI-NEB method.  I want to find the 
rate constant for the above, I need to know the normal mode 
vibrational frequencies at initial and transition states.


So I calculated the phonon frequencies using frozen atomic coordinate 
(nat_todo option only on metal adatom) since the system size is very 
large.  I expected the transition state to have one less real 
frequencies (3N-1) relative to initial state (3N). However, I am 
getting 3N real frequencies for the transition state (correct me if I 
am wrong).


 I have also checked it by including more atoms in the linear response 
(up to 33 atoms) but still I got 3N real frequencies for the TS.


I have attached the input files and the output file generated by dynmat.x

Any suggestion will be highly appreciated.

Thanks in advance.

regards,

*NANDHA KUMAR V.*
Research scholar,
IISER, PUNE.


___
Pw_forum mailing list
Pw_forum@pwscf.org
http://pwscf.org/mailman/listinfo/pw_forum



___
Pw_forum mailing list
Pw_forum@pwscf.org
http://pwscf.org/mailman/listinfo/pw_forum

Re: [Pw_forum] Convergence of LaCoO3

2016-10-10 Thread stefano de gironcoli
dear Amita Sihag

is it reasonable that La-O and Sr-O distances are only slightly above 1 AA ?
is it reasonable to assume starting magnetization on all atoms including 
O and Pt ?
is it reasonable to use occupation fixed in this non collinear magnetic 
system ?
how important is non-collinearity in the system ?
I would start by optimizing the system in the collinear case, even 
starting with the undoped case to see where it wants to go and add 
complications one at at time..
HTH
stefano
PS: i would not insist beyond a few tens of electronic iteration, let's 
say 100 if the system is magnetic... If it does not converge this means 
there is something wrong in the system.

On 10/10/2016 09:31, Amita Sihag wrote:
>
> 
>title = 'LaCoO3' ,
>   prefix = 'LaCoO3'
>  calculation = 'vc-relax' ,
>  max_seconds = 1.0d+7 ,
>   wf_collect = .TRUE. ,
> restart_mode = 'restart'
>   outdir = 
> '/home/SSM_grp/amitasihag/LaCoO3/221/doping/SrPt_mag/out' ,
>   pseudo_dir = './' ,
> /
> 
>ibrav = 0,
> celldm(1)= 1.89 ,
>  nat = 40,
> ntyp = 5 ,
> noncolin = .true. ,
>starting_magnetization(1) = 1.5 ,
>starting_magnetization(2) = 1.5 ,
>starting_magnetization(3) = 1.0 ,
>starting_magnetization(4) = 1.5 ,
>starting_magnetization(5) = 1.5 ,
>  ecutwfc = 30 ,
>  ecutrho = 300 ,
>  occupations = 'fixed'
> /
> 
> electron_maxstep = 1500 ,
> conv_thr = 1.0d-4 ,
>  mixing_mode = 'plain' ,
>  mixing_beta = 0.2 ,
>  diagonalization = 'david' ,
>
> /
> 
> /
> 
> /
> CELL_PARAMETERS alat
>10.737711   0.00   0.00
> 5.2412480519   9.3742687151   0.00
> 2.6206240259   1.5372037335   4.4278926328
>
> ATOMIC_SPECIES
>Sr   87.62  Sr.pbe-nsp-van.UPF
>Pt   195.08 Pt.pbe-n-van.UPF
>La   138.91 La.pbe-nsp-van.UPF
>Co   58.933 Co.pbe-sp-van.UPF
>O15.999 O.pbe-van_ak.UPF
>
> ATOMIC_POSITIONS alat
> Sr   2.652812105   1.556084500   1.106973092
> Pt   7.990623911   4.687134358   0.0
> La   5.273436131   6.243218857   1.106973092
> La   8.022811831   1.556084500   1.106973092
> La  10.643435857   6.243218857   1.106973092
> La   7.958435953   4.668253568   3.320919475
> La  10.579059979   9.355387926   3.320919475
> La  13.328435839   4.668253568   3.320919475
> La  15.949059865   9.355387926   3.320919475
> Co   0.0   0.0   0.0
> Co   2.620624026   4.687134358   0.0
> Co   5.36886   0.0   0.0
> Co   5.305624211   3.112169000   2.213946184
> Co   7.926248237   7.799303357   2.213946184
> Co  10.675623776   3.112169000   2.213946184
> Co  13.296247802   7.799303357   2.213946184
> O7.414334322   5.595837165   3.320919475
> O   10.034958348  10.282971523   3.320919475
> O   12.784333887   5.595837165   3.320919475
> O   15.404957913  10.282971523   3.320919475
> O3.196913420   0.628500623   1.106973092
> O5.817537290   5.315634702   1.106973092
> O8.566913626   0.628500623   1.106973092
> O   11.187537496   5.315634702   1.106973092
> O7.958435953   4.044882307   4.197199414
> O   10.579059823   8.732016385   4.197199414
> O   13.328435839   4.044882307   4.197199414
> O   15.949059708   8.732016385   4.197199414
> O2.652812135   2.179455779   0.230693202
> O5.273436317   6.866590416   0.230693202
> O8.022811860   2.179455779   0.230693202
> O   10.643436042   6.866590416   0.230693202
> O8.502537428   4.364040954   2.444639535
> O   11.123161454   9.051175311   2.444639535
> O   13.872537634   4.364040954   2.444639535
> O   16.493161660   9.051175311   2.444639535
> O2.108710530   1.860297068   1.983253098
> O4.729334595   6.547431495   1.983253098
> O7.478710676   1.860297068   1.983253098
> O   10.099334741   6.547431495   1.983253098
>
> K_POINTS automatic
> 2 2 4  0 0 0


___
Pw_forum mailing list
Pw_forum@pwscf.org
http://pwscf.org/mailman/listinfo/pw_forum


Re: [Pw_forum] QE output about stress and pressure

2016-10-09 Thread stefano de gironcoli
Dear Tsung-Lung Li,
stress is given in Cartesian coordinates.
stefano

On 09/10/2016 05:40, Tsung-Lung Li wrote:
> Dear Stefano:
>
> Thank you for your answers.
>
> The crystal I am working on is monoclinic.
>
> Is the stress given in QE outout the components on the three adjacent
> planes of a monoclinic lattice?  or on the three perpendicular Cartesian
> planes of a cube?
>
> Tsung-Lung Li
>
> On 10/08/2016 09:12 PM, stefano de gironcoli wrote:
>> it's the current estimate of the pressure
>> 1/3 of the trace of the stress tensor
>> stefano
>>
>> On 08/10/2016 15:03, quantum wrote:
>>> Dear QE Friends:
>>>
>>> I am working on the high-pressure mechanical characteristics of
>>> crystals.
>>>
>>> At the end of a scf computation, the following lines are in the
>>> output file,
>>>
>>> 
>>>
>>>entering subroutine stress ...
>>>
>>> total   stress  (Ry/bohr**3) (kbar) P=  241.85
>>>  0.00051810  -0.  -0.8740 76.22 -0.00-12.86
>>> -0.   0.00304364   0. -0.00 447.74  0.00
>>> -0.8740   0.   0.00137038-12.86 0.00201.59
>>> 
>>>
>>>
>>> Two questions:
>>>
>>> (1) What is the meaning of "P=  241.85"?
>>> (2) Could you please point me to the proper references to understand
>>> the meanings
>>>   of the above output lines?
>>>
>>> Thank you in advance.
>>>
>>> Sincerely,
>>> Tsung-Lung Li
>>>
>>>
>>> Tsung-Lung Li, Ph. D.
>>> Professor
>>> Department of Applied Physics
>>> National Chia-Yi University
>>> 300 Hsueh-Fu Road, Chiayi 60004, Taiwan
>>> Phone: 886-5-2717904.  FAX: 886-5-2717909.
>>> E-mail:quan...@mail.ncyu.edu.tw
>>> URL:http://web.ncyu.edu.tw/~quantum
>>>
>>> ___
>>> Pw_forum mailing list
>>> Pw_forum@pwscf.org
>>> http://pwscf.org/mailman/listinfo/pw_forum
>>
>>

___
Pw_forum mailing list
Pw_forum@pwscf.org
http://pwscf.org/mailman/listinfo/pw_forum


Re: [Pw_forum] QE output about stress and pressure

2016-10-08 Thread stefano de gironcoli
it's the current estimate of the pressure
1/3 of the trace of the stress tensor
stefano

On 08/10/2016 15:03, quantum wrote:
> Dear QE Friends:
>
> I am working on the high-pressure mechanical characteristics of crystals.
>
> At the end of a scf computation, the following lines are in the output file,
>
> 
>   entering subroutine stress ...
>
>total   stress  (Ry/bohr**3)   (kbar) P=  
> 241.85
> 0.00051810  -0.  -0.8740 76.22 -0.00-12.86
>-0.   0.00304364   0. -0.00447.74  0.00
>-0.8740   0.   0.00137038-12.86  0.00201.59
> 
>
> Two questions:
>
> (1) What is the meaning of "P=  241.85"?
> (2) Could you please point me to the proper references to understand the 
> meanings
>  of the above output lines?
>
> Thank you in advance.
>
> Sincerely,
> Tsung-Lung Li
>
>
> Tsung-Lung Li, Ph. D.
> Professor
> Department of Applied Physics
> National Chia-Yi University
> 300 Hsueh-Fu Road, Chiayi 60004, Taiwan
> Phone: 886-5-2717904.  FAX: 886-5-2717909.
> E-mail:quan...@mail.ncyu.edu.tw
> URL:http://web.ncyu.edu.tw/~quantum
>
> ___
> Pw_forum mailing list
> Pw_forum@pwscf.org
> http://pwscf.org/mailman/listinfo/pw_forum


___
Pw_forum mailing list
Pw_forum@pwscf.org
http://pwscf.org/mailman/listinfo/pw_forum


Re: [Pw_forum] (no subject)

2016-10-06 Thread stefano de gironcoli

don't really know anymore... Quantum ESPRESSO 6.0 has been just released.
stefano

On 06/10/2016 20:01, Kanak Datta wrote:

Dear researchers

Can Quantum Espresso 3.0 can be run with mpi distribution provided by 
microsoft i.e. MSMPI? In that case, will the command be the same?


Best
Kanak

--
Kanak Datta
Lecturer
Department of Electrical and Electronic Engineering
Bangladesh University of Engineering and Technology
Dhaka-1205
Website: https://sites.google.com/site/kanak0806085/home


___
Pw_forum mailing list
Pw_forum@pwscf.org
http://pwscf.org/mailman/listinfo/pw_forum



___
Pw_forum mailing list
Pw_forum@pwscf.org
http://pwscf.org/mailman/listinfo/pw_forum

  1   2   3   4   5   6   7   >